HESI Exit Exam Questions

अब Quizwiz के साथ अपने होमवर्क और परीक्षाओं को एस करें!

Which intervention is most important for the nurse to include in the plan of care for an older woman with osteoporosis?

Place the client on fall precautions

A 10 year old who has terminal brain cancer asks the nurse, "What will happen to my body when I die?" How should the nurse respond?

"The heart will stop beating & you will stop breathing."

A school-age child who weighs 42 pounds receives a post-tonsillectomy prescription for promethazine (Phenergan) 0.5 mg/kg IM to prevent postoperative nausea. The medication is available in 25 mg/ml ampules. How many ml should the nurse administer? (Enter numeric value only. If rounding is required, round to the nearest tenth).

0.4 Rationale: Convert pounds to kg 42lbs = 19.09 kg Next calculate to prescribed dose, 0.5 mg x 1909 kg = 9.545 Then use the desired dose/ dose on hand x volume on hand (9.545/25x1ml =0.3818=0.4 ml) Or use ratio proportion (9.545 mg: x ml = 25 mg: 1ml 25x = 9.545 X= 0.3818 = 0.4)

The healthcare provider prescribes Morphine Sulfate Oral Solution 38 mg PO q4 hours for a client who is opioid-tolerant. The available 30 mL bottle is labeled, 100 mg/5 mL (20mg/mL), and is packaged with a calibrated oral syringe to provide to provide accurate dose measurements. How many mL should the nurse administer? (Enter the numerical value only. If rounding is required, round to the nearest tenth.)

1.9 Rationale: D/H x Q 38/20x1=1.9 mL

A client is receiving an IV solution labeled Heparin Sodium 20,000 Units in 5% dextrose injection 500 ml at 25 ml/hour. How many units of heparin is the client receiving each hour?

1000 units/hour Rationale:20000/500=40x25=1000

A client currently receiving an infusion labeled Heparin Sodium 25,000 Units in 5% Dextrose Injection 500 mL at 14 mL/hour. A prescription is received to change the rate of the infusion to 900 units of Heparin per hour. The nurse should set the infusion pump to deliver how many mL/hour? (Enter numeric value only).

18 Rationale: 450000/25000=18

The nurse notes the client receiving heparin infusion labeled, Heparin Na 25,000 Units in 5% Dextrose injection 500 ml at 50ml/hr. What dose of Heparin is the client receiving per hour?

2,500

A client is receiving an IV solution of nitroglycerin 100mg/500ml D5W at 10 mcg/ minute. The nurse should program the infusion pump to deliver how many ml/hour? ( Enter numeric value only)

3 ml/hour Rationale : 0.01 x 500 x 60 / 100 = 3

The nurse mixes 250 mg of debutamine in 250 ml of D5W and plans to administer the solution at rate client weighing 110 pounds. The nurse should set the infusion pump to administer how many ml per hour only. If rounding is required, round the nearest whole number.)

45

Dopamine protocol is prescribed for a male client who weigh 198 pounds to maintain the mean arterial pressure (MAP) greater than 65 mmHg. His current MAP is 50 mmHg, so the nurse increases the infusion to 7 mcg/kg/minute. The infusion is labeled dextrose 5% in water (D5W) 500 ml with dopamine 400 mg. The nurse should program the infusion pump to deliver how many ml/hour?

47

A 154 pound client with diabetic ketoacidosis is receiving an IV of normal saline 100 ML with regular insulin 100 units. The healthcare provider prescribes a rate of 0.1 units/kg/hour. To deliver the correct dosage, the nurse should set the infusion pump to Infuse how many ml/hour? enter numeric value only

7 Rationale: Convert the client's weight to kg, 2.2 pound: 1 kg:: 154 pounds: x kg = 154/2.2 = 70kg. Calculate the client infusion rate, 0.1 x 70 kg = 7 units/hour. Using the formula, D/H x Q = 7 units/hour / 100 units x 100 ml = 7ml / hour

A client is receiving an IV of heparin sodium 25000 units in 5% dextrose injection 500 ml at 14 ml/hour...verify that the client is receiving the prescribed amount of heparin. How many units is the client receiving?

700 Rationale: 25000/500x14=700

A female client reports that she drank a liter of a solution to cleanse her intestines...immediately. How many ml of fluid intake should the nurse document? Whole number

760 Rationale: 1L=1000ml Subtract the emesis, 1 cup (8 oz)=240ml 1000-240=760 ml

A nurse with 10 years experience working in the emergency room is reassigned to the perinatal unit to work an 8 hour shift. Which client is best to assign to this nurse?

A mother with an infected episiotomy

A client with a serum sodium level of 125 meq/mL should benefit most from the administration of which intravenous solution? A. 0.9% sodium chloride solution (normal saline) B. 0.45% sodium chloride solution (half normal saline) C. 10% Dextrose in 0.45% sodium chloride D. 5% dextrose in 0.2% sodium chloride

A. 0.9% sodium chloride solution (normal saline)

The nurse is assigned to care for clients on a medical unit. Based on the notes taken during the shift report, which client situation warrants the nurse's immediate attention? A. A 10-year-old who is receiving chemotherapy and the infusion pump is beeping B. A young adult with Crohn's disease who reports having diarrheal stools C. An older adult with type 2 diabetes whose breakfast tray arrives 20 minutes late D. A teenager who reports continued pain 30 minutes after receiving an oral analgesic

A. A 10-year-old who is receiving chemotherapy and the infusion pump is beeping Rationale: an infiltration of a caustic agent can cause tissue damage and children are at greater risk for fluid volume imbalances

Which location should the nurse choose as the best for beginning a screening program for hypothyroidism? A. A business and professional women's group. B. An African-American senior citizens center C. A daycare center in a Hispanic neighborhood D. An after-school center for Native-American teens

A. A business and professional women's group.

A 6 -years-old who has asthma is demonstrating a prolonged expiratory phase and wheezing, and has 35% personal best peak expiratory flow rate (PEFR). Based on these finding, which action should the nurse implement first? A. Administer a prescribed bronchodilator. B. Report finding to the healthcare provider. C. Encourage the child to cough and deep breath D. Determine what trigger precipitated this attack.

A. Administer a prescribed bronchodilator. Rationale: If the PEFR is below 50% in as asthmatic child, there is severe narrowing of the airway, and a bronchodilator should be administered immediately. Be should be implemented after A. C will not alleviate the symptoms and D is not a priority.

The nurse assesses a client with new onset diarrhea. It is most important for the nurse to question the client about recent use of which type of medication? A. Antibiotics B. Anticoagulants C. Antihypertensive D. Anticholinergics

A. Antibiotics

A client has a prescription for lorazepam 2mg for alcohol withdrawal symptoms. Which finding... the client? A. Blood pressure 149/101 B. Irregular pulse rate of 80 C. Oral temperature is 98.9 F (37.1 C) D. Pain rated 7 on scale 1-10

A. Blood pressure 149/101

A client is receiving oxytocin (Pitocin) to augment early labor. Which assessment is most important time the infusion rate is increases? A. Contraction pattern B. Blood pressure C. Infusion site D. Pain level

A. Contraction pattern

The nurse receives a newborn within the first minutes after a vaginal delivery and intervenes to establish adequate respirations. What priority issue should the nurse address to ensure the n ewborn's survival? A. Heat loss B. Hypoglycemia C. Fluid balance D. Bleeding tendencies

A. Heat loss

A group of nurse-managers is asked to engage in a needs assessment for a piece of equipment that will be expensed to the organization's budget. Which question is most important to consider when analyzing the cost-benefit for this piece of equipment? A. How many departments can use this equipment? B. Will the equipment require annual repair? C. Is the cost of the equipment reasonable? D. Can the equipment be updated each year?

A. How many departments can use this equipment?

A nurse is caring for a client with Diabetes Insipidus. Which assessment finding warrants immediate intervention by the nurse? A. Hypernatremia B. Excessive thirst C. Elevated heart rate D. Poor skin turgor

A. Hypernatremia

A client is admitted to the intensive care unit with diabetes insipidus due to a pituitary gland tumor. Which potential complication should the nurse monitor closely? A. Hypokalemia B. Ketonuria. C. Peripheral edema D. Elevated blood pressure

A. Hypokalemia Rational: pituitary tumors that suppress antidiuretic hormone (ADH) result in diabetes insipidus, which causes massive polyuria and serum electrolyte imbalances, including hypokalemia, which can lead to lethal arrhythmias.

A male client arrives at the clinic with a severe sunburn and explains that he did not use sun screen because it was an overcast day. Large blisters are noted over his back and chest and his shirt is soaked with serosanguinous fluid. Which assessment finding warrants immediate intervention by the nurse? A. Hypotension. B. Fever and chills C. Dizziness D. Headache

A. Hypotension.

During orientation, a newly hired nurse demonstrates suctioning of a tracheostomy in a skills class, as seen in the video. After the demonstration, the supervising nurse expresses concern that the demonstrated procedure increased the client's risk for which problem? A. Infection B. Ineffective airway clearance C. Altered comfort D. Impaired gas exchange

A. Infection

An infant born to a heroin-addicted mother is admitted to the neonatal care unit. What behaviors can...to exhibit? A. Irritability and a high-pitched cry B. Lethargy and poor suck C. Facial abnormalities and microcephaly D. Low birth weight and intrauterine growth retardation

A. Irritability and a high-pitched cry

A client is admitted to isolation with the diagnosis of active tuberculosis (TB). Which infection control measures should the nurse implement? A. Negative pressure environment B. Contact precautions C. Droplet precautions D. Protective environment

A. Negative pressure environment

A client is admitted to isolation with the diagnosis of active tuberculosis. Which infection control measures should the nurse implement? A. Negative pressure environment B. Contact precautions C. Droplet precautions D. Protective environment

A. Negative pressure environment

Two days after admission a male client remembers that he is allergic to eggs, and informs the nurse of the allergy. Which actions should the nurse implement? (Select all that apply)

A. Notify the food services department of the allergy. B. Enter the allergy information in the client's record. C. Add egg allergy to the client's allergy arm band.

When changing a diaper on a 2-day-old infant, the nurse observes that the baby's legs are... this finding, what action should the nurse take next? A. Notify the healthcare provider B. Continue care since this is a normal finding C. Document the finding in the record D. Perform range of motion to the joint

A. Notify the healthcare provider

An adult male with schizophrenia who has been noncompliant in taking oral antipsychotic medications refuses a prescribed IM medication. Which action should the nurse take? A. Notify the healthcare provider of the client's refusal B. Administer an oral PRN medication for agitation C. Ask for staff assistance with administering the injection D. explain that oral medications will no longer be required

A. Notify the healthcare provider of the client's refusal

The nurse is assessing and elderly bedridden client. Which finding indicates that the turning and positioning schedule is effective in protecting the client's skin? A. Reddened skin areas disappear within 15 minutes of being turned and positioned. B. No complaints of pressure or pain are verbalized by the client after being turned C. Only small areas of redness remain longer than 30 min after the client is turned. D. The client verbalizes feeling better after being turned and positioned

A. Reddened skin areas disappear within 15 minutes of being turned and positioned.

A male client receives a thrombolytic medication following a myocardial infarction. When the client has a bowel movement, what action should the nurse implement? A. Send stool sample to the lab for a guaiac test B. Observe stool for a day-colored appearance. C. Obtain specimen for culture and sensitivity analysis D. Asses for fatty yellow streaks in the client's stool.

A. Send stool sample to the lab for a guaiac test Rationale: Thrombolytic drugs increase the tendency for bleeding. So guaiac (occult blood test) test of the stool should be evaluated to detect bleeding in the intestinal tract.

***...An Insulin infusion for a client with diabetes mellitus who is experiencing hyperglycemic hyperosmolar...in addition to the client's glucose, which laboratory value is most important for the nurse to monitor? A. Serum potassium B. Urine ketones C. Urine albumin D. Serum protein

A. Serum potassium

A client is admitted to a mental health unit after attempting suicide by taking a handful of medications. In developing a plan of care for this client, which goal has the highest priority? A. Signs a no-self-harm contract. B. Sleep at least 6 hours nightly. C. Attends group therapy every day D. Verbalizes a positive self-image.

A. Signs a no-self-harm contract.

An older male client with a history of type 1 diabetes has not felt well the past few days and arrives at the clinic with abdominal cramping and vomiting. He is lethargic, moderately, confused, and cannot remember when he took his last dose of insulin or ate last. What action should the nurse implement first? A. Start an intravenous (IV) infusion of normal saline B. obtain a serum potassium level C. administer the client's usual dose of insulin D. assess pupillary response to light

A. Start an intravenous (IV) infusion of normal saline Rationale: the nurse should first start an intravenous infusion of normal saline to replace the fluids and electrolytes because the client has been vomiting, and it is unclear when he last ate or took insulin. The symptoms of confusion, lethargy, vomiting, and abdominal cramping are all suggestive of hyperglycemia, which also contributes to diuresis and fluid electrolyte imbalance.

When administering ceftriaxone sodium (Rocephin) intravenously to a client before...most immediate intervention by the nurse? A. Stridor B. Nausea C. Headache D. Pruritis

A. Stridor

The nurse is assisting a new mother with infant feeding. Which information should the nurse provide that is most likely to result in a decrease milk supply for the mother who is breastfeeding? A. Supplemental feedings with formula B. Maternal diet high in protein C. Maternal intake of increased oral fluid D. Breastfeeding every 2 or 3 hours.

A. Supplemental feedings with formula Rationale: Infant sucking at the breast increases prolactin release and proceeds a feedback mechanism for the production of milk, the nurse should explain that supplemental bottle formula feeding minimizes the infant's time at the breast and decreases milk supply. B promotes milk production and healing after delivery. C support milk production. C is recommended routine for breast feeding that promote adequate milk supply.

An older client is admitted for repair of a broken hip. To reduce the risk for infection in the postoperative period, which nursing care interventions should the nurse include in the client's plan of care? (Select all that apply) A. Teach client to use incentive spirometer q2 hours while awake. B. Remove urinary catheter as soon as possible and encourage voiding. C. Maintain sequential compression devices while in bed. D. Administer low molecular weight heparin as prescribed E. Assess pain level and medicate PRN as prescribed.

A. Teach client to use incentive spirometer q2 hours while awake. B. Remove urinary catheter as soon as possible and encourage voiding.

A client with a large pleural effusion undergoes a thoracentesis. Following the procedure, which assessment finding warrants immediate intervention by the nurse? A. The client has asymmetrical chest wall expansion B. The clients complain of pain at the insertion site C. The client chest's x-ray indicates decreased pleural effusion D. The client's arterial blood gases are pH 7.35, PaO2 85, Pa CO2 35, HCO3 26

A. The client has asymmetrical chest wall expansion Rationale: A potential complication of thoracentesis is a pneumothorax. The symptoms of a pneumothorax are uneven, unequal movement of the chest wall. A is an expected finding after the local anesthetic effects "wear off" B is a desired result of thoracentesis and C is within normal limits.

The nurse performs a prescribed neurological check at the beginning of the shift on a client who was admitted to the hospital with a subarachnoid brain attack (stroke). The client's Glasgow Coma Scale (GCS) score is 9. What information is most important for the nurse to determine? A. The client's previous GCS score B. When the client's stroke symptoms started C. If the client is oriented to time D. The client's blood pressure and respiration rate

A. The client's previous GCS score Rationale: The normal GCS is 15, and it is most important for the nurse to determine if it abnormal score a sign of improvement or a deterioration in the client's condition

A mother brings her 4-month-old son to the clinic with a quarter taped over his umbilicus, and tells the nurse the quarter is supposed to fix her child's hernia. Which explanations should the nurse provide? A. This hernia is a normal variation that resolves without treatment. B. Restrictive clothing will be adequate to help the hernia go away. C. An abdominal binder can be worn daily to reduce the protrusion. D. The quarter should be secured with an elastic bandage wrap.

A. This hernia is a normal variation that resolves without treatment. Rational: an umbilical hernia is a normal variation in infants that occurs due to an incomplete fusion of the abdominal musculature through the umbilical ring that usually resolves spontaneously as the child learns to walk. Other choices are ineffective and unnecessary.

A 2-year-old is bleeding from a laceration on the right lower extremity that occurred as the result of a motor vehicle collision. The nurse is selecting supplies to start an IV access. Which assessment finding is most significant in the nurse's selection of catheter size? A. Thready brachial pulse. B. Respirations of 24/minute C. Right foot cool to touch D. Swelling at the site of injury

A. Thready brachial pulse.

Following and gunshot wound, an adult client a hemoglobin level of 4 grams/dl (40 mmol/L SI). The nurse prepares to administer a unit of blood for an emergency transfusion. The client has AB negative blood type and the blood bank sends a unit of type A Rh negative, reporting that there is not type AB negative blood currently available. Which intervention should the nurse implement? A. Transfuse Type A negative blood until type AB negative is available. B. Recheck the client's hemoglobin, blood type and Rh factor. C. Administer normal saline solution until type AB negative is available D. Obtain additional consent for administration of type A negative blood

A. Transfuse Type A negative blood until type AB negative is available. Rationale: those who have type AB blood are considered universal recipients using A or B blood types that is the same Rh factor. The client's hemoglobin is critically low and the client should receive a unit of blood that is type A, which must be Rh negative blood. Other options are not indicated in this situation.

A woman who takes pyridostigmine for myasthenia gravis (MG) arrives at the emergency department complaining of extreme muscle weakness. Her adult daughter tells the nurse that since yesterday her mother has been unable to smile, which assessment finding warrants immediate intervention by the nurse? A. Uncontrollable drooling B. Inability to raise voice C. Tingling of extremities D. Eyelid drooling

A. Uncontrollable drooling

After removing a left femoral arterial sheath, which assessment finding warrant immediately interventions by the nurse? (Select all that applied.) A. Unrelieved back and flank pain. B. Quarter-size red drainage at site C. Cool and pale left leg and foot. D. Tenderness over insertion site E. Left groin egg-size hematoma.

A. Unrelieved back and flank pain. C. Cool and pale left leg and foot. E. Left groin egg-size hematoma.

A client in septic shock has a double lumen central venous catheter with one liter of 0.9% Normal Saline Solution infusing at 1 ml/hour through one lumen and TPN infusing at 50 ml/hr. through one port. The nurse prepared newly prescribed IV antibiotic that should take 45 mints to infuse. What intervention should the nurse implement? A. Use a secondary port of the Normal Saline solution to administer the antibiotic. B. Add the antibiotic to the TPN solution, and continue the normal saline solution. C. Stop the TPN infusion for the time needed to administer the prescribed antibiotic. D. Add the antibiotic to the Normal Saline solution and continue both infusions.

A. Use a secondary port of the Normal Saline solution to administer the antibiotic. Rationale: A client in septic shock needs antibiotic administered in a timely manner to ensure maintenance of therapeutic serum level. The nurse should administer the antibiotic using a secondary port of the Normal Saline solution. No other medications should be administered using TPN tubing or solution. TPN not should be place on hold because sudden cessation will cause rapid change in serum glucose levels. Excessively delays in the administration of the antibiotics.

A client is discharged with automated peritoneal dialysis (PD) to be used nightly...which instructions should the nurse include? A. Wash hands before cleaning exit site B. Keep the head of the bed flat at night C. Feel for a thrill and a distal pulse nightly D. Do not get up if fluid is left in the abdomen

A. Wash hands before cleaning exit site

In caring for a client who is receiving linezolid IV for nosocomial pneumonia, which assessment finding is most important for the nurse to report to the healthcare provider? A. Watery diarrhea B. Yellow-tinged sputum C. Increased fatigue D. Nausea and headache

A. Watery diarrhea

A client is being discharged home after being treated for heart failure (HF). What instruction should the nurse include in this client's discharge teaching plan? A. Weigh every morning B. Eat a high protein diet C. Perform range of motion exercises D. Limit fluid intake to 1,500 ml daily

A. Weigh every morning

When planning care for a client with acute pancreatitis, which nursing intervention has the highest priority? A. Withhold food and fluid intake. B. Initiate IV fluid replacement. C. Administer antiemetic as needed. D. Evaluate intake and output ratio.

A. Withhold food and fluid intake. Rational: The pathophysiologic processes in acute pancreatitis result from oral fluid and ingestion that causes secretion of pancreatic enzymes, which destroy ductal tissue and pancreatic cells, resulting in auto digestion and fibrosis of the pancreas. The main focus of the nursing care is reducing pain caused by pancreatic destruction through interventions that decrease GI activity, such as keeping the client NPO. Other choices are also important intervention but are secondary to pain management.

The healthcare provider prescribes the antibiotic Cefdinir (cephalosporin) 300mg PO every 12 h for a client with postoperative wound infections. Which foods should the nurse encourage this client to eat? A. Yogurt and/or buttermilk. B. Avocados and cheese C. Green leafy vegetables D. Fresh fruits

A. Yogurt and/or buttermilk.

When conducting diet teaching for a client who was diagnosed with hypoparathyroidism, which foods should the nurse encourage the client to eat? A. Yogurt. B. Processed cheese. C. Nuts D. Fresh turkey E. Fresh chicken

A. Yogurt. B. Processed cheese.

A postoperative female client has a prescription for morphine sulfate 10 mg IV q3 hours for pain. One dose of morphine was administered when the client was admitted to the post anesthesia care unit (PACU) and 3 hours later, the client is again complaining of pain. Her current respiratory rate is 8 breaths/minute. What action should the nurse take?

Administer Naxolone IV

The nurse caring for a 3-month-old boy one day after a pylorotomy notices that the infant is restless, is exhibiting facial grimaces, and is drawing his knees to his chest. What action should the nurse take?

Administer a prescribed analgesia for pain.

A client was admitted to the cardiac observation unit 2 hours ago complaining of chest pain. On admission, the client's EKG showed bradycardia, ST depression, but no ventricular ectopy. The client suddenly reports a sharp increase in pain, telling the nurse, "I feel like an elephant just stepped on my chest" The EKG now shows Q waves and ST segment elevations in the anterior leads. What intervention should the nurse perform?

Administer prescribed morphine sulfate IV and provide oxygen at 2 L/min per nasal cannula.

While assessing a radial artery catheter, the client complains of numbness and pain distal to the insertion site. What interventions should the nurse implement?

Promptly remove the arterial catheter from the radial artery.

A young adult who is hit with a baseball bat on the temporal area of the left skull is conscious when admitted to the ED and is transferred to the Neurological Unit to be monitored for signs of closed head injury. Which assessment finding is indicative of a developing epidural hematoma?

Altered consciousness within the first 24 hours after injury.

The healthcare provider prescribes heparin protocol at18 units/kg/hr for a client with a possible pulmonary embolism. This client weighs 144 pounds. The available solution is labeled, heparin sodium 25,000 units in 5% dextrose 250 ml. the nurse should program the pump to deliver how many ml/hr? (Enter numeric value only. If rounding is require round to the nearest whole number.)

Answer 12 Rationale: 144/2.2= 65kg 18units/kg/hr 65 kg x 18units/kg/hr= 1170 units/hr 25000 units heparin/250 ml of D5W = 100 units heparin per ml of solution Formula D/H x A = X

A client is receiving and oral antibiotic suspension labeled 250 mg/2ml. The healthcare provider prescribes 200mg every 6 hours. How many ml should the nurse administer at each dose? (Enter numerical value only. If rounding is required, round to the nearest tenth)

Answer: 1.6 Rational: using the formula D/H x Q 200mg/250 mg x 2ml = 200/250 = 1.6 ml

At the end of a preoperative teaching session on pain management techniques, a client starts to cry and states, "I just know I can't handle all the pain." What is the priority nursing diagnosis for this client?

Anxiety

After repositioning an immobile client, the nurse observes an area of hyperemia. To assess for blanching, what action should the nurse take?

Apply light pressure over the area.

While teaching a young male adult to use an inhaler for his newly diagnosed asthma, the client stares into the distance and appears to be concentrating on something other than the lesson the nurse is presenting. What action should the nurse take?

Ask the client what he is thinking about at his time.

An older male comes to the clinic with a family member. When the nurse attempts to take the client's health history, he does not respond to questions in a clear manner. What action should the nurse implement first?

Assess the surroundings for noise and distractions.

The nurse should teach the client to observe which precaution while taking dronedarone?

Avoid grapefruits and its juice

An elderly male client is admitted to the urology unit with acute renal failure due to a post-renal obstruction. Which questions best assists the nurse in obtaining relevant historical data? A. "Have you had a heart attack in the last 6 months" B. "Have you had any difficulty in starting your urinary stream" C. "Have you taken any antibiotics recently" D. "Have you received any blood products in the last year"

B. "Have you had any difficulty in starting your urinary stream"

While the school nurse is teaching a group of 14-year-olds, one of the participants remarks, "You are too young to be our teacher! You're not much older than we are!" How should the nurse respond? A. "How old do you think I am?" B. "We need to stay focused on the topic." C. "I think I am qualified to teach this group." D. "Do you think you can teach it any better?"

B. "We need to stay focused on the topic."

***A client is scheduled to receive an IW dose of ondansetron (Zofran) eight hours after receiving chemotherapy. The client has saline lock and is sleeping quietly without any restlessness. The nurse caring for the client is not certified in chemotherapy administration. What action should the nurse take? A. Ask a chemotherapy-certified nurse to administer the Zofran B. Administer the ondasentron (Zofran) after flushing the saline lock with saline C. Hold the scheduled dose of Zofran until the client awakens D. Awaken the client to assess the need for administration of the Zofran.

B. Administer the ondasentron (Zofran) after flushing the saline lock with saline

An antacid is prescribed for a client with gastroesophageal (GERD). The client asks the nurse, "How does this help my GERD?" What is the best response by the nurse? A. This medication will coat the lining of your esophagus B. Antacids will neutralize the acid in your stomach C. It will improve the emptying of food through your stomach D. antacids decrease the production of gastric secretions

B. Antacids will neutralize the acid in your stomach

During the transfer of a client who had major abdominal surgery this morning, the post anesthesia care unit (PACU) nurse reports that the client, who is awake and responsive continues to report pain and nausea after receiving morphine 2 mg IV and ondansetron 4 mg IV 45 mints ago. Which elements of SBAR communication are missing from the report given by the PACU nurse? (Select all that apply) A. Situation B. Background C. Assessment D. Recommendation E. Rationales.

B. Background C. Assessment D. Recommendation Rationale: BCD are correct. The current situation is reported regarding the client's nausea and pain (A). Based on SBAR communication, critical information about the client's clinical history (B), and assessment (C) such as pain scale or vital signs related to client's response to medication, are not included, nor are any recommendations for further follow-up (D). (E) Is not a component of SBAR communication

A client admitted to the emergency center had inspiratory and expiratory wheezing, nasal flaring, and thick, tenacious sputum secretions observed during the physical examination. Based on these assessment findings, what classification of pharmacologic agents should the nurse anticipate administering? A. Beta blockers B. Bronchodilators C. Corticosteroids D. Beta-adrenergics

B. Bronchodilators

The unit clerk reports to the charge nurse that a healthcare provider has written several prescriptions that are illegible and it appears the healthcare provider used several unapproved abbreviations in the prescriptions. What actions should the charge nurse take? A. Complete and file an incident (variance) report B. Call the healthcare provider who wrote the prescription C. Contact the healthcare provider review board for instructions D. Report the situation to the house supervisor

B. Call the healthcare provider who wrote the prescription

While the nurse is preparing a scheduled intravenous (IV) medication, the client states that the IV site hurts and refuses to allow the nurse to administer a flush to assess the site. Which intervention should the nurse implement? A. Apply ice first, then a warm compress to the IV site B. Discontinue the painful IV after a new IV is inserted C. Review the medical record for the date of insertion D. Document that the medication was not administered

B. Discontinue the painful IV after a new IV is inserted

In caring for a client with Cushing syndrome, which serum laboratory value is most important for the nurse to monitor? A. Lactate B. Glucose C. Hemoglobin D. Creatinine

B. Glucose

In assessing a client at 34-weeks' gestation, the nurse notes that she has a slightly elevated total T4 with a slightly enlarged thyroid, a hematocrit of 28%, a heart rate of 92 beats per minute, and a systolic murmur. Which finding requires follow-up? A. Elevated thyroid hormone level. B. Hematocrit of 28%. C. Heart rate of 92 beats per minute. D. Systolic murmur.

B. Hematocrit of 28%. Rational: although physiologic anemia is expected in pregnancy, a hematocrit of 28% is below pregnant norms and could signify iron-deficiency anemia. Other options are normal finding pregnancy

A 75-year-old female client is admitted to the orthopedic unit following an open reduction and internal fixation of a hip fracture. On the second postoperative day, the client becomes confused and repeatedly asks the nurse she is. What information for the nurse to obtain? A. Use of sleeping medications. B. History of alcohol use, C. Use of antianxiety medications, D. History of this behavior.

B. History of alcohol use

A client with Addison's disease becomes weak, confused, and dehydrated following the onset of an acute viral infection. The client's laboratory values include; sodium 129 mEq/l (129mmol/l SI), glucose 54 mg/dl (2.97mmol/l SI) and potassium 5.3 mmol/l SI). When reporting the findings to the HCP, the nurse anticipates a prescription for which intravenous medications? A. Regular insulin. B. Hydrocortisone C. Broad spectrum antibiotic D. Potassium chloride

B. Hydrocortisone

When implementing a disaster intervention plan, which intervention should the nurse implement first? A. Initiate the discharge of stable clients from hospital units B. Identify a command center where activities are coordinated C. Assess community safety needs impacted by the disaster D. Instruct all essential off-duty personnel to report to the facility

B. Identify a command center where activities are coordinated

An older adult female admitted to the intensive care unit (ICU) with a possible stroke is intubated with ventilator setting of tidal volume 600, PlO2 40%, and respiratory rate of 12 breaths/minute. The arterial blood gas (ABG) results after intubation are PH 7.31. PaCO2 60, PaO2 104, SPO2 98%, HCO3 23. To normalize the client's ABG finding, which action is required? A. Report the results to the healthcare provider. B. Increase ventilator rate. C. Administer a dose of sodium carbonate. D. Decrease the flow rate of oxygen.

B. Increase ventilator rate. Rationale: This client is experience respiratory acidosis. Increasing the ventilator rate depletes CO2 a, which returns the PH toward normal. Report findings is important but only after increasing ventilator rate.

Which intervention should the nurse include in the plan of care for a client with leukocytosis? A. Avoid intramuscular injections B. Monitor temperature regularly C. Assess skin for petechiae or bruising D. Implement protective isolation measures

B. Monitor temperature regularly

A client with superficial burns to the face, neck, and hands resulting from a house fire...which assessment finding indicates to the nurse that the client should be monitored for carbon monoxide...? A. Expiratory stridor and nasal flaring B. Mucous membranes cherry red color C. Carbonaceous particles in sputum D. Pulse oximetry reading of 80 percent

B. Mucous membranes cherry red color

In early septic shock states, what is the primary cause of hypotension? A. Peripheral vasoconstriction B. Peripheral vasodilation C. Cardiac failure D. A vagal response

B. Peripheral vasodilation Rationale: Toxins released by bacteria in septic shock create massive peripheral vasodilation and increase microvascular permeability at the site of the bacterial invasion.

A client is admitted to the hospital after experiencing a brain attack, commonly referred to as a stroke or cerebral vascular accident (CVA). The nurse should request a referral for speech therapy if the client exhibits which finding? A. Abnormal responses for cranial nerves I and II B. Persistent coughing while drinking C. Unilateral facial drooping D. Inappropriate or exaggerated mood swings

B. Persistent coughing while drinking

***The nurse is collecting sterile sample for culture and sensitivity from a disposable three chamber water-seal drainage system connected to a pleural chest tube. The nurse should obtain the sample from which site on the drainage system? A. Stopper port located above the water-seal level B. Plastic tubing located at the chest insertion site C. Rubberized port at the bottom of collection chamber D. Tubbing located on the top of the suction chamber

B. Plastic tubing located at the chest insertion site

A client with a history of dementia has become increasingly confused at night and is picking at an abdominal surgical dressing and the tape securing the intravenous (IV) line. The abdominal dressing is no longer occlusive, and the IV insertion site is pink. What intervention should the nurse implement? A. Replace the IV site with a smaller gauge. B. Redress the abdominal incision C. Leave the lights on in the room at night. D. Apply soft bilateral wrist restraints.

B. Redress the abdominal incision Rationale: The abdominal incision should be redressed using aseptic-techniques. The IV site should be assessed to ensure that it has not been dislodged and a dressing reapplied, if need it. Leaving the light on at night may interfere with the client's sleep and increase confusion. Restraints are not indicated and should only be used as a last resort to keep client from self-harm.

A male client returns to the mental health clinic for assistance with his anxiety reaction that is manifested by a rapid heartbeat, sweating, shaking, and nausea while driving over the bay bridge. What action I the treatment plan should the nurse implement? A. Tell the client to drive over the bridge until fear is manageable B. Teach client to listen to music or audio books while driving C. Encourage client to have spouse drive in stressful places. D. Recommend that the client avoid driving over the bridge.

B. Teach client to listen to music or audio books while driving Rationale: Desensitization is component in the treatment plan for clients with panic attacks which is best approached with anxiety-reducing strategies, such as listening to audio book (B) during situation that precipitate symptoms (A) is a flooding technique that requires professional guidance.

One year after being discharged from the burn trauma unit, a client with a history of 40% full-thickness burns is admitted with bone pain and muscle weakness. Which intervention should the nurse include in the clients plan of care? A. Encourage Progressive active range of motion B. Teach need for dietary and supplementary vitamin D3 C. Explain the need for skin exposure to sunlight without sunscreen D. Instruct the client to use of muscle strengthening exercises

B. Teach need for dietary and supplementary vitamin D3 Rationale: Burn injury results in the acute loss of bone as well as the development of progressive vitamin D deficiency because burn scar tissue and adjacent normal-appearing skin cannot convert normal quantities of the precursors for vitamin D3 that is synthesized from ultraviolet sun rays which is needed for strong bones. Clients with a history of full thickness burns should increase their dietary resources of vitamin D and supplemental D3 (B). range of motion (A) and muscle strengthening exercises (D) do not treat he underlying causes of the bone pain and weakness unprotected sunlight (C) should be avoided.

The nurse is explaining the need to reduce salt intake to a client with primary hypertension. What explanation should the nurse provide? A. High salt can damage the lining of the blood vessels B. Too much salt can cause the kidneys to retain fluid C. Excessive salt can cause blood vessels to constrict D. Salt can cause information inside the blood vessels

B. Too much salt can cause the kidneys to retain fluid Rationale: Excessive salt intake can contribute to primary hypertension by causing renal salt retention which influence water retention that expands blood volume and pressure (ACD) are not believed to contribute to primary hypertension.

A client receiving chemotherapy has severe neutropenia. Which snack is best for the nurse to recommend to the client?

Baked apples topped with dried raisins

After changing to a new brand of laundry detergent, an adult male reports that he has a fine itchy rash. Which assessment finding warrants immediate intervention by the nurse?

Bilateral Wheezing.

A client is receiving mesalamine 800 mg PO TID. Which assessment is most important for the nurse to perform to assess the effectiveness of the medication?

Bowel patterns Rationale: the client should be assessed for a change in bowel patterns to evaluate the effectiveness of this medication because Mesalamine is used to treat ulcerative colitis (a condition which causes swelling and sores in the lining of the colon [large intestine] and rectum) and also to maintain improvement of ulcerative colitis symptoms. Mesalamine is in a class of medications called anti-inflammatory agents. It works by stopping the body from producing a certain substance that may cause inflammation.

A client with hyperthyroidism is receiving propranolol (Inderal). Which finding indicates that the medication is having the desired effect? A. Decrease in serum T4 levels B. Increase in blood pressure C. Decrease in pulse rate D. Goiter no longer palpable

C. Decrease in pulse rate

When finding a client sitting on the floor, the nurse calls for help from the unlicensed assistive personnel (UAP). Which task should the nurse ask the UAP to do? A. Check for any abrasions or bruises. B. Help the client to stand. C. Get a blood pressure cuff. D. Report the fall to the nurse-manager.

C. Get a blood pressure cuff.

The healthcare provider prescribes carboprost tromethamine (Hemabate) 250 mcg IM for a multigravida postpartum client who is experiencing heavy, bright red vaginal bleeding. Prior to administering this medication, which interventions should the RN implement? A. Obtain a second IV access. B. Decrease the room temperature. C. Give the prescribed antiemetic. D. Insert an indwelling catheter.

C. Give the prescribed antiemetic.

What is the nurse's priority goal when providing care for a 2-year-old child experiencing a seizure? A. Stop the seizure activity B. Decrease the temperature C. Manage the airway D. Protect the body from injury

C. Manage the airway

***A child with heart failure (HF) is taking digitalis. Which signs indicates to the nurse that the child may be experiencing digitalis toxicity? A. Tachycarcia B. Dyspnea C. Vomiting D. Muscle cramps

C. Vomiting

When teaching a group of school-age children how to reduce the risk of Lyme disease which instruction should the camp nurse include? A. Wash hands frequently B. Avoid drinking lake water C. Wear long sleeves and pants D. Do not share personal products

C. Wear long sleeves and pants Rationale: Lyme disease is it tick bone disorder and is transmitted to a child via a tick bite. Keeping the skin covered reduces the risk

A preschooler with constipation needs to increase fiber intake. Which snack suggestion should the nurse provide? A. soft pretzels B. fruit-flavored yogurt C. oatmeal cookies D. low fat cheese sticks

C. oatmeal cookies

The nurse is completing a head to toe assessment for a client admitted for observation after falling out of a tree. Which finding warrants immediate intervention by the nurse?

Clear fluid leaking from the nose.

The nurse is caring for a client receiving continuous IV fluids through a single lumen central venous catheter (CVC). Based on the CVC care bundle, which action should be completed daily to reduce the risk for infection?

Confirm the necessity for continued use of the CVC.

Based on the information provided in this client's medical record during labor, which should the nurse implement? (Click on each chart tab for additional information. Please be sure to scroll to the bottom right corner of each tab to view all information contained in the client's medical record.)

Continue to monitor the progress of labor.

A client with a chronic health problem has difficulty ambulating short distance due to generalized weakness, but is able to bear weight on both legs. To assist with ambulation and provide the greatest stability, what assistive device is best for this client? Crutches with 2 point gait. Crutches with 3 point gait. Crutches with 4 point gait. A quad cane

Crutches with 4 point gait.

A male client with COPD smokes two packs of cigarettes per day and is admitted to the hospital for a respiratory infection. He complains that he has trouble controlling respiratory distress at home when using his rescue inhaler. Which comment from the client indicates to the nurse that he is not using his inhaler properly? A. "I have a hard time inhaling and holding my breath after I squeeze the inhaler, but I do my best" B. "I never use the inhaler unless I am feeling really short of breath" C. I always shake the inhaler several times before I start" D. "After I squeeze the inhaler and swallow, I always feel a slight wave of nausea, bit it goes away"

D. "After I squeeze the inhaler and swallow, I always feel a slight wave of nausea, bit it goes away"

An older male client with type 2 diabetes mellitus reports that has experiences legs pain when walking short distances, and that the pain is relieved by rest. Which client behavior indicates an understanding of healthcare teaching to promote more effective arterial circulation? A. Consistently applies TED hose before getting dressed in the morning. B. Frequently elevated legs thorough the day. C. Inspect the leg frequently for any irritation or skin breakdown D. Completely stop cigarette/ cigar smoking.

D. Completely stop cigarette/ cigar smoking. Rationale: Stopping cigarette smoking helps to decrease vasoconstriction and improve arterial circulation to the extremity.

A client with chronic alcoholism is admitted with a decreased serum magnesium level. Which snack option should the nurse recommend to this client? A. Cheddar cheese and crackers. B. Carrot and celery sticks. C. Beef bologna sausage slices. D. Dry roasted almonds.

D. Dry roasted almonds. Rational: alcoholism promotes inadequate food intake and gastrointestinal loss of magnesium include green leafy vegetables and nuts and seeds. Other snacks listed provide much lower amounts of magnesium per serving.

A client who is admitted to the intensive care unit with a right chest tube attached to a THORA-SEAL chest drainage unit becomes increasingly anxious and complain of difficulty breathing. The nurse determine the client is tachypneic with absent breath sounds in the client's right lungs fields. Which additional finding indicates that the client has developed a tension pneumothorax? A. Continuous bubbling in the water seal chamber B. Decrease bright red blood drainage C. Tachypnea and difficulty breathing D. Tracheal deviation toward the left lung.

D. Tracheal deviation toward the left lung. Rationale: Tracheal deviation toward the unaffected left lung with absent breath sounds over the affected right lung are classic late signs of a tension pneumothorax.

***A new mother tells the nurse that she is unsure if she will be able to transition into parenthood. What action should the nurse take?

Determine if she can ask for support from family, friend, or the baby's father.

The nurse is preparing to administer an oral antibiotic to a client with unilateral weakness, ptosis, mouth drooping and, aspiration pneumonia. What is the priority nursing assessment that should be done before administering this medication?

Determine which side of the body is weak.

A female client reports she has not had a bowel movement for 3 days, but now is defecating frequent small amount of liquid stool. Which action should the nurse implement?

Digitally check the client for a fecal impaction

A female client with acute respiratory distress syndrome (ARDS) is chemically paralyzed and sedated while she is on as assist-control ventilator using 50% FIO2. Which assessment finding warrants immediate intervention by the nurse?

Diminished left lower lobe sounds Rationale: Diminished lobe sounds indicate collapsed alveoli or tension pneumothorax, which required immediate chest tube insertion to re-inflate the lung.

***In assessing a client 48 hours following a fracture, the nurse observes ecchymosis at the fracture site, and recognizes that hematoma formation at the bone fragment site has occurred. What action should the nurse implement?

Document the extend of the bruising in the medical record

A client is being discharged with a prescription for warfarin (Coumadin). What instruction should the nurse provide this client regarding diet?

Eat approximated the same amount of leafy green vegetables daily so the amount of vitamin K consumed is consistent.

Which instruction should the nurse provide a pregnant client who is complaining of heartburn?

Eat small meal throughout the day to avoid a full stomach.

***The nurse note a visible prolapse of the umbilical cord after a client experiences spontaneous rupture of the membranes during labor. What intervention should the nurse implement immediately?

Elevate the presenting part off the cord.

The nurse note a visible prolapse of the umbilical cord after a client experiences spontaneous rupture of the membranes during labor. What intervention should the nurse implement immediately?

Elevate the presenting part off the cord.

After a sudden loss of consciousness, a female client is taken to the ED and initial assessment indicate that her blood glucose level is critically low. Once her glucose level is stabilized, the client reports that was recently diagnosed with anorexia nervosa and is being treated at an outpatient clinic. Which intervention is more important to include in this client's discharge plan?

Encourage a low-carbohydrate and high-protein diet

An elderly female is admitted because of a change in her level of sensorium. During the evening shift, the client attempts to get out bed and falls, breaking her left hip. Buck's skin traction is applied to the left leg while waiting for surgery. Which intervention is most important for the nurse to include in this client's plan care?

Ensure proper alignment of the leg in traction.

An adult female client tells the nurse that though she is afraid her abusive boyfriend might one day kill her, she keeps hoping that he will change. What action should the nurse take first?

Explore client's readiness to discuss the situation.

A client presents in the emergency room with right-sided facial asymmetry. The nurse asks the client to perform a series of movements that require use of the facial muscles. What symptoms suggest that the client has most likely experience a Bell's palsy rather than a stroke?

Inability to close the affected eye, raise brow, or smile

The nurse should teach the parents of a 6 year-old recently diagnosed with asthma that the symptom of acute episode of asthma are due to which physiological response?

Inflammation of the mucous membrane & bronchospasm

***A client with pneumonia has arterial blood gases levels at: PH 7.33; PaCO2 49 mm/hg; HCO3 25 mEq/L; PaO2 95. What intervention should the nurse implement based on these results?

Institute coughing and deep breathing protocols

The nurse caring for a client with acute renal fluid (ARF) has noted that the client has voided 800 ml of urine in 4 hours. Based on this assessment, what should the nurse anticipate that client will need?

Large amounts of fluid and electrolyte replacement.

***A client is receiving continuous bladder irrigation via a triple-lumen suprapubic catheter that was placed during prostatectomy. Which report by the unlicensed assistive personnel (UAP) requires intervention by the nurse?

Leakage around catheter insertion site

The nurse is developing a teaching program for the community. What population characteristic is most influential when choosing strategies for implementing a teaching plan?

Literacy level

The nurse is caring for a client who had gastric bypass surgery yesterday. Which intervention is most important for the nurse to implement during the first 24 postoperative hours?

Measure hourly urinary output. Rationale: a serious early complications of gastric bypass surgery is an anastomoses leak, often resulting in death.

Which intervention should the nurse include in the plan of care for a child with tetanus?

Minimize the amount of stimuli in the room

One day following an open reduction and internal fixation of a compound fracture of the leg, a male client complains of "a tingly sensation" in his left foot. The nurse determines the client's left pedal pulses are diminished. Based on these finding, what is the client's greatest risk?

Neurovascular and circulation compromise related to compartment syndrome.

One day after abdominal surgery, an obese client complains of pain and heaviness in the right calf. What action should the nurse implement?

Observe for unilateral swelling

A client with a peripherally inserted central catheter (PICC) line has a fever. What client assessment is most important for the nurse to perform?

Observe the antecubital fossa for inflammation.

A client is admitted with an epidural hematoma that resulted from a skateboarding accident. To differentiate the vascular source of the intracranial bleeding, which finding should the nurse monitor?

Rapid onset of decreased level of consciousness.

A female client with breast cancer who completed her first chemotherapy treatment today at an out-patient center is preparing for discharge. Which behavior indicates that the client understands her care needs?

Rented movies and borrowed books to use while passing time at home

During an annual physical examination, an older woman's fasting blood sugar (FBS) is determined to be 140 mg/dl or 7.8 mmol/L (SI). Which additional finding obtained during a follow-up visit 2 weeks later is most indicative that the client has diabetes mellitus (DM)?

Repeated fasting blood sugar (FBS) is 132 mg/dl or 7.4 mmol/L (SI).

Following discharge teaching, a male client with duodenal ulcer tells the nurse the he will drink plenty of dairy products, such as milk, to help coat and protect his ulcer. What is the best follow-up action by the nurse?

Review with the client the need to avoid foods that are rich in milk and cream

When preparing to discharge a male client who has been hospitalized for an adrenal crisis, the client expresses concern about having another crisis. He tells the nurse that he wants to stay in the hospital a few more days. Which intervention should the nurse implement?

Schedule an appointment for an out-patient psychosocial assessment.

A school nurse is called to the soccer field because a child has a nose bleed (epistaxis). In what position should the nurse place the child?

Sitting up and leaning forward

A nurse-manager is preparing the curricula for a class for charge nurses. A staffing formula based on what data ensures quality client care and is most cost-effective?

Skills of staff and client acuity

The nursing staff on a medical unit includes a registered nurse (RN), practical nurse (PN), and an unlicensed assistive personnel (UAP). Which task should the charge nurse assign to the RN?

Supervise a newly hired graduate nurse during an admission assessment.

A client with multiple sclerosis (MS) has decreased motor function after taking a hot bath (Uhthoff's sign). Which pathophysiological mechanism supports this response?

Temporary vasodilation

For the past 24 hours, an antidiarrheal agent, diphenoxylate, has been administered to a bedridden, older client with infectious gastroenteritis. Which finding requires the nurse to take further action?

Tented skin turgor.

***A client with hyperthyroidism is admitted to the postoperative after subtotal thyroidectomy. Which of the client's serum laboratory values requires intervention by the nurse?

Total calcium 5.0 mg/dl

A client with history of bilateral adrenalectomy is admitted with a week, irregular pulse, and hypotension. Which assessment finding warrants immediate intervention by the nurse?

Ventricular arrhythmias. Rationale: adrenal crisis, a potential complication of bilateral adrenalectomy, results in the loss of mineralocorticoids and sodium excretions that is characterized by hyponatremia, hyperkalemia, dehydration, and hypotension. Ventricular arrhythmias are life threatening and required immediate intervention to correct critical potassium levels.

***The nurse administers an antibiotic to a client with respiratory tract infection. To evaluate the medication's effectiveness, which laboratory values should the nurse monitor? Select all that apply

White blood cell (WBC) count Sputum culture and sensitivity

A client currently receiving an infusion labeled Heparin Sodium 25,000 Units in 5% Dextrose Injection 500 mL at 14 mL/hour. A prescription is received to change the rate of the infusion to 900 units of Heparin per hour. The nurse should set the infusion pump to deliver how many mL/hour? (Enter numeric value only).

700 Rationale: D/H x Q = 25000 / 500 x 14 = 700

When assessing a male client, the nurse notes that he has unequal lung expansion. What conclusion regarding this finding is most likely to be accurate? The client has A. A collapsed lung B. A history of COPD C. A chronic lung infection D. Normally functioning lungs

A. A collapsed lung

After a routine physical examination, the healthcare admits a woman with a history of Systemic Lupus Erythematous (SLE) to the hospital because she has 3+ pitting ankle edema and blood in her urine. Which assessment finding warrants immediate intervention by the nurse? A. Blood pressure 170/98 B. Joint and muscle aches C. Urine output 300 ml/hr D. Dark, rust-colored urine

A. Blood pressure 170/98

After a third hospitalization 6 months ago, a client is admitted to the hospital with ascites and malnutrition. The client is drowsy but responding to verbal stimuli and reports recently spitting up blood. What assessment finding warrants immediate intervention by the nurse? A. Capillary refill of 8 seconds B. Bruises on arms and legs C. Round and tight abdomen D. Pitting edema in lower legs

A. Capillary refill of 8 seconds

A child with heart failure is receiving the diuretic furosemide (Lasix) and has serum potassium level 3.0 mEq/L. Which assessment is most important for the nurse to obtain? A. Cardiac rhythm and heart rate. B. Daily intake of foods rich in potassium. C. Hourly urinary output D. Thirst ad skin turgor.

A. Cardiac rhythm and heart rate.

A newly admitted client vomits into an emesis basin as seen in the picture. The nurse should consult with the healthcare provider before administering which of the client's prescribes medications? A. Clopidogrel (Plavix), an antiplatelet agent, given orally B. Nitroglycerin (nitro-dur), an antianginal, to be given transdermally C. Methylprednisolone (solu-medrol), a corticosteroid, to be given IV D. Furosemide (lasix), a loop diuretic, to be given intravenously E. Enoxaparin (lovenox), a low-molecular weight heparin to be given subcutaneous

A. Clopidogrel (Plavix), an antiplatelet agent, given orally C. Methylprednisolone (solu-medrol), a corticosteroid, to be given IV E. Enoxaparin (lovenox), a low-molecular weight heparin to be given subcutaneous

A nurse is conducting a physical assessment of a young adult. Which information provides the best indication of the individual nutritional status? A. Condition of hair, nails, and skin B. A 24-hour diet history C. History of a recent weight loss D. Status of current petite

A. Condition of hair, nails, and skin Rationale: the assessment of hair, nails, and skin is most effective of long-term nutritional status, which is important in the healing process.

A client is being treated for syndrome of inappropriate antidiuretic hormone (SIADH). On examination, the client has a weight gain of 4.4 lbs (2 kg) in 24 hours and an elev ated blood pressure. Which intervention should the nurse implement first? A. Ensure client takes a diuretic q AM B. Obtain serum creatinine levels daily C. Measure ankle circumference D. Monitor daily sodium intake

A. Ensure client takes a diuretic q AM

What is the priority nursing action when initiating morphine therapy via an intravenous patient-controlled analgesia (PCA) pump? A. Initiate the dosage lockout mechanism on the PCA pump B. Instruct the client to use the medication before the pain becomes severe C. Assess the abdomen for bowel sounds. D. Assess the client ability to use a numeric pain scale

A. Initiate the dosage lockout mechanism on the PCA pump

When conducting diet teaching for a client who is on a postoperative soft diet, which foods should eat? (Select all that apply) A. Pasta, noodles, rice. B. Egg, tofu, ground meat. C. Mashed, potatoes, pudding, milk. D. Brussel sprouts, blackberries, seeds. E. Corn bran, whole wheat bread, whole grains.

A. Pasta, noodles, rice. B. Egg, tofu, ground meat. C. Mashed, potatoes, pudding, milk. Rational: a client's postoperative diet is commonly progressed as tolerated. A soft diet includes foods that are mechanically soft in texture (pasta, egg, ground meat, potatoes, and pudding. High fiber foods that require thorough chewing and gas forming foods, such as cruciferous vegetables and fresh fruits with skin, grains and seeds are omitted.

A client with multiple sclerosis is receiving beta-1b interferon every other day. To assess for possible bone marrow suppression caused by the medication, which serum laboratory test findings should the nurse monitor? (Select all that apply) A. Platelet count B. Red blood cell count (RBC) C. White blood cell count (WBC). D. Albumin and protein E. Sodium and potassium

A. Platelet count B. Red blood cell count (RBC) C. White blood cell count (WBC).

At 1615, prior to ambulating a postoperative client for the first time, the nurse reviews the client's medical record. Based on date contained in the record, what action should the nurse take before assisting the client with ambulation: A. Remove sequential compression devices. B. Apply PRN oxygen per nasal cannula. C. Administer a PRN dose of an antipyretic. D. Reinforce the surgical wound dressing.

A. Remove sequential compression devices. Rationale: Sequential compression devices should be removed prior to ambulation and there is no indication that this action is contraindicated. The client's oxygen saturation levels have been within normal limits for the previous four hours, so supplemental oxygen is not warranted.

The leg of a client who is receiving hospice care have become mottled in appearance. When the nurse observes the unlicensed assistive personal (UAP) place a heating pad on the mottled areas, what action should the nurse take? A. Remove the heating pads and place a soft blanket over the client's leg and feet. B. Advise the UAP to observe the client's skin while the heating pads are in place. C. Elevate the client's feet on a pillow and monitor the client's pedal pulses frequently. D. Instruct the UAP to reposition the heating pads to the sides of the legs and feet.

A. Remove the heating pads and place a soft blanket over the client's leg and feet.

The nurse is preparing to discharge an older adult female client who is at risk for hy...nurse include with this client's discharge teaching? A. Report any muscle twitching or seizures B. Take vitamin D with calcium daily C. Low fat yogurt is a good source of calcium D. Keep a diet record to monitor calcium intake E. Avoid seafood, particularly selfish

A. Report any muscle twitching or seizures B. Take vitamin D with calcium daily C. Low fat yogurt is a good source of calcium D. Keep a diet record to monitor calcium intake

When caring for a client who has acute respiratory distress syndrome (ARDS), the nurse elevates the head of the bed 30 degrees. What is the reason for this intervention? A. To reduce abdominal pressure on the diaphragm B. To promote retraction of the intercostal accessory muscle of respiration C. To promote bronchodilation and effective airway clearance D. To decrease pressure on the medullary center which stimulates breathing

A. To reduce abdominal pressure on the diaphragm Rationale: a semi-sitting position is the best position for matching ventilation and perfusion and for decreasing abdominal pressure on the diaphragm, so that the client can maximize breathing.

A client with eczema is experiencing severe pruritus. Which PRN prescriptions should the nurse administer? (Select all that apply) A. Topical corticosteroid. B. Topical scabicide. C. Topical alcohol rub. D. Transdermal analgesic. E. Oral antihistamine

A. Topical corticosteroid. E. Oral antihistamine Rationale: anti-inflammatory actions of topical corticosteroids and oral antihistamines provide relief from severe pruritus (itching). Other options are not indicated.

Which interventions should the nurse include in a long-term plan of care for a client with COPD? A. Reduce risk factors for infection B. Limit fluid intake to reduce secretions C. Use diaphragmatic breathing to achieve better exhalation D. Administer high flow oxygen during sleep

A. Reduce risk factors for infection

Which intervention should the nurse include in a long-term plan of care for a client with Chronic Obstructive Pulmonary Disease (COPD)? A. Reduce risks factors for infection B. Administer high flow oxygen during sleep C. Limit fluid intake to reduce secretions D. Use diaphragmatic breathing to achieve better exhalation

A. Reduce risks factors for infection

The nurse observes an adolescent client prepare to administer a prescribed corticosteroid medication using a metered dose inhaler as seen in the picture. What action should the nurse take? A. Remind the client to hold his breath after inhaling the medication B. Confirm that the client has correctly shaken the inhaler C. Affirm that the client has correctly positioned the inhaler D. Ask the client if he has a spacer to use for this medication

A. Remind the client to hold his breath after inhaling the medication

A client with atrial fibrillation receives a new prescription for dabigatran. What instruction should the nurse include in this client's teaching plan?

Avoid use of nonsteroidal ant-inflammatory drugs (NSAID).

A male client with diabetes mellitus type 2, who is taking pioglitazone PO daily, reports to the nurse the recent onset of nausea, accompanied by dark-colored urine, and a yellowish cast to his skin. What instructions should the nurse provide? A. "You have become dehydrated from the nausea. You will need to rest and increase fluid intake" B. "you need to seek immediate medical assistance to evaluate the cause of these symptoms" C. A urine specimen will be needed to determine what kind of infection you have developed" D. use insulin per sliding scale until the nausea resolves, and then resume your oral medication"

B. "you need to seek immediate medical assistance to evaluate the cause of these symptoms"

During a routine clinic visit, an older female adult tells the nurse that she is concerned that the flu season is coming soon, but is reluctant to obtain the vaccination. What action should the nurse take first? A. Determine when the client last had an influenza vaccination. B. Discuss the concerns expressed by the client about the vaccination. C. Ask about any recent exposure to persons with the flu or other viruses. D. Review the informed consent form for the vaccination with the client.

B. Discuss the concerns expressed by the client about the vaccination. Rationale: the nurse should first address the concerns identified by the client, before taking other actions, such as obtaining information about past vaccinations, exposure to the flu, or reviewing the informed consent form.

Which assessment is more important for the nurse to include in the daily plan of care for a client with a burned extremity? A. Range of Motion B. Distal pulse intensity C. Extremity sensation D. Presence of exudate

B. Distal pulse intensity Rationale: Distal pulse intensity assesses the blood flow through the extremity and is the most important assessment because it provides information about adequate circulation to the extremity. Range of motions evaluates the possibility of long term contractures sensation. C evaluates neurological involvement, and exudate. D provides information about wound infection, but this assessment do not have the priority of determining perfusion to the extremity.

While a child is hospitalized with acute glomerulonephritis, the parents ask why blood pressure readings are taken so often. Which response by the nurse is most accurate? A. Blood pressure fluctuations means that the condition has become chronic B. Elevated blood pressure must be anticipated and identified quickly C. Hypotension leading to sudden shock can develop at any time D. Sodium intake with meals and snacks affects the blood pressure

B. Elevated blood pressure must be anticipated and identified quickly

The charge nurse is making assignment on a psychiatric unit for a practical nurse (PN) and newly license register nurse (RN). Which client should be assigned to the RN? A. An adult female who has been depress for the past several month and denies suicidal ideations. B. A middle-age male who is in depressive phase on bipolar disease and is receiving Lithium. C. A young male with schizophrenia who said voices is telling him to kill his psychiatric. D. An elderly male who tell the staff and other client that he is superman and he can fly.

C. A young male with schizophrenia who said voices is telling him to kill his psychiatric. Rationale: The RN should deal with the client with command hallucinations and these can be very dangerous if the client's acts on the commands, especially if the command is a homicidal in nature. Other client present low safety risk.

On a busy day, one hour after the shift report is completed, the charge nurse learns that a female staff nurse who lives one hour away from the hospital forgot her prescription eye glasses at home. What action should the charge nurse take? A. Encourage the nurse to purchase reading glasses in the hospital gift shop B. Request another nurse to assist the staff nurse with her documentation C. Ask the nurse to return home and get her prescription eyeglasses for work. D. Tell the staff nurse to take a day off and change her weekly work schedule

C. Ask the nurse to return home and get her prescription eyeglasses for work.

An older woman who was recently diagnosed with end stage metastatic breast cancer is admitted because she is experiencing shortness of breath and confusion. The client refuses to eat and continuously asks to go home. Arterial blood gases indicate hypoxia. Which intervention is most important for the nurse to implement? A. Prepare for emergent oral intubation B. Offer sips of favorite beverages C. Clarify end of life desires D. Initiate comfort measures

C. Clarify end of life desires

A 7-year-old boy is brought to the clinic because of facial edema. He reports that he has been voiding small amounts of dark, cloudy, tea-colored urine. The parents state that their son had a sore throat 2 weeks earlier, but it has resolved. After assessing the child's vital signs and weight, what intervention should the nurse implement next? A. Measure the child's abdominal girth B. Perform an ostoscopic examination C. Collect a urine specimen for routine urinalysis D. Obtain a blood specimen for serum electrolytes

C. Collect a urine specimen for routine urinalysis

An adult female client is admitted to the psychiatric unit because of a complex handwashing ritual she performs daily that takes two hours or longer to complete. She worries about staying clean and refuses to sit on any of the chairs in the day area. This client's handwashing is an example of which clinical behavior? A. Addiction B. Phobia C. Compulsion D. Obsession

C. Compulsion

***A client exposed to tuberculosis is scheduled to begin prophylactic treatment with isoniazid. Which information is most important for the nurse to note before administering the initial dose? A. Conversion of the client's PPD test from negative to positive. B. Length of time of the exposure to tuberculosis. C. Current diagnosis of hepatitis D. History of intravenous drug abuse.

C. Current diagnosis of hepatitis Rationale: prophylactic treatment of tuberculosis with isoniazid is contraindicated for persons with liver disease because it may cause liver damage. The nurse should withhold the prescribed dose and contact the healthcare provider. Other options do not provide data indicating the need to question or withhold the prescribed treatment.

The nurse is preparing to administer 1.6 ml of medication IM to a 4 month old infant. Which action should the nurse include? A. Select a 22 gauge 1 ½ inch (3.8 cm) needle for the intramuscular injection B. Administer into the deltoid muscle while the parent holds the infant securely C. Divide the medication into two injection with volumes under 1ml D. Use a quick dart-like motion to inject into the dorsogluteal site.

C. Divide the medication into two injection with volumes under 1ml Rationale: IM injection for children under 3 of age should not exceed 1ml. divide the dose into smaller volumes for injection in two different sites.

During the initial newborn assessment, the nurse finds that a newborn's heart rate is irregular. Which intervention should the nurse implement? A. Notify the pediatrician immediately. B. Teach the parents about congenital heart defects. C. Document the finding in the infant's record. D. Apply oxygen per nasal cannula at 3 L/min.

C. Document the finding in the infant's record.

The nurse is reviewing a client's electrocardiogram and determines the PR interval (PRI) is prolonged. What does this finding indicate? A. Initiation of the impulses from a location outside the SA node B. Inability of the SA node to initiate an impulse at the normal rate C. Increased conduction time from the SA node to the AV junction D. Interference with the conduction through one or both ventricles.

B. Inability of the SA node to initiate an impulse at the normal rate Rationale: A prolonged PRI reflects an increased amount of time for an impulse to travel from the SA node through the AV node and is characteristic of a first degree heart block.

A client who had an emergency appendectomy is being mechanically ventilated, and soft wrist restrain are in place to prevent self extubation. Which outcome is most important for the nurse to include in the client's plan of care? A. Understand pain management scale B. Maintain effective breathing patterns Absence of ventilator associated pneumonia No injuries refer to soft restrains occur

B. Maintain effective breathing patterns Rationale: Basic airway management (B) is the priority. Pain management (A), risk of infection (C), and prevention of injury (D) do not have the same priority as (C)

A client with type 2 diabetes mellitus is admitted for antibiotic treatment for a leg ulcer. To monitor the client for the onset of hyperosmolar hyperglycemic nonketotic syndrome (HHNS), what actions should the nurse take? (Select all that apply) A. Check urine for ketones B. Measure blood glucose C. Monitor vital signs D. Assessed level of consciousness E. Obtain culture of wound

B. Measure blood glucose C. Monitor vital signs D. Assessed level of consciousness Rationale: Blood glucose greater than 600 mg/dl (33.3 mmol/L SI), vital sign changes in mental awareness are indicators of possible HHNS. Urine ketones are monitored in diabetic ketoacidosis. Wound culture is performed prior to treating the wound infection but is not useful in monitoring for HHNS

A client who developed syndrome of inappropriate antidiuretic hormone (SIADH) associated with small carcinoma of the lung is preparing for discharge. When teaching the client about self-management with demeclocycline (Declomycin), the nurse should instruct the client to report which condition to the health care provider? A. Insomnia B. Muscle cramping C. Increase appetite D. Anxiety.

B. Muscle cramping Rationale: SIADH causes dilution hyponatremia because of the increased release of ADH, which is treated with water restriction and demeclocycline, a tetracycline derivate that blocks the action of ADH. Signs of hyponatremia (normal 136-145), which indicate the need for increasing the dosage of demeclocycline, should be reported to the healthcare provider. The signs include: plasma sodium level less than 120, anorexia, nausea, weight changes related to fluid disturbance, headache, weakness, fatigue, and muscle cramping. AC& D are not related to hyponatremia.

The nurse is caring for several clients on a telemetry unit. Which client should the nurse assess first? The client who is demonstrating A. A paced rhythm with 100% capture after pacemaker replacement B. Normal sinus rhythm and complaining of chest pain C. Atrial fibrillation with congestive heart failure and complaining of fatigue D. Sinus tachycardia 3 days after a myocardial infarction

B. Normal sinus rhythm and complaining of chest pain

A client with hypertension receives a prescription for enalapril, an angiotensin...instruction should the nurse include in the medication teaching plan? A. Increase intake of potassium-rich foods B. Report increased bruising of bleeding C. Stop medication if a cough develops D. Limit intake of leafy green vegetables

B. Report increased bruising of bleeding

The nurse administers an oral antiviral to a client with shingles. Which finding is most important for the nurse report to the health care provider? A. Decreased white blood cell count B. Pruritus and muscle aches C. Elevated liver function tests D. Vomiting and diarrhea

C. Elevated liver function tests Rationale: Elevated liver function enzymes are a serious side effect of antivirals and should be reported. A decrease white blood count is a consistent finding with shingle B and (C and D) are side effects that affect that are of less priority than A.

A male client's laboratory results include a platelet count of 105,000/ mm3 Based on this finding the nurse should include which action in the client's plan of care? A. Cluster care to conserve energy B. Initiate contact isolation C. Encourage him to use an electric razor D. Asses him for adventitious lung sounds

C. Encourage him to use an electric razor Rationale: This client is at risk for bleeding based on his platelet count (normal 150,000 to 400,000/ mm3). Safe practices, such as using an electric razor for shaving, should be encouraged to reduce the risk of bleeding.

Which type of Leukocyte is involved with allergic responses and the destruction of parasitic worms? A. Neutrophils B. Lymphocytes C. Eosinophils D. Monocytes

C. Eosinophils Rationale: Eosinophils are involved in allergic responses and destruction of parasitic worms.

The nurse ends the assessment of a client by performing a mental status exam. Which statement correctly describes the purpose of the mental status exam? A. Determine the client's level of emotional functioning' B. Assess functional ability of the primary support system. C. Evaluate the client's mood, cognition and orientation. D. Review the client's pattern of adaptive coping skill

C. Evaluate the client's mood, cognition and orientation. Rational: the mental status exam assesses the client for abnormalities in cognitive functioning; potential thought processes, mood and reasoning, the other options listed are all components of the client's psychosocial assessment.

A client with C-6 spinal cord injury rehabilitation. In the middle of the night the client reports a severe, pounding headache, and has observable piloerection or "goosebumps". The nurse should asses for which trigger? A. Loud hallway noise. B. Fever C. Full bladder D. Frequent cough.

C. Full bladder Rational: a pounding headache is a sign of autonomic hyperreflexia, an acute emergency that occurs because of an exaggerated sympathetic response in a client with a high level spinal cord injury. Any stimulus below the level of injury can trigger autonomic hyperreflexia, but the most common cause is an overly distended bladder. The other options are unlikely to produce the manifestation of autonomic hyperreflexia.

A client's telemetry monitor indicates ventricular fibrillation (VF). After delivering one counter shock, the nurse resumes chest compression, after another minute of compression , the client's rhythm converts to supraventricular tachycardia (SVT) on the monitor, at this point , what is the priority intervention for the nurse? A. Prepare for transcutaneous pacing B. Administer IV epinephrine per ACLS protocol. C. Give IV dose of adenosine rapidly over 1-2 seconds. D. Deliver another defibrillator shock.

C. Give IV dose of adenosine rapidly over 1-2 seconds.

An adolescent, whose mother recently died, comes to the school nurse complain headache. Which statement made by the students should warrant further explanation nurse? A. "I've had dreams about Mon since she died." B. "I've been very sad and cry a lot at night." C. "I miss Mon and would like to go see her'". D. " it's hard to concentrate on my homework"

C. "I miss Mon and would like to go see her'".

An adult client experiences a gasoline tank fire when riding a motorcycle and is admitted to the emergency department (ED) with full thickness burns to all surfaces of both lower extremities. What percentage of body surface area should the nurse document in the electronic medical record (EMR)? A. 9 % B. 18 % C. 36 % D. 45 %

C. 36 % Rational: according to the rule of nines, the anterior and posterior surfaces of one lower extremity is designated as 18 %of total body surface area (TBSA), so both extremities equals 36% TBSA, other options are incorrect.

A client is admitted with an exacerbation of heart failure secondary to COPD. Which observations by the nurse require immediate intervention to reduce the likelihood of harm to this client? (Select all that apply). A. A bedside commode is positioned near the bed B. A saline lock is present in the right forearm C. A full pitcher of water is on the bedside table D. The client is lying in a supine position in bed E. A low sodium diet tray was brought to the room

C. A full pitcher of water is on the bedside table D. The client is lying in a supine position in bed

After multiple attempts to stop drinking, an adult male is admitted to the medical intensive care unit (MICU) with delirium tremens. He is tachycardic, diaphoretic, restless, and disoriented. Which finding indicates a life- threatening condition? A.CIWA-Ar for alcohol withdrawal score of 30 B. Acute onset of unrelenting chest pain C. Widening QRS complexes and flat waves D. Intense tremor and involuntary muscle activity

C. Widening QRS complexes and flat waves

The charge nurse is planning for the shift and has a registered nurse (RN) and a practical nurse (PN) on the team. Which client should the charge nurse assign to the RN? A. A 64 year old client who had a total hip replacement the previous day. B. A 75 year old client with renal calculi who requires urine straining. C. An adolescent with multiple contusions due to a fall that occurred 2 days ago. D. A 30 year old depressed client who admits to suicide ideation.

D. A 30 year old depressed client who admits to suicide ideation. RATIONALE: A client who is suicidal requires psychological assessment, therapeutic communication and knowledge beyond the educational level of a practical nurse (RN). Other clients could be cared for by the PN or the UAP, with supervision by the registered nurse.

To reduce the risk of being named in malpractice lawsuit, which action is most important for the nurse to take? A. Establish a trusting nurse-client relationship. B. Complete an incident report following a client injury. C. Maintain current professional malpractice insurance, D. Adhere consistently to standards of care.

D. Adhere consistently to standards of care.

A female client is admitted for diabetic crisis resulting from inadequate dietary practices. After stabilization, the nurse talks to the client about her prescribed diet. What client characteristic is most import for successful adherence to the diabetic diet? A. Knows that insulin must be given 30 min before eating B. Frequently eats fruits and vegetables at meals and between meals/ C. Has someone available who can prepare and oversee the diet D. Demonstrates willingness to adhere to the diet consistently

D. Demonstrates willingness to adhere to the diet consistently

A client who has a suspected brain tumor is schedules for a computed (CT) scan. When preparing the client for the client for the CT scan, which intervention should the nurse implement? A. Determine if the client has had a knee or hip replacement B. Immobilize the client's neck before moving onto stretcher C. Give an antiemetic to control nausea D. Obtain the client's food allergy history

D. Obtain the client's food allergy history

A client with a liver abscess develops septic shock. A sepsis resuscitation bundle protocol is initiated and the client receives a bolus of IV fluids. Which parameter should the nurse monitor to assess effectiveness of the fluid bolus? A. Mean arterial pressure (MAP) B. White blood cell count C. Blood culture D. Oxygen saturation

D. Oxygen saturation

A 26-year-old female client is admitted to the hospital for treatment of a simple goiter, and levothyroxine sodium (Synthroid) is prescribed. Which symptoms indicate to the nurse that the prescribed dosage is too high for this client? The client experiences A. Bradycardia and constipation B. Lethargy and lack of appetite C. Muscle cramping and dry, flushed skin D. Palpitations and shortness of breath

D. Palpitations and shortness of breath

In early septic shock states, what is the primary cause of hypotension? A. Cardiac failure B. A vagal response C. Peripheral vasoconstriction D. Peripheral vasodilation

D. Peripheral vasodilation

An 11-year-old client is admitted to the mental health unit after trying to run away from home and threatening self-harm. The nurse establishes a goal to promote effective coping, and plans to ask the client to verbalize three ways to deal with stress. Which activity is best to establish rapport and accomplish this therapeutic goal? A. Bring the client to the team meeting to discuss the treatment plan B. Ask the client to write feeling in a journal and then review it together C. Explain the purpose of each medication the client is currently taking D. Play a board game with the client and begin taking about stressors

D. Play a board game with the client and begin taking about stressors

At 40 week gestation, a laboring client who is lying is a supine position tells the nurse that she has finally found a comfortable position. What action should the nurse take? A. Encourage the client to turn on her left side. B. Place a pillow under the client's head and knees. C. Explain to the client that her position is not safe. D. Place a wedge under the client's right hip.

D. Place a wedge under the client's right hip. Rationale: Hypotension from pressure on the vena cava is a risk for the full-term client. Placing a wedge under the right hip will relieve pressure on the vena cava. Other options will either not relieve pressure on the vena cava or would not allow the client the remaining her position of choice.

When entering a client's room, the nurse discovers that the client is unresponsive and pulseless. The nurse initiate CPR and Calls for assistance. Which action should the nurse take next? A. Prepare to administer atropine 0.4 mg IVP B. Gather emergency tracheostomy equipment C. Prepare to administer lidocaine at 100 mg IVP D. Place cardiac monitor leads on the client's chest.

D. Place cardiac monitor leads on the client's chest. Rationale: Before further interventions can be done, the client's heart rhythm must be determined. This can be done by connecting the client to the monitor. A or C are not a first line drug given for any of the life threatening, pulses dysrhythmias

When development a teaching plan for a client newly diagnosed type 1 diabetes, the nurse should explain that an increase thirst is an early sing of diabetes ketoacidosis (DKA), which action should the nurse instruct the client to implement if this sign of DKA occur? A. Resume normal physical activity B. Drink electrolyte fluid replacement C. Give a dose of regular insulin per sliding scale D. Measure urinary output over 24 hours.

C. Give a dose of regular insulin per sliding scale Rationale: As hyperglycemia persist, ketone body become a fuel source, and the client manifest early signs of DKA that include excessive thirst, frequent urination, headache, nausea and vomiting. Which result in dehydration and loss of electrolyte. The client should determine fingersticks glucose level and self-administer a dose of regular insulin per sliding scale.

While changing a client's chest tube dressing, the nurse notes a crackling sensation when gentle pressure is applied to the skin at the insertion site. What is the best action for the nurse to take? A. Apply a pressure dressing around the chest tube insertion site. B. Assess the client for allergies to topical cleaning agents. C. Measure the area of swelling and crackling. D. Administer an oral antihistamine per PRN protocol.

C. Measure the area of swelling and crackling. Rational: a crackling sensation, or crepitus, indicates subcutaneous emphysema, or air leaking into the skin. This area should be measured and the finding documented. Other options are not indicated for crepitus.

Which problem reported by a client taking lovastatin requires the most immediate fallow up by the nurse? A. Diarrhea and flatulence B. Abdominal cramps C. Muscle pain D. Altered taste

C. Muscle pain Rationale: statins can cause rhabdomyolysis, a potentially fatal disease of skeletal muscle characterized by myoglobinuria and manifested with muscle pain, so this symptom should immediately be reported to the HCP.

In evaluating the effectiveness of a postoperative client's intermittent pneumatic compression devices, which assessment is most important for the nurse to complete? A. Evaluate the client's ability to use an incentive spirometer B. Monitor the amount of drainage from the client's incision C. Observe both lower extremities for redness and swelling D. Palpate all peripheral pulse points for volume and strength

C. Observe both lower extremities for redness and swelling Rationale: Intermittent compression devices (ICDs) are used to reduce venous stasis and prevent venous thrombosis in mobile and postoperative clients and its effectiveness is best assessed by observing the client's lower extremities for early signs of thrombophlebitis.

A client with possible acute kidney injury (AKI) is admitted to the hospital and mannitol is prescribed as a fluid challenge. Prior to carrying out this prescription, what intervention should the nurse implement? A. Collect a clean catch urine specimen. B. Instruct the client to empty the bladder. C. Obtain vital signs and breath sounds. D. No specific nursing action is required

C. Obtain vital signs and breath sounds. Rational: the client's baseline cardiovascular status should be determined before conducting the fluid challenge. If the client manifests changes in the vital signs and breath sounds associated with pulmonary edema, the administration of the fluid challenge should be terminate. Other options would not assure a safe administration of the medication.

The nurse instructs an unlicensed assistive personnel (UAP) to turn an immobilized elderly client with an indwelling urinary catheter every two hours. What additional action should the nurse instruct the UAP to take each time the client is turned? A. Empty the urinary drainage bag B. Feed the client a snack C. Offer the client oral fluids D. Assess the breath sounds

C. Offer the client oral fluids Rationale: Increasing oral fluid intake reduces the risk of problems associated with immobility, so the UAP should be instructed to offer the client oral fluids every two hours, or whenever turning he client. It is not necessary to empty the urinary bag or feed the client every two hours. Assessment is a nursing function, and UAPs do not have the expertise to perform assessment of breath sounds.

In determine the client position for insertion of an indwelling urinary catheter, it is most important for the nurse to recognize which client condition? A. High urinary PH B. Abdominal Ascites C. Orthopnea D. Fever.

C. Orthopnea Rationale: If the client is orthopneic, the nurse needs to adapt the insertion position that does not place the client in a supine position (the head of the bed should be elevated as much as possible).

A client who received multiple antihypertensive medications experiences syncope due to a drop in blood pressure to 70/40. What is the rationale for the nurse's decision to hold the client's scheduled antihypertensive medication? A. Increased urinary clearance of the multiple medications has produced diuresis and lowered the blood pressure B. The antagonistic interaction among the various blood pressure medications has reduced their effectiveness C. The additive effect of multiple medications has caused the blood pressure to drop too low D. the synergistic effect of the multiple medications has resulted in drug toxicity and resulting hypotension

C. The additive effect of multiple medications has caused the blood pressure to drop too low

The RN is assigned to care for four surgical clients. After receiving report, which client should the nurse see first? The client who is A. Two days postoperative bladder surgery with continuous bladder irrigation infusing. B. One day postoperative laparoscopic cholecystectomy requesting pain medication. C. Three days postoperative colon resection receiving transfusion of packed RBCs. D. Preoperative, in buck's traction, and scheduled for hip arthroplasty within the next 12 hours.

C. Three days postoperative colon resection receiving transfusion of packed RBCs.

***Immediately after extubation, a client who has been mechanically ventilated is placed on a 50% non-rebreather. The client is hoarse and complaining of a sore throat. Which assessment finding should the nurse report to the healthcare provider immediately? A. Blood tinged sputum B. Expiratory wheezing C. Upper airway stridor D. Oxygen saturations 90%

C. Upper airway stridor

Following a motor vehicle collision, an adult female with a ruptured spleen and a blood pressure of 70/44, had an emergency splenectomy. Twelve hours after the surgery, her urine output is 25 ml/hour for the last two hours. What pathophysiological reason supports the nurse's decision to report this finding to the healthcare provider?

Oliguria signals tubular necrosis related to hypoperfusion

Suicide precautions are initiated for a child admitted to the mental health unit following an intentional narcotic overdose. After a visitor leaves, the nurse finds a package of cigarettes in the client's room. Which intervention is most important for the nurse to implement?

Remove cigarettes for the client's room

When preparing a client for discharge from the hospital following a cystectomy and a urinary diversion to treat bladder cancer, which instruction is most important for the nurse to include in the client's discharge teaching plan?

Report any signs of cloudy urine output.

***During shift report, the central electrocardiogram (EKG) monitoring system alarms. Which client alarm should the nurse investigate firs?

Respiratory apnea of 30 seconds

What is the nurse's priority goal when providing care for a 2-year-old child experiencing seizure... A. Stop the seizure activity B. Decrease the temperature C. Manage the airway D. Protect the body from injury

C. Manage the airway

A family member of a frail elderly adult asks the nurse about eligibility requirements for hospice care. What information should the nurse provide? (Select all that apply.)

A. A client must be willing to accept palliative care, not curative care. B. The healthcare provider must project that the client has 6 months or less to live.

An older adult resident of a long-term care facility has a 5-year history of hypertension. The client has a headache and rate the pain 5 on a pain scale 0 to 10. The client's blood pressure is currently 142/89. Which interventions should the nurse implement? (Select all that apply) A. Administer a daily dose of lisinopril as scheduled. B. Assess the client for postural hypotension. C. Notify the healthcare provider immediately D. Provide a PRN dose of acetaminophen for headache E. Withhold the next scheduled daily dose of warfarin.

A. Administer a daily dose of lisinopril as scheduled. D. Provide a PRN dose of acetaminophen for headache Rational: the client' routinely scheduled medication, lisinopril, is an antihypertensive medication and should be administered as scheduled to maintain the client's blood pressure. A PRN dose of acetaminophen should be given for the client's headache. The other options are not indicated for this situation.

A client with a traumatic brain injury becomes progressively less responsive to stimuli. The client has a "Do Not Resuscitate" prescription, and the nurse observes that the unlicensed assistive personnel (UAP) has stopped turning the client from side to side as previously schedules. What action should the nurse take? A. Advise the UAP to resume positioning the client on schedule B. Encourage the UAP to provide comfort care measures only C. Assume total care of the client to monitor neurologic function D. Assign a practical nurse to assist the UAP in turning the client

A. Advise the UAP to resume positioning the client on schedule

The nurse requests a meals tray for a client follows Mormon beliefs and who is on clear liquid diet following abdominal surgery. Which meal item should the nurse request for this client? (Select all that apply) A. Apple juice B. Chicken broth. C. Hot chocolate D. Orange juice E. Black coffee

A. Apple juice B. Chicken broth.

The charge nurse of the Intensive Care Unit is making assignments for the permanent staff and one RN who was floated from a medical unit. The client with which condition is the best to assign to the float nurse? A. Diabetic ketoacidosis and titrated IV insulin infusion B. Emphysema extubated 3 hours ago receiving heated mist C. Subdural hematoma with an intracranial monitoring device D. Acute coronary syndrome treated with vasopressors

A. Diabetic ketoacidosis and titrated IV insulin infusion

A middle-aged woman, diagnosed with Graves' disease, asks the nurse about this condition. Which etiological pathology should the nurse include in the teaching plan about hyperthyroidism? (Select all that apply.) A. Graves' disease, an autoimmune condition, affects thyroid stimulating hormone receptors. B. T3 and T4 hormone levels are increased C. Large protruding eyeballs are a sign of hyperthyroid function D. Weight gain is a common complaint in hyperthyroidism E. Early treatment includes levothyroxine (Synthroid).

A. Graves' disease, an autoimmune condition, affects thyroid stimulating hormone receptors. B. T3 and T4 hormone levels are increased C. Large protruding eyeballs are a sign of hyperthyroid function

The nurse is auscultating a client's lung sounds. Which description should the nurse use to document this sound? A. High pitched or fine crackles. B. Rhonchi C. High pitched wheeze D. Stridor

A. High pitched or fine crackles.

The nurse is caring for a client with hypovolemic shock who is receiving two units of packed red blood cells (RBCs) through a large bore peripheral IV. What action promotes maintenance of the client's cardiopulmonary stability during the blood transfusion? A. Increase the oxygen flow via nasal cannula if dyspnea is present. B. Place in a Trendelenburg position to increase cerebral blood flow C. Monitor capillary glucose measurements hourly during transfusion. D. Encourage increased intake of oral fluid to improve skin turgor.

A. Increase the oxygen flow via nasal cannula if dyspnea is present.

In planning strategies to reduce a client's risk for complications following orthopedic surgery, the nurse recognizes which pathology as the underlying cause of osteomyelitis? A. Infectious process B. Metastatic process C. Autoimmune disorder D. Inflammatory disorder

A. Infectious process

The nurse is preparing a 4-day-old I infant with a serum bilirubin level of 19 mg/dl (325 micromol/L) for discharge from the hospital. When teaching the parents about home phototherapy, which instruction should the nurse include in the discharge teaching plan? A. Reposition the infant every 2 hours. B. Perform diaper changes under the light. C. Feed the infant every 4 hours. D. Cover with a receiving blanket.

A. Reposition the infant every 2 hours. Rational: An infant, who is receiving phototherapy for hyperbilirubinemia, should be repositioned every two hours. The position changes ensure that the phototherapy lights reach all of the body surface areas. Bathing, feedings, and diaper changes are ways for the parents to bond with the infant, and can occur away from the treatment. Feedings need to occur more frequently than every 4 hours to prevent dehydration. The infant should wear only a diaper so that the skin is exposed to the phototherapy.

Which assessment finding for a client who is experiencing pontine myelinolysis should the nurse report to the healthcare provider? A. Sudden dysphagia B. Blurred visual field C. Gradual weakness D. Profuse diarrhea

A. Sudden dysphagia

***A male client who was admitted with an acute myocardial infarction receives a cardiac diet with sodium restriction and complains that his hamburger is flavorless. Which condiment should the nurse offer?

Fresh horseradish

An unlicensed assistive personnel (UAP) assigned to obtain client vital signs reports to the charge nurse that a client has a weak pulse with a rate of 44 beat/ minutes. What action should the charge nurse implement?

Assign a practical nurse (LPN) to determine if an apical radial deficit is present

Several months after a foot injury, and adult woman is diagnosed with neuropathic pain. The client describes the pain as severe and burning and is unable to put weight on her foot. She asks the nurse when the pain will "finally go away." How should the nurse respond?

Assist the client in developing a goal of managing the pain

While removing staples from a male client's postoperative wound site, the nurse observes that the client's eyes are closed and his face and hands are clenched. The client states, "I just hate having staples removed." After acknowledging the client's anxiety, what action should the nurse implement?

Attempt to distract the client with general conversation

A 3-year-old boy with a congenital heart defect is brought to the clinic by his mother...During the assessment, the mother asks the nurse why her child is at the 5th percent...response is best for the nurse to provide? A. Does your child seem mentally slower than his peers also? B. "His smaller size is probably due to the heart disease" C. Haven't you been feeding him according to recommended daily allowances for children? D. You should not worry about the growth tables. They are only averages for children

B. "His smaller size is probably due to the heart disease"

A female client comes to the clinic complaining of fatigue and inability to sleep because she is the full-time caretaker for 22-year-old son who was paralyzed by a motor vehicle collision. She adds that her husband left her because he says he can't take her behavior any more since all she does is care for their son. What intervention should the nurse implement? A. Schedule a home visit in the afternoon to assess the son and client role as caregiver. B. Acknowledge the client's stress and suggest that she consider respite care. C. Provide feedback to the client about her atonement for guilt about her son's impairment. D. Teach the client to problem-solve for herself and establish her own priorities.

B. Acknowledge the client's stress and suggest that she consider respite care. Rationale: When this amount of disclosure is offered, the client is usually seeking information focuses on the client's expression of worry, concern and stress and addresses the client's need to initiate a request for assistance with respite care.

A client is scheduled to receive an IW dose of ondansetron (Zofran) eight hours after receiving chemotherapy. The client has saline lock and is sleeping quietly without any restlessness. The nurse caring for the client is not certified in chemotherapy administration. What action should the nurse take? A. Ask a chemotherapy-certified nurse to administer the Zofran B. Administer the ondasentron (Zofran) after flushing the saline lock with saline C. Hold the scheduled dose of Zofran until the client awakens D. Awaken the client to assess the need for administration of the Zofran.

B. Administer the ondasentron (Zofran) after flushing the saline lock with saline

The nurse provides feeding tube instructions to the wife of a client with end stage cancer. The client's wife performs a return demonstration correctly, but begins crying and tells the nurse, "I just don't think I can do this every day." The nurse should direct further teaching strategies toward which learning domain? A. Cognitive B. Affective C. Comprehension D. Psychomotor

B. Affective

An adult who is 5 feet 5 inches (165.1 cm) tall and weighs 90 lb. (40.8 Kg) is admitted with a diagnosis of chronic anorexia. The client receives a regular diet for 2 days, and the client's medical records indicates that 100% of the diet provided has been consumed. However the client's weight on the third day morning after admission is 89 lb. (40.4 Kg). What action should the nurse implement? A. Examine the client's room for hidden food. B. Assign staff to monitor what the client eats. C. Ask the client if the food provided is being eaten or discarded. D. Provide the client with a high calorie diet.

B. Assign staff to monitor what the client eats. Rationale: clients with an eating disorder have an unhealthy obsession with food. The client's continued weight loss, despites indication that the client has consumed 100% of the diet, should raise questions about the client's intake of the food provided, so the client should be observed during meals to prevent hiding or throwing away food. Other options may be accurate but ineffective and unnecessary.

A client in the intensive care unit is being mechanically ventilated, has an indwelling urinary catheter in place, an exhibiting signs of restlessness. Which action should the nurse take fist? A. Administer PRN dose of lorazepam B. Auscultate bilateral breath sounds C. Check urinary catheter for obstruction D. Review the heart rhythms on cardiac monitor.

B. Auscultate bilateral breath sounds

***The mother of the 12- month-old with cystic fibrosis reports that her child is experiencing increasing congestion despite the use of chest physical therapy (CPT) twice a day, and has also experiences a loss of appetite. What instruction should the nurse provide? A. Perform CPT after meals to increase appetite and improve food intake. B. CPT should be performed more frequently, but at least an hour before meals. C. Stop using CPT during the daytime until the child has regained an appetite. D. Perform CPT only in the morning, but increase frequency when appetite improves.

B. CPT should be performed more frequently, but at least an hour before meals. Rationale: CPY with inhalation therapy should be performed several times a day to loosen the secretions and move them from the peripheral airway into the central airways where they can be expectorated. CPT should be done at least one hour before meals or two hours after meals.

***A young adult female with chronic kidney disease (CKD) due to recurring pyelonephritis is hospitalized with basilar crackles and peripheral edema. She is complaining of severe nausea and the cardiac monitor indicates sinus tachycardia with frequent premature ventricular contraction. Her blood pressure is 200 /110 mm Hg, and her temperature is 101 F which PRN medication should the nurse administers first? A. Enalapril B. Furosemide C. Acetaminophen D. Promethazine

B. Furosemide

During a staff meeting, a nurse verbally attacks the nurse manager conducting the meeting, stating, "you always let your favorites have holidays off give then easier assignments. You are unfair and prejudiced" how should the nurse-manager respond? A. I would prefer to discuss this with you privately. B. Give me specific examples to support your statements. C. Does anyone else on the staff fell the same way D. Your remarks are not true and are very unkind

B. Give me specific examples to support your statements.

A female client presents in the Emergency Department and tells the nurse that she was raped last night. Which question is most important for the nurse to ask? A. Does she knows the person who raped her? B. Has she taken a bath since the raped occurred? C. Is the place where she lived a safe place? D. Did she report the rape to the police Department?

B. Has she taken a bath since the raped occurred? RATIONALE: The priority action is collected the forensic evidence, so asking if the has taken a bath since the rape occurred is the most important information to obtain. Other options are used by law enforcement to determine the perpetrator and are not vital in providing client care at this time.

***A client with arthritis has been receiving treatment with naproxen and now reports ongoing stomach pain, increasing weakness, and fatigue. Which laboratory test should the nurse monitor? A. Sed rate (ESR) B. Hemoglobin C. Calcium D. Osmolality.

B. Hemoglobin Rational: naproxen can cause gastric bleeding, so the nurse should monitor the client's hemoglobin to assess for possible bleeding. Other options are not likely to be affected by the used of naproxen and are not related to the client's current symptoms.

During a well-baby, 6-month visit, a mother tells the nurse that her infant has had fewer ear infections than her 10-year-old daughter. The nurse should explain that which vaccine is likely to have made the difference in the siblings' incidence of otitis media? A. Varicella Virus Vaccine Live B. Hemophilic Influenza Type B (HiB) vaccine C. Pneumococcal vaccine D. Palivizumab vaccine for RSV

B. Hemophilic Influenza Type B (HiB) vaccine

The psychiatric nurse is talking to a newly admitted client when a male client diagnosed with antisocial behavior intrudes on the conversation and tells the nurse, "I have to talk to you right now! It is very important!" how should the nurse respond to this client? A. Put his behavior on extinction and continue talking with the newly admitted. B. Inform him that the nurse is busy admitting a new client and will talk to him later. C. Encourage him to go to the nurse's station and talk with another nurse. D. Introduce him to the newly admitted client and ask him to him to join in the conversation.

B. Inform him that the nurse is busy admitting a new client and will talk to him later. Rational: the psychiatric nurse must set limits with antisocial behavior so that appropriate behavior is demonstrated. Interrupting a conversation is rude and inappropriate, so telling the client that they can talk later is the best course of action. Other options may cause the client to become angry and they do not address the client's behavior. The nurse should not involve this client with newly admitted client's admission procedure.

A male client with a long history of alcoholism is admitted because of mild confusion and fine motor tremors. He reports that he quit drinking alcohol and stopped smoking cigarettes one month ago after his brother died of lung cancer. Which intervention is most important for the nurses to include in the client's plan of care? A. Determine client's level current blood alcohol level. B. Observe for changes in level of consciousness. C. Involve the client's family in healthcare decisions. D. Provide grief counseling for client and his family.

B. Observe for changes in level of consciousness. Rationale: Based on the client's history of drinking, he may be exhibiting sing of hepatic involvement and encephalopathy. Changes in the client's level of consciousness should be monitored to determine if he able to maintain consciousness, so neurological assessment has the highest priority.

To reduce staff nurse role ambiguity, which strategy should the nurse manager implemented? A. Confirm that all the staff nurses are being assigned to equal number of clients. B. Review the staff nurse job description to ensure that it is clear, accurate, and recurrent. C. Assign each staff nurse a turn unit charge nurse on a regular, rotating basis. D. Analyze the amount of overtime needed by the nursing staff to complete assignments.

B. Review the staff nurse job description to ensure that it is clear, accurate, and recurrent. Rationale: Role ambiguity occurs when there is inadequate explanation of job descriptions and assigned tasks, as well as the rapid technological changes that produce uncertainty and frustration. A and D may be implemented if the nurse manager is concerned about role overload, which is the inability to accomplish the tasks related to one's role. C is not related to ambiguity.

A nurse stops at the site of a motorcycle accident and finds a young adult male lying face down in the road in a puddle of water. It is raining, no one is available to send for help, and the cell phone is in the car about 50 feet away. What action should the nurse take first? A. Examine the victim's body surfaces for arterial bleeding B. Stabilize the victim's neck and roll over to evaluate his status C. Return to the car to call emergency response 911 for help D. Open the airway and initiate resuscitative measures

B. Stabilize the victim's neck and roll over to evaluate his status

A woman with an anxiety disorder calls her obstetrician's office and tells the nurse of increased anxiety since the normal vaginal delivery of her son three weeks ago. Since she is breastfeeding, she stopped taking her antianxiety medications, but thinks she may need to start taking them again because of her increased anxiety. What response is best for the nurse to provide this woman? A. Describe the transmission of drugs to the infant through breast milk B. Encourage her to use stress relieving alternatives, such as deep breathing exercises C. Inform her that some antianxiety medications are safe to take while breastfeeding D. Explain that anxiety is a normal response for the mother of a 3-week-old.

C. Inform her that some antianxiety medications are safe to take while breastfeeding Rationale: there are several antianxiety medications that are not contraindicated for breastfeeding mothers.

When entering a client's room to administer an 0900 IV antibiotic, the nurse finds that the client is engaged in sexual activity with a visitor. Which actions should the nurse implement? A. Ignore the behavior and hang the IV antibiotic B. tell the client to stop the inappropriate behavior C. Leave the room and close the door quietly D. Complete an unusual occurrence report

C. Leave the room and close the door quietly

While performing a skin inspection for a female adult client, the nurse observes a rash that is well circumscribed, has silvery scales and plaques, and is located on the elbows and knees. These assessment findings are likely to indicate which condition? A. Tinea corporis B. Herpes zoster C. Psoriasis D. Drug reaction

C. Psoriasis

An unlicensed assistive personnel (UAP) reports that a client's right hand and fingers spasms when taking the blood pressure using the same arm. After confirming the presence of spams what action should the nurse take? A. Ask the UAP to take the blood pressure in the other arm B. Tell the UAP to use a different sphygmomanometer. C. Review the client's serum calcium level D. Administer PRN antianxiety medication.

C. Review the client's serum calcium level

The nurse makes a supervisory home visit to observe an unlicensed assistive personnel (UAP) who is providing personal care for a client with Alzheimer's disease. The nurse observes that whenever the client gets upset, the UAP changes the subject. What action should the nurse take in response to this observation? A. Tell the UAP to offer more choices during the personal care to prevent anxiety B. Meet with the UAP later to role model more assertive communication techniques C. Assume care of the client to ensure that effective communication is maintained. D. Affirm that the UAP is using and effective strategy to reduce the client's anxiety.

D. Affirm that the UAP is using and effective strategy to reduce the client's anxiety. Rationale: Reduction is an effective technique is managing the anxiety of client with Alzheimer's disease, so the nurse should affirm the UAP is using an effective strategy (A). Nurse assertive communication and offering more choices (B) may increase... an agitation (C) is not indicated since the UAP is using redirection, an effective strategy.

The nurse and an unlicensed assistive personnel (UAP) are providing care for a client with a nasogastric tube (NGT) when the client begins to vomit. How should the nurse manage this situation? A. Determine the presence of hematemesis as the UAP irrigates the NGT B. Instruct the UAP to bring an antiemetic to the nurse at the bedside C. Assess the appearance of the emesis while the UAP checks bowel sounds D. Direct the UAP to measure the emesis while the nurse irrigates the NGT

D. Direct the UAP to measure the emesis while the nurse irrigates the NGT

The nurse is evaluating the health teaching of a female client with condyloma acuminate. Which statement by the client indicates that teaching has been effective? A. Early treatment is very effective B. I will clean my hot tub better C. These warts are caused by a fungus D. I need to have regular pap smears

D. I need to have regular pap smears

When assessing a mildly obese 35-year-old female client, the nurse is unable to locate the gallbladder when palpating below the liver margin at the lateral border of the rectus abdominal muscle. What is the most likely explanation for failure to locate the gallbladder by palpation? A. The client is too obese B. Palpating in the wrong abdominal quadrant C. Deeper palpation technique is needed D. The gallbladder is normal

D. The gallbladder is normal Rationale: a normal healthy gallbladder is not palpable

During a visit to the planned parenthood clinic, a young woman tells the nurse that she is going to discontinue taking the oral contraceptives she has taken for three years because she wants to get pregnant. History indicates that her grandfather has adult onset diabetes and that she was treated for chlamydia six months ago, which factor in this client's history poses the greatest risk for this woman's pregnancy? A. Family history of adult onset diabetes. B. Treatment for chlamydia in the past year C. Client's age and previous sexual behavior D. Three year history of taking oral contraceptives

D. Three year history of taking oral contraceptives

Which assessment is more important for the nurse to include in the daily plan of care for a client with a burned extremity?

Distal pulse intensity

A client in the emergency center demonstrates rapid speech, flight of ideas, and reports sleeping only three hours during the past 48h. Based on these finding, it is most important for the nurse to review the laboratory value for which medication?

Divalproex. Rationale: divalproex is the first line of treatment for bipolar disorder BPD because it has a high therapeutic index, few side effects, and a rapid onset in controlling symptoms and preventing recurrent episodes of mania and depression. The serum value of divalproex should be determined since the client is exhibiting symptoms of mania, which may indicate non-compliance with the medication regimen.

When performing postural drainage on a client with Chronic Obstructive Pulmonary Disease (COPD), which approach should the nurse use?

Explain that the client may be placed in five positions

Which action should the school nurse take first when conducting a screening for scoliosis?

Inspect for symmetrical shoulder height.

A client with leukemia undergoes a bone marrow biopsy. The client's laboratory values indicate the client has thrombocytopenia. Based on this data, which nursing assessment is most important following the procedure? Observe aspiration site. Assess body temperature Monitor skin elasticity Measure urinary output

Observe aspiration site.

The nurse finds a client at 33 weeks gestation in cardiac arrest. What adaptation to cardiopulmonary resuscitation (CPR) should the nurse implement?

Position a firm wedge to support pelvis and thorax at 30 degree tilt.

A client who had a right hip replacement 3 day ago is pale has diminished breath sound over the left lower lung fields, a temperature of 100.2 F, and an oxygen saturation rate of 90%. The client is scheduled to be transferred to a skilled nursing facility (SNF) tomorrow for rehabilitative critical pathway. Based on the client's symptoms, what recommendation should the nurse give the healthcare provider?

Reassess readiness for SNF transfer.

The nurse enters a client's room and observes the client's wrist restraint secured as seen in the picture. What action should the nurse take?

Reposition the restraint tie onto the bedframe.

A client with intestinal obstructions has a nasogastric tube to low intermittent suction and is receiving an IV of lactated ringer's at 100 ml/H. which finding is most important for the nurse to report to the healthcare provider?

Serum potassium level of 3.1 mEq/L or mmol/L (SI) Rationale: The normal potassium level in the blood is 3.5-5.0 milliEquivalents per liter (mEq/L).

A male client who is admitted to the mental health unit for treatment of bipolar disorder has a slightly slurred speech pattern and an unsteady gait. Which assessment finding is most important for the nurse to report to the healthcare provider?

Serum lithium level of 1.6 mEq/L or mmol/l (SI) Rationale: The therapeutic level of Serum lithium is 0.8 to 1.5 mEq/L or mmol/l (SI). Slurred speech and ataxia are sign of lithium toxicity.

A client who was admitted yesterday with severe dehydration is complaining of pain a 24 gauge IV with normal saline is infusing at a rate of 150 ml/hour. Which intervention should the nurse implement first?

Stop the normal saline infusion.

A 4-year-old with acute lymphocytic leukemia (ALL) is receiving a chemotherapy (CT) protocol that includes methotrexate (Mexate, Trexal, MIX), an antimetabolite. Which information should the nurse provide the parents about caring for their child?

Use sunblock or protective clothing when outdoors.

***Azithromycin is prescribed for an adolescent female who has lower lobe pneumonia and recurrent chlamydia. What information is most important for the nurse to provide to this client?

Use two forms of contraception while taking this drug.

***Which client should the nurse assess frequently because of the risk for overflow incontinence? A client

Who is confused and frequently forgets to go to the bathroom

The healthcare provider prescribes the antibiotic cephradine 500mg PO every 6 hours for a client with a postoperative wound infection. Which foods should the nurse encourage this client to eat?

Yogurt and/or buttermilk.

Following a lumbar puncture, a client voices several complaints. What complaint indicated to the nurse that the client is experiencing a complication? A. "I have a headache that gets worse when I sit up" B. "I am having pain in my lower back when I move my legs" C. "My throat hurts when I swallow" D.Q "I feel sick to my stomach and am going to throw up"

"I have a headache that gets worse when I sit up"

A primigravida client is 36 weeks gestation is admitted to labor and delivery unit because her membranes ruptured 30minutes ago. Initial assessment indicates 2cm dilation, 50% effaced, -2 station, vertex presentation greenish colored amniotic fluid, and contractions occurring 3-5 minutes with a low FHR after the last contraction peaks: A. Administer Oxygen via face mask B. Apply an internal fetal heart monitor C. Notify the healthcare provider D. Use a vibroacoustic stimulator

A. Administer Oxygen via face mask

After several hours of non-productive coughing, a client presents to the emergency room complaining of chest tightness and shortness of breath. History includes end stage chronic obstructive pulmonary disease (COPD) and diabetes mellitus. While completing the pulmonary assessment, the nurse hears wheezing and poor air movement bilaterally. Which actions should the nurse implement? (Select all that apply.)

A. Administer PRN nebulizer treatment. B. Obtain 12 lead electrocardiogram. C. Monitor continuous oxygen saturation.

Which information is more important for the nurse to obtain when determining a client's risk for (OSAS)? A. Body mass index B. Level of consciousness C. Self-description of pain D. Breath sounds

A. Body mass index

A child is diagnosed with acquired aplastic anemia. The nurse knows that this child has the best prognosis with which treatment regimen? A. Bone marrow transplantation B. Blood transfusion C. Chemotherapy D. Immunosuppressive therapy

A. Bone marrow transplantation

A young adult female client with recurrent pelvic pain for 3 year returns to the clinic for relief of severe dysmenorrhea. The nurse reviews her medical record which indicates that the client has endometriosis. Based on this finding, what information should the nurse provide this client? A) Oral contraceptives increase the symptoms of endometriosis. B) The symptoms of endometriosis can increase with menopause. C) An option to diagnose disease extent and provide therapeutic treatment is laparoscopy. D) Infertile is successfully treated with removal of intra-abdominal endometrial lesions.

A) Oral contraceptives increase the symptoms of endometriosis.

The home health nurse is preparing to make daily visits to a group of clients. Which client should the nurse visit first? A. A client with congestive heart failure who reports a 3 pound weight gain in the last two days B. An immobile client with a stage 3 pressure ulcer on the coccyx who is having low back pain C. A client diagnosed with chronic obstructive pulmonary disease (COPD) who is short of breath D. A terminally ill older adult who has refused to eat or drink anything for the last 48 hours

A. A client with congestive heart failure who reports a 3 pound weight gain in the last two days

The nurse is caring for a group of clients with the help of a practical nurse (PN). Which nursing actions should the nurse assign to the PN? (Select all that apply.) A. Administer a dose of insulin per sliding scale for a client with type 2 diabetes mellitus (DM). B. Obtain postoperative vital signs for a client one day following unilateral knee arthroplasty C. Perform daily surgical dressing change for a client who had an abdominal hysterectomy D. Initiate patient controlled analgesia (PCA) pumps for two clients immediately postoperative E. Start the second blood transfusion for a client twelve hours following a below knee amputation

A. Administer a dose of insulin per sliding scale for a client with type 2 diabetes mellitus (DM). B. Obtain postoperative vital signs for a client one day following unilateral knee arthroplasty C. Perform daily surgical dressing change for a client who had an abdominal hysterectomy

A client diagnosed with calcium kidney stones has a history of gout. A new prescription for aluminum hydroxide (Amphogel) is scheduled to begin at 0730. Which client medication should the nurse bring to the healthcare provider's attention? A. Allopurinol (Zyloprim) B. Aspirin, low dose C. Furosemide (lasix) D. Enalapril (vasote)

A. Allopurinol (Zyloprim)

A male Korean-American client looks away when asked by the nurse to describe his problem. What is the best initial nursing action? A. Allow several minutes for the client to respond B. Ask social services to find a Korean interpreter C. Repeat the question slowly and distinctly D. Establish direct eye contact with the client

A. Allow several minutes for the client to respond

A young adult who is hit with a baseball bat on the temporal area of the left skull is conscious when admitted to the ED and is transferred to the Neurological Unit to be monitored for signs of closed head injury. Which assessment finding is indicative of a developing epidural hematoma? A. Altered consciousness within the first 24 hours after injury. B. Cushing reflex and cerebral edema after 24 hours C. Fever, nuchal rigidity and opisthotonos within hours D. Headache and pupillary changes 48 hours after a head injury

A. Altered consciousness within the first 24 hours after injury.

Which client is at the greatest risk for developing delirium? A. An adult client who cannot sleep due to constant pain. B. n older client who attempted 1 month ago C. A young adult who takes antipsychotic medications twice a day D. A middle-aged woman who uses a tank for supplemental oxygen

A. An adult client who cannot sleep due to constant pain.

In making client care assignment, which client is best to assign to the practical nurse (PN) working on the unit with the nurse? A. An immobile client receiving low molecular weight heparin q12 h. B. A client who is receiving a continuous infusion of heparin and gets out of bed BID C. A client who is being titrated off heparin infusion and started on PO warfarin (Coumadin) D. An ambulatory client receiving warfarin (Coumadin) with INR of 5 second.

A. An immobile client receiving low molecular weight heparin q12 h. Rationale: A describe the most stable client. The other ones are at high risk for bleeding problems and require the assessment skills.

After receiving the Braden scale findings of residents at a long-term facility, the charge nurse should to tell the unlicensed assistive personnel (UAP) to prioritize the skin care for which client? A. An older adult who is unable to communicate elimination needs. B. An older man whose sheets are damped each time he is turned. C. A woman with osteoporosis who is unable to bear weight. D. A poorly nourished client who requires liquid supplement.

A. An older adult who is unable to communicate elimination needs. Rational: a Braden score of less than 18 indicates a risk for skin breakdown, and clients with such score require intensive nursing care. Constant moisture places the client at a high risk for skin breakdown, and interventions should be implemented to pull moisture away from the client's skin. Other options may be risk factors but do not have as high a risk as constant exposure to moisture.

A male client with cancer is admired to the oncology unit and tells the nurse that he is in the hospital for palliative care measures. The nurse notes that the client's admission prescription include radiation therapy. What action should the nurse implement? A. Ask the client about his expected goals for the hospitalization B. Explain the palliative care measures can be provided at home C. Notify do radiation department to withhold the treatment for now D. Determine if the client wishes to cancel further radiation treatment

A. Ask the client about his expected goals for the hospitalization Rationale: Palliative care measures provide relief or control of symptoms, so it is important for the nurse to determine the client's goals for symptom control while receiving treatment in the hospital. Although home care is available the client may not be legible for palliative care at home. Radiation therapy is an effective positive care measure used to manage symptoms and would be appropriate unless the radiation conflicts with the client goals.

A young adult male who is being seen at the employee health care clinic for an annual assessment tell the nurse that his mother was diagnosed with schizophrenia when she was his age and that life with a schizophrenic mother was difficulty indeed. Which response is best for the nurse to provide? A. Ask the client if he is worried about becoming schizophrenic at the age his mother was diagnosed. B. Encourage the client to seek genetic counseling to determine his risk for mental illness. C. Informed the client that his mother schizophrenic has affected his psychological development. D. Tell the client that mental illness has a familial predisposition so he should see a psychiatrist.

A. Ask the client if he is worried about becoming schizophrenic at the age his mother was diagnosed.

While the nurse is conducting a daily assessment of an older woman who resides in a long-term facility, the client begins to cry and tells the nurse that her family has stopped calling and visiting. What action should the nurse take first? A. Ask the client when a family member last visited her. B. Determine the client's orientation to time and space C. Review the client's record regarding social interactions D. Reassure the client of her family's love for her

A. Ask the client when a family member last visited her.

The healthcare provider prescribes oxycodone/ aspirin 1 tab PO every 4h as needed for pain, for a client with polycystic kidney disease. Before administering this medication, which component of the prescription should the nurse question? A. Aspirin content. B. Dose C. Route D. Risk for addiction

A. Aspirin content.

***An adult client with severe depression was admitted to the psychiatric unit yesterday evening. Although the client ran one year ago, his spouse states that the client no longer runs, bur sits and watches television most of the day. Which is most important for the nurse to include in this client's plan of care for today? A. Assist client in identifying goals for the day. B. Encourage client to participate for one hour in a team sport. C. Schedule client for a group that focuses on self-esteem. D. Help client to develop a list of daily affirmations.

A. Assist client in identifying goals for the day. Rationale: clients with severe depression have low energy and benefit from structured activities because concentration is decreased. The client participate in care by identifying goals for the day is the most important intervention for the client's first day at the unit. Other options can be implemented over time, as the depression decreases.

A client who had an open cholecystectomy two weeks ago comes to the emergency department with complaints of nausea, abdominal distention, and pain. Which assessment should the nurse implement? A. Auscultate all quadrant of the abdomen. B. Perform a digital rectal exam C. Palpate the liver and spleen D. Obtain a hemoccult of the client's stool

A. Auscultate all quadrant of the abdomen.

The nurse notes that a client has been receiving hydromorphone (Dilaudid) every six hours for four days. What assessment is most important for the nurse to complete? A. Auscultate the client's bowel sounds B. Observe for edema around the ankles C. Measure the client's capillary glucose level D. Count the apical and radial pulses simultaneously

A. Auscultate the client's bowel sounds Rationale: hydromorphone is a potent opioid analgesic that slows peristalsis and frequently causes constipation, so it is most important to Auscultate the client's bowel sounds

. A male adult is admitted because of an acetaminophen overdose. After transfer to the mental health unit, the client is told he has liver damage. Which information is most important for the nurse to include in the client's discharge plan? A. Avoid exposure to large crowds B. Do not take any over-the-counter medications C. Call the crisis hot line if feeling lonely D. Eat a high carbohydrate, low fat, low protein diet

A. Avoid exposure to large crowds

Which instruction is most important for the nurse to provide a client who is being discharge following treatment for Guillain-Barre syndrome? A. Avoid exposure to respiratory infections B. Use relaxation exercises when anxious C. Plan short, frequent rest periods D. Continue physical therapy at home

A. Avoid exposure to respiratory infections

The home care nurse provide self-care instruction for a client chronic venous insufficiency cause by deep vein thrombosis. Which instructions should the nurse include in the client's discharge teaching plan? Select all that apply A. Avoid prolonged standing or sitting B. Use recliner for long period of sitting C. Continue wearing elastic stocking D. Maintain the bed flat while sleeping E. Cross legs at knee but not at ankle

A. Avoid prolonged standing or sitting B. Use recliner for long period of sitting C. Continue wearing elastic stocking

In conducting a health assessment, the nurse determines that both parents of a child with asthma smoke cigarettes. What recommendation is best to the nurse to recommend to the parents? A. Avoid smoking in the house B. Stop smoking immediately C. Decrease the number of cigarettes smoke daily D. Obtain nicotine patches to assist in smoking sensation

A. Avoid smoking in the house

Which statement is accurate regarding the pathological changes in the pulmonary system associated with acute (adult) respiratory distress syndrome (ARDS)? A. Capillary hydrostatic pressure exceeds colloid osmotic pressure, producing interstitial edema B. A high ventilation-to-perfusion ratio is characteristic of affected lung fields in ARDS C. Functional residual capacity and lung compliance increase as the disease progresses D. Interstitial edema that occurs due to capillary fluid shifts is usually more serious than alveolar edema

A. Capillary hydrostatic pressure exceeds colloid osmotic pressure, producing interstitial edema

A postpartal client complains that she has the urge to urinate every hour but is only able to void a small amount. What interventions provides the nurse with the most useful information? A. Catheterize for residual urine after next voiding B. Initiate a perineal pad count C. Assess for a perineal hematoma D. Determine the client's usual voiding pattern

A. Catheterize for residual urine after next voiding

The nurse is preparing to administer an infusion of amino acid-dextrose total parenteral nutrition (TPN) through a central venous catheter (CVC) line. Which action should the nurse implement first? A. Check the TPN solution for cloudiness B. Attach the IV tubing to the central line C. Set the infusion pump at the prescribed rate D. Prime the IV tubing with TPN solution

A. Check the TPN solution for cloudiness

A mother brings her 6-year-old child, who has just stepped on a rusty nail, to the pediatrician's office. Upon inspection, the nurse notes that the nail went through the shoe and pierced the bottom of the child's foot. Which action should the nurse implement first? A. Cleanse the foot with soap and water and apply an antibiotic ointment B. Provide teaching about the need for a tetanus booster within the next 72 hours. C. Have the mother check the child's temperature q4h for the next 24 hours D. Transfer the child to the emergency department to receive a gamma globulin injection

A. Cleanse the foot with soap and water and apply an antibiotic ointment

A male client is admitted for the removal of an internal fixation that was inserted for the fracture ankle. During the admission history, he tells the nurse he recently received vancomycin (vancomycin) for a methicillin-resistant Staphylococcus aureus (MRSA) wound infection. Which action should the nurse take? (Select all that apply.) A. Collect multiple site screening culture for MRSA B. Call healthcare provider for a prescription for linezolid (Zyrovix) C. Place the client on contact transmission precautions D. Obtain sputum specimen for culture and sensitivity E. Continue to monitor for client sign of infection.

A. Collect multiple site screening culture for MRSA C. Place the client on contact transmission precautions E. Continue to monitor for client sign of infection. Rationale: Until multi-site screening cultures come back negative (A), the client should be maintained on contact isolation(C) to minimize the risk for nosocomial infection. Linezolid (Zyvox), a broad spectrum anti-infectant, is not indicated, unless the client has an active skin structure infection cause by MRSA or multidrug- resistant strains (MDRSP) of Staphylococcus aureus. A sputum culture is not indicated9D) based on the client's history is a wound infection.

A client is admitted with acute pancreatitis. The client admits to drinking a pint of bourbon daily. The nurse medicates the client for pain and monitors vital signs q2 hours. Which finding should the nurse report immediately to the healthcare provider? A. Confusion and tremors B. Yellowing and itching of skin. C. Abdominal pain and vomiting D. Anorexia and abdominal distention

A. Confusion and tremors Rationale: daily alcohol is the likely etiology for the client's pancreatitis. Abrupt cessation of alcohol can result in delirium tremens (DT) causing confusion and tremors, which can precipitate cardiovascular complications and should be reported immediately to avoid life-threatening complications. The other options are expected findings in those with liver dysfunction or pancreatitis, but do not require immediate action.

A female client with severe renal impairment is receiving enoxaparin (lovenox) 30 mg SUBQ BID. Which laboratory value due to enoxaparin should the nurse report to the healthcare provider? A. Creatinine clearance 25 mL/ minute B. Calcium 9 mg/dl C. Hemoglobin 12 grams/dl D. Partial thromboplastin time (PTT) 30 seconds

A. Creatinine clearance 25 mL/ minute

The nurse is assessing the emotional status of a client with Parkinson's disease. Which client finding is most helpful in planning goals to meet the client's emotional needs? A. Cries frequently during the interview B. Stares straight ahead without blinking C. Face does not convey any emotion D. Uses a monotone when speaking

A. Cries frequently during the interview

A client is admitted with a wound on the right hand and associated cellulitis. In assessing the client's hand, which finding required most immediate follow-up by the nurse? A. Cyanotic nailbeds B. Localized tenderness C. Diffuse erythema D. Skin hot to touch

A. Cyanotic nailbeds

Following insertion of a LeVeen shunt in a client with cirrhosis of the liver, which assessment finding indicates to the nurse that the shunt is effective? A. Decrease abdominal girth B. Increased blood pressure C. Clear breath sounds D. Decrease serum albumin.

A. Decrease abdominal girth

An elderly male client is admitted to the mental health unit with a sudden onset of global disorientation and is continuously conversing with his mother, who died 50 years ago. The nurse reviews the multiple prescriptions he is currently taking and assesses his urine specimen, which is cloudy, dark yellow, and has foul odor. These findings suggest that his client is experiencing which condition? A. Delirium B. Depression C. Dementia D. Psychotic episode

A. Delirium

***A young couple who has been unsuccessful in conceiving a child for over a year is seen in the family planning clinic. During an initial visit, which intervention is most important for the nurse to implement? A. Determine current sexual practice B. Prepare a female client for an ultrasound C. Request an sperm sample for ovulation D. Evaluate hormone levels on both client

A. Determine current sexual practice Rationale: First a history should be obtained including practices that might be related to the infertility, such as douching, daily ejaculation or the male partner's exposure to heat, such as frequent sauna or work environment which can decrease sperm production (A B or C) may be indicated after a complete assessment is obtained.

A client with persistent low back pain has received a prescription for electronic stimulator (TENS) unit. After the nurse applies the electrodes and turns on the power, the client reports feeling a tingling sensation. How should the nurse respond? A. Determine if the sensation feels uncomfortable. B. Decrease the strength of the electrical signals. C. Remove electrodes and observe for skin redness. D. Check the amount of gel coating on the electrodes.

A. Determine if the sensation feels uncomfortable. Rational: electronic stimulators, such as a transelectrical nerve stimulator (TENS) unit, have been found to be effective in reducing low back pain by "closing the gate" to pain stimuli. A tingling sensation should be felt when the power is turned on, and the nurse should assess whether the sensation is too strong, causing discomfort or muscle twitching. Decreasing the electrical signal may be indicated if the sensation is too strong. Other options are not necessary because the tingling sensation is expected.

The nurse is caring for four clients...postoperative hemoglobin of 8.7 mg/dl; client C, newly admitted with potassium...an appendectomy who has a white blood cell count of 15,000mm3. What intervention... A. Determine the availability of two units of packed cells in the blood bank for client B B. Increase the oxygen flow rate to 4 liters/minute per face mask for client A C. Remove any foods, such as banana or orange juice, for the breakfast tray for client C D. Inform client D that surgery is likely to be delayed until the infection responds to antibiotics

A. Determine the availability of two units of packed cells in the blood bank for client B

A male client was transferred yesterday from the emergency department to the telemetry unit because he had ST depression and resolved chest pain. When his EKG monitor alarms for ventricular tachycardia (VT), what action should the nurse take first? A. Determine the client's responsiveness and respirations B. Bring the crash cart to the room to defibrillate the client. C. Immediately initiate chest compressions. D. Notify the emergency response team

A. Determine the client's responsiveness and respirations Rationale: Activities, such as brushing teeth, can mimic the waveform of VI, so first he client should be assessed (A) to determine if the alarm is accurate. The crash cart can be brought to the room by someone else and defibrillation (B) delivered as indicated by the client's rhythm. Based on as assessment of the client, CPR© as summoning the emergency response team (D) may be indicated.

To prevent infection by auto contamination during the acute phase of recovery from multiple burns, which intervention is most important for the nurse to implement? A. Dress each wound separately. B. Avoid sharing equipment between multiple clients. C. Use gown, mask and gloves with dressing change. D. Implement protective isolation.

A. Dress each wound separately. Rational: each wound should be dressed separately using a new pair of sterile glove to avoid auto contamination (the transfer of microorganisms form one infected wound to a non-infected wound). The other choices do not prevent auto contamination.

A client in her first trimester of pregnancy complains of nausea. Which complementary therapy should the nurse recommend? A. Drink chamomile tea at breakfast and in the evening. B. Eat food high in garlic with the evening meal C. Join a yoga class that meets at least weekly D. Increase cocoa in the diet and drink before bedtime

A. Drink chamomile tea at breakfast and in the evening.

The nurse assesses a 78-year-old male client who has left sides heart failure. Which symptoms would the nurse expect this client to exhibit? A. Dyspnea, cough, and fatigue. B. Hepatomegaly and distended neck veins C. Pain over the pericardium and friction rub. D. Narrowing pulse pressure and distant heart sounds.

A. Dyspnea, cough, and fatigue.

A male client with cancer, who is receiving antineoplastic drugs, is admitted to the...what findings is most often manifest this condition? A. Ecchymosis and hematemesis B. Weight loss and alopecia C. Weakness and activity intolerance D. Sore throat and fever

A. Ecchymosis and hematemesis

A client is admitted to the surgical unit with symptoms of a possible intestinal obstruction. When preparing to insert a nasogastric (NG) tube, which intervention should the nurse implement? A. Elevate the head of the bed 60 to 90 degrees B. Measure from corner of mouth to angle of jaw C. Administer a PRN analgesic D. Assess for a gag reflex

A. Elevate the head of the bed 60 to 90 degrees

Before leaving the room of a confused client, the nurse notes that a half bow knot was used to attach the client's wrist restraints to the movable portion of the client's bed frame. What action should the nurse take before leaving the room? A. Ensure that the knot can be quickly released. B. Tie the knot with a double turn or square knot. C. Move the ties so the restraints are secured to the side rails. D. Ensure that the restraints are snug against the client's wrist.

A. Ensure that the knot can be quickly released.

In monitoring tissue perfusion in a client following an above the knee amputation (aka), which action should the nurse include in the plan of care? A. Evaluate closet proximal pulse. B. Asses skin elasticity of the stump. C. Observe for swelling around the stump. D. Note amount color of wound drainage.

A. Evaluate closet proximal pulse. Rationale: A primary focus of care for a client with an AKA is monitoring for signs of adequate tissue perfusion, which include evaluating skin color and ongoing assessment of pulse strength.

The nurse ask the parent to stay during the examination of a male toddler's genital area. Which intervention should the nurse implement? A. Examine the genitalia as the last part of the total exam. B. Use soothing statements to facilitate cooperation C. Allow the child to keep underpants on to examine genitalia D. Work slowly and methodically so not to stress the child

A. Examine the genitalia as the last part of the total exam.

The nurse is developing a plan of care for a middle-aged woman who is diagnosed with type 2 diabetes mellitus (DM). To lower her blood glucose and increase her serum high-density lipoprotein (HDL) levels, which instruction is most important for the nurse to provide? A. Exercise at least three times weekly B. Monitor blood glucose levels daily C. Limit intake of foods high in saturated fat D. Learn to read all food product labels

A. Exercise at least three times weekly

A young adult woman visits the clinic and learns that she is positive for BRCA1 gene mutation and asks the nurse what to expect next. How should the nurse respond? A. Explain that counseling will be provided to give her information about her cancer risk B. Gather additional information about the client's family history for all types of cancer. C. Offer assurance that there are a variety of effective treatments for breast cancer. D. Provide information about survival rates for women who have this genetic mutation.

A. Explain that counseling will be provided to give her information about her cancer risk Rational: BRACA1or BRACA2 genetic mutation indicates an increased risk for developing breast or ovarian cancer and genetic counseling should be provided to explain the increased risk (A)to the client along with options for increased screening or preventative measures. (B) Is completed by the genetic counselor before the client undergoes genetic testing. a positive BRACA1test is not an indicator of the presence of cancer and (C and D) are not appropriate responses prior to genetic counseling.

An adult male was diagnosed with stage IV lung cancer three weeks ago. His wife approaches the nurse and asks how she will know that her husband's death is imminent because their two adult children want to be there when he dies. What is the best response by the nurse? A. Explain that the client will start to lose consciousness and his body system will slow down B. Reassure the spouse that the healthcare provider will let her know when to call the children C. Offer to discuss the client's health status with each of the adult children D. Gather information regarding how long it will take for the children to arrive

A. Explain that the client will start to lose consciousness and his body system will slow down Rationale: Expected signs of approaching death include noticeable changes in the client's level of consciousness and a slowing down of body systems. The nurse should answer the spouse's questions about the signs of imminent death rather than offering reassurance that may or may not be true. Other options listed may be implemented but the nurse should first answer the spouse's question directly.

A client with type 2 diabetes mellitus is admitted for frequent hyperglycemic episodes and a glycosylated hemoglobin (HbA1c) of 10%. Insulin glargine 10 units subcutaneously once a day at bedtime and a sliding scale with insulin aspart q6h are prescribed. What action should the nurse include in this client's plan of care? A. Fingerstick glucose assessment q6h with meals B. Mix bedtime dose of insulin glargine with insulin aspart sliding scale dose C. Review with the client proper foot care and prevention of injury D. Do not contaminate the insulin aspart so that it is available for iv use E. Coordinate carbohydrate controlled meals at consistent times and intervals F. Teach subcutaneous injection technique, site rotation and insulin management

A. Fingerstick glucose assessment q6h with meals C. Review with the client proper foot care and prevention of injury E. Coordinate carbohydrate controlled meals at consistent times and intervals F. Teach subcutaneous injection technique, site rotation and insulin management

When conducting diet teaching for a client who was diagnosed with a myocardial infarction, which snack foods should the nurse encourage the client to eat? (Select all that apply). A. Fresh turkey slices and berries B. Fresh vegetables with mayonnaise dip C. Soda crackers and peanut butter D. Chicken bouillon soup and toast E. raw unsalted almonds and apples

A. Fresh turkey slices and berries D. Chicken bouillon soup and toast E. raw unsalted almonds and apples

A neonate with a congenital heart defect (CHD) is demonstrating symptoms of heart failure (HF). Which interventions should the nurse include in the infant's plan of care? SATA A. Give O2 at 6 L/nasal cannula for 3 repeated oximetry screens below 90% B. Administer diuretics via secondary infusion in the morning only C. Evaluate heart rate for effectiveness of cardio tonic medications D. Use high energy formula 30 calories/ounce at Q3 hours feeding via soft nipples E. Ensure Interrupted and frequent rest periods between procedures.

A. Give O2 at 6 L/nasal cannula for 3 repeated oximetry screens below 90% C. Evaluate heart rate for effectiveness of cardio tonic medications D. Use high energy formula 30 calories/ounce at Q3 hours feeding via soft nipples E. Ensure Interrupted and frequent rest periods between procedures. Rationale: Pulse oximetry screening supports prescribed level of O2. HR provides an evaluative criterion for cardiac medications, which reduce heart rate, increase strength contractions (inotropic effects) and consequently affect systemic circulation and tissue oxygenation. Breast milk or basic formula provide 20 calories/ounce, so frequent feedings with high energy formula. D minimize fatigue is necessary.

The client with which type of wound is most likely to need immediate intervention by the nurse? A. Laceration B. Abrasion C. Contusion D. Ulceration

A. Laceration Rationale: A laceration is a wound that is produced by the tearing of soft body tissue. This type of wound is often irregular and jagged. A laceration wound is often contaminated with bacteria and debris from whatever object caused the cut.

***The nurse is triaging clients in an urgent care clinic. The client with which symptoms should be referred to the health care provider immediately? A. Headache, photophobia, and nuchal rigidity B. High fever, skin rash, and a productive cough C. Nausea, vomiting, and poor skin turgor D. Malaise, fever, and stiff, swollen joints

A. Headache, photophobia, and nuchal rigidity Rationale: Headache, photophobia, and nuchal rigidity are classic signs of meningeal infection, so this client should immediately be referred to the health care provider. AC D do not have priority of B

After placement of a left subclavian central venous catheter (CVC), the nurse receives report of the x-ray findings that indicate the CVC tip is in the client's superior vena cava. Which action should the nurse implement? A. Initiate intravenous fluid as prescribed B. Notify the HCP of the need to reposition the catheter C. Remove the catheter and apply direct pressure for 5 minute D. Secure the catheter using aseptic technique

A. Initiate intravenous fluid as prescribed Rationale: Venous blood return to the heart and drains from the subclavian vein into the superior vena cava. The X-ray findings indicate proper placement of the CVC, so prescribed intravenous fluid can be started. A and B are not indicated at this time. The catheter should be secure immediate following insertion (C)

An adult is admitted to the emergency department following ingestion of a bottle of antidepressants secondary to chronic paint. A nasogastric tube and a left subclavian venous catheter are placed. The nurse auscultates audible breath sounds on the right side, faint sounds procedure should the nurse prepare for first? A. Insertion of a left- sided chest tube. B. Placement of an endotracheal tube. C. Retraction of the nasogastric tube D. Setup of patient- controlled analgesia

A. Insertion of a left- sided chest tube.

The nurse is preparing a client who had a below-the-knee (BKA) amputation for discharge to home. Which recommendations should the nurse provide this client? (Select all that apply) A. Inspect skin for redness B. Use a residual limb shrinker C. Apply alcohol to the stump after bathing D. Wash the stump with soap and water E. Avoid range of motion exercises

A. Inspect skin for redness B. Use a residual limb shrinker D. Wash the stump with soap and water Rationale: Several actions are recommended for home care following an amputation. The skin should be inspected regularly for abnormalities such as redness, blistering, or abrasions. A residual limb shrinker should be applied over the stump to protect it and reduce edema. The stump should be washed daily with a mild soap and carefully rinse and dried. The client should avoid cleansing with alcohol because it can dry and crack the skin. Range of motion should be done daily.

The mother of a 7-month-old brings the infant to the clinic because the skin in the diaper area is excoriated and red, but there are no blisters or bleeding. The mother reports no evidence of watery stools. Which nursing intervention should the nurse implement? A. Instruct the mother to change the child's diaper more often. B. Encourage the mother to apply lotion with each diaper charge C. Tell the mother to cleanse with soap and water at each diaper change D. Ask the mother to decrease the infant's intake of fruits for 24 hours.

A. Instruct the mother to change the child's diaper more often.

A child newly diagnosed with sickle cell anemia (SCA) is being discharged from the hospital. Which information is most important for the nurse to provide the parents prior to discharge? A. Instructions about how much fluid the child should drink daily B. Information about non-pharmaceutical pain reliever measures C. Referral for social services for the child and family D. Signs of addiction to opioid and medications

A. Instructions about how much fluid the child should drink daily Rationale: It is essential that the child and family understands the importance of adequate hydration in preventing the stasis-thrombosis-ischemia cycle of a crisis that has a specific plan for hydration is developed so that a crisis can be delayed. Other choices listed are not the most important topics to include in the discharge teaching.

Which medication should the nurse anticipate administering to a client who is diagnosed with myxedema coma? A. Intravenous administration of thyroid hormones B. Oral administration of hypnotic agents C. Intravenous bolus of hydrocortisone D. Subcutaneous administration of vitamin k

A. Intravenous administration of thyroid hormones Rationale: The high mortality of myxedema coma requires immediate administration of IV thyroid hormones (A). (B) Is contraindicated, because eves small doses can cause profound somnolence lasting longer than expected. (C) Is administered to clients diagnosed with adrenal insufficiency (Addisonian crisis) and (D) to clients who have had an overdose of warfarin.

The health care provider prescribes atenolol 50 mg daily for a client with angina pectoris...to the health care provider before administering this medication? A. Irregular pulse B. Tachycardia C. Chest pain D. Urinary frequency

A. Irregular pulse

The nurse is teaching a client about the antiulcer medications ranitidine which was... statement best describes the action of this drug? A. It blocks the effects of histamine, causing decreased secretion of acid B. Ranitidine will neutralize gastric acid and decrease gastric pH C. This drug provides a protective coating over the gastric mucosa D. It effectively blocks 97% of the gastric acid secreted in the stomach

A. It blocks the effects of histamine, causing decreased secretion of acid

The nurse is caring for a client who is taking a macrolide to treat a bacterial infection. Which finding should the nurse report to the healthcare provider before administering the next dose? A. Jaundice B. Nausea C. Fever D. Fatigue

A. Jaundice

***An adult male client is admitted to the emergency room following an automobile collision in which he sustained a head injury. What assessment data would provide the earliest that the client is experiencing increased intracranial pressure (ICP)? A. Lethargy B. Decorticate posturing C. Fixed dilated pupil D. Clear drainage from the ear.

A. Lethargy Rationale: Lethargy is the earliest sign of ICP along with slowing of speech and response to verbal commands. The most important indicator of increase ICP is the client's level or responsiveness or consciousness. B and C are very late signs of ICP.

***A client is receiving lactulose (Portalac) for signs of hepatic encephalopathy. To evaluate the client's therapeutic response to this medication, which assessment should the nurse obtain? A. Level of consciousness B. Percussion of abdomen C. Serum electrolytes D. Blood glucose.

A. Level of consciousness Rationale: Colonic bacteria digest lactulose to create a drug-induces acidic and hyperosmotic environment that draws water and blood ammonia into the colon and coverts ammonia to ammonium, which is trapped in the intestines and cannot be reabsorbed into the systemic circulation. This therapeutic action of lactulose is to reduce serum ammonia levels, which improves the client's level of consciousness and metal status.

The nurse is assessing a middle-aged adult who is diagnosed with osteoarthritis. Which factor in this client's history is a contributor to the osteoarthritis? A. Long distance runner since high school. B. Lactose intolerant since childhood C. Photosensitive to a drug currently taking D. Recently treated for deep vein thrombosis

A. Long distance runner since high school.

A 59-year-old male client comes to the clinic and reports his concern over a lump that, "just popped up on my neck about a week ago." In performing an examination of the lump, the nurse palpates a large, nontender, hardened left subclavian lymph node. There is not overlying tissue inflammation. What do these findings suggest? A. Malignancy B. Bacterial infection C. Viral infection D. Lymphangitis

A. Malignancy

A client delivers a viable infant, but begins to have excessive uncontrolled vaginal...notifying the health care provider of the clients' condition, what information is most.... A. Maternal blood pressure B. Maternal apical pulse rate C. Time Pitocin infusion completed D. Total amount of Pitocin infused

A. Maternal blood pressure

A female client has been taking a high dose of prednisone, a corticosteroid, for several months. After stopping the medication abruptly, the client reports feeling "very tired". Which nursing intervention is most important for the nurse to implement? A. Measure vital signs B. Auscultate breath sounds C. Palpate the abdomen D. Observe the skin for bruising

A. Measure vital signs

***A client with cirrhosis of the liver is admitted with complications related to end stage liver disease. Which intervention should the nurse implement? (Select all that apply.) A. Monitor abdominal girth. B. Increase oral fluid intake to 1500 ml daily. C. Report serum albumin and globulin levels. D. Provide diet low in phosphorous. E. Note signs of swelling and edema.

A. Monitor abdominal girth. C. Report serum albumin and globulin levels. E. Note signs of swelling and edema. Rational: monitoring for increasing abdominal girth and generalized tissue edema and swelling are focused assessments that provide data about the progression of disease related complications. In advanced cirrhosis, liver function failure results in low serum albumin and serum protein levels, which caused third spacing that results in generalized fluid retention and ascites. Other options are not indicated in end stage liver disease.

The nurse is preparing a client for discharge from the hospital following a liver transplant. Which instruction is most important for the nurse to include in this client's discharge teaching plan? A. Monitor for an elevated temperature B. Measure the abdominal girth daily C. Report the onset of sclera jaundice D. Keep a record of daily urinary output

A. Monitor for an elevated temperature Rationale: The client should be instructed to monitor or elevated temperature because immunosuppressant agents, which are prescribed to reduce rejection after transplantation, place the client at risk for infection. The client should recognize sign of liver rejection, such as sclera jaundice and increasing abdominal girths, but fever may be the only sign of infection. A is not as important and monitoring for signs of infection.

A client with acute pancreatitis is complaining of pain and nausea. Which interventions should the nurse implement (Select all that apply) A. Monitor heart, lung, and kidney function. B. Notify healthcare provider of serum amylase and lipase levels. C. Review client's abdominal ultrasound findings. D. Position client on abdomen to provide organ stability E. Encourage an increased intake of clear oral fluids

A. Monitor heart, lung, and kidney function. B. Notify healthcare provider of serum amylase and lipase levels. C. Review client's abdominal ultrasound findings.

A client with acute renal failure (ARF) is admitted for uncontrolled type 1 diabetes Mellitus and hyperkalemia. The nurse administers an IV dose of regular insulin per sliding scale. Which intervention is the most important for the nurse to include in this client's plan of care? A. Monitor the client's cardiac activity via telemetry. B. Maintain venous access with an infusion of normal saline. C. Assess glucose via fingerstick q4 to 6 hours. D. Evaluate hourly urine output for return of normal renal function.

A. Monitor the client's cardiac activity via telemetry. Rational: as insulin lowers the blood glucose of a client with diabetic ketoacidosis (DKA), potassium returns to the cell but may not impact hyperkalemia related to acute renal failure. The priority is to monitor the client for cardiac dysrhythmias related to abnormal serum potassium levels. IV access, assessment of glucose level, and monitoring urine output are important interventions, but do not have the priority of monitoring cardiac function.

A male client is returned to the surgical unit following a left nephrectomy and is medicated with morphine. His dressing has a small amount of bloody drainage, and a Jackson-Pratt bulb surgical drainage device is in place. Which interventions is most important for the nurse to include in this clients plan of care? A. Monitor urine output hourly. B. Assess for back muscle aches C. Record drainage from drain D. Obtain body weight daily

A. Monitor urine output hourly.

An older female who ambulate with a quad-cane prefer to use a wheel chair because she has a halting and unsteady gait at times. Which interventions should the nurse implement? (Select all that apply) A. Move personal items within client's reach B. Lower bed to the lower possible position C. Give directions to call for assistance D. Assist client to the bathroom in 2 hours. E. Encourage the use of the wheelchair F. Raise all bed rails when the client is resting

A. Move personal items within client's reach B. Lower bed to the lower possible position C. Give directions to call for assistance D. Assist client to the bathroom in 2 hours. Rationale: A client who needs assistive devices, such as quad-cane is at risk for falls. Precautions that should implement include ensuring that personal items are within reach the bed is in the lowest position and directions are given to call assistance to minimize the risk for falls. Frequently assisting the client to the bathroom help ensure this client does not go the bathroom by herself, thereby decreasing the possibility of falling.

The nurse teaches an adolescent male client how to use a metered dose inhaler. Seen in the picture. What instruction should the nurse provide? A. Move the device one to two inches away from the mouth B. Secure the mouthpiece under the tongue C. Press down on the device after breathing in fully D. Breathe out slowly and deeply while compressing the device

A. Move the device one to two inches away from the mouth

An older client is admitted to the intensive care unit with severe abdominal pain, abdominal distention, and absent bowel sound. The client has a history of smoking 2 packs of cigarettes daily for 50 years and is currently restless and confused. Vital signs are: temperature 96`F, heart rate 122 beats/minute, respiratory rate 36 breaths/minute, mean arterial pressure(MAP) 64 mmHg and central venous pressure (CVP) 7 mmHg. Serum laboratory findings include: hemoglobin 6.5 grams/dl, platelets 6o, 000, and white blood cell count (WBC) 3,000/mm3. Based on these findings this client is at greatest risk for which pathophysiological condition? A. Multiple organ dysfunction syndrome (MODS) B. Disseminated intravascular coagulation (DIC) C. Chronic obstructive disease. D. Acquired immunodeficiency syndrome (AIDS)

A. Multiple organ dysfunction syndrome (MODS) Rational: MODS are a progressive dysfunction of two or more major organs that requires medical intervention to maintain homeostasis. This client has evidence of several organ systems that require intervention, such as blood pressure, hemoglobin, WBC, and respiratory rate. DIC may develop as a result of MODS. The other options are not correct.

The nurse is auscultating a client's heart sounds. Which description should the nurse use to document this sound? (Please listen to the audio file to select the option that applies.) A. Murmur B. s1 s2 C. pericardial friction rub D. s1 s2 s3

A. Murmur

A client with urticaria due to an environmental allergies is taking diphenhydramine... Which complaint should the nurse identify to the client as a side effect of the OTC medication? A. Nausea and indigestion. B. Hypersalivation C. Eyelid and facial twitching D. Increased appetite

A. Nausea and indigestion.

An adult client with schizophrenia begin treatment three days ago with the Antipsychotic risperidone. The client also received prescription for trazodone as needed for sleep and clonazepam as needed for severe anxiety. When the client reports difficulty with swallowing, what action should the nurse take? A. Obtain a prescription for an anticholinergic medication B. Determine how many hours declined slept last night C. Administer the PRN prescription for severe anxiety D. Watch the thyroid cartilage move while the client swallows

A. Obtain a prescription for an anticholinergic medication Rationale: Antipsychotic medications have an extrapyramidal side effects one of which is difficult to swallowing the nurse should obtain a prescription for an anticholinergic medication which is used for the treatment of extrapyramidal symptoms. Other options are not warranted actions based on the symptoms presented.

***A young adult female college student visits the health clinic in early winter to obtain birth control pills. The clinic nurse asks if the student has received an influenza vaccination. The student stated she did not receive vaccination because she has asthma. How should the nurse respond? A. Offer to provide the influenza vaccination to the student while she is at the clinic B. Encourage the student to obtain a vaccination prior to the next influenza season. C. Confirm that a history of asthma can increase risks associated with the vaccine. D. Advise the student that the nasal spray vaccine reduces side effects for people with asthma.

A. Offer to provide the influenza vaccination to the student while she is at the clinic Rationale: person with asthma are at increased risk related to influenza and should receive the influenza vaccination prior to or during influenza season. Waiting until the start of the next season places the student at risk for the current season. The vaccination does not increase risk for persons with asthma but the nasal spray may result in increased wheezing after receiving that form of the vaccination.

A client on a long-term mental health unit repeatedly takes own pulse regardless of the circumstance. What action should the nurse implement? A. Overlook the client's behavior. B. Distract client to interfere with the ritual. C. Ask why the client checks the pulse. D. Hold client's hand to stop the behavior.

A. Overlook the client's behavior.

To obtain an estimate of a client's systolic B/P. What action should the nurse take first? A. Palpate the client's brachial pulse B. Pump up the blood pressure cuff C. Position the stethoscope diaphragm D. Release the blood pressure cuff valve

A. Palpate the client's brachial pulse

***A client with multiple sclerosis is receiving beta-1b interferon every other day. To assess for possible bone marrow suppression caused by the medication, which serum laboratory test findings should the nurse monitor? (Select all that apply) A. Platelet count B. Red blood cell count (RBC) C. White blood cell count (WBC). D. Albumin and protein E. Sodium and potassium

A. Platelet count B. Red blood cell count (RBC) C. White blood cell count (WBC).

The nurse is caring for a one week old infant who has a ventriculoperitoneal (VP) shunt that was placed 2 days after birth. Which findings are an indication of a postoperative complication? A. Poor feeding and vomiting B. Leakage of CSF from the incisional site C. Hyperactive bowel sound D. Abdominal distention E. WBC count of 10000/mm3

A. Poor feeding and vomiting B. Leakage of CSF from the incisional site D. Abdominal distention

Following an esophagogastroduodenoscopy (EGD) a male client is drowsy and difficult to arouse, and his respiration are slow and shallow. Which action should the nurse implement? Select all that apply. A. Prepare medication reversal agent B. Check oxygen saturation level C. Apply oxygen via nasal cannula D. Initiate bag- valve mask ventilation. E. Begin cardiopulmonary resuscitation

A. Prepare medication reversal agent B. Check oxygen saturation level C. Apply oxygen via nasal cannula Rationale: Sedation, given during the procedure may need to be reverse if the client does not easily wake up. Oxygen saturation level should be asses, and oxygen applied to support respiratory effort and oxygenation. The client is still breathing so the bag- valve mask ventilation and CPR are not necessary.

A client with coronary artery disease who is experiencing syncopal episodes is admitted for an electrophysiology study (EPS) and possible cardiac ablation therapy. Which intervention should the nurse delegate to the unlicensed assistive personnel (UAP)? A. Prepare the skin for procedure. B. Identify client's pulse points C. Witness consent for procedure D. Check telemetry monitoring

A. Prepare the skin for procedure.

The nurse is teaching a group of clients with rheumatoid arthritis about the need to modify daily activities. Which goal should the nurse emphasize? A. Protect joint function B. Improve circulation C. Control tremors D. Increase weight bearing

A. Protect joint function

A 56-years-old man shares with the nurse that he is having difficulty making decision about terminating life support for his wife. What is the best initial action by the nurse? A. Provide an opportunity for him to clarify his values related to the decision B. Encourage him to share memories about his life with his wife and family C. Advise him to seek several opinions before making decision D. Offer to contact the hospital chaplain or social worker to offer support.

A. Provide an opportunity for him to clarify his values related to the decision Rationale: When a client is faced with a decisional conflict, the nurse should first provide opportunities for the client to clarify values important in the decision. The rest may also be beneficial once the client as clarified the values that are important to him in the decision-making process.

While assessing a client's chest tube (CT), the nurse discovers bubbling in the water seal chamber of the chest tube collection device. The client's vital signs are: blood pressure of 80/40 mmHg, heart rate 120 beats/minutes, respiratory rate 32 breaths/minutes, oxygen saturation 88%. Which interventions should the nurse implement? A. Provide supplemental oxygen B. Auscultate bilateral lung fields C. Administer a nebulizer treatment D. Reinforce occlusive CT dressing E. Give PRN dose of pain medication

A. Provide supplemental oxygen B. Auscultate bilateral lung fields D. Reinforce occlusive CT dressing Rationale: the air bubbles indicate an air leak from the lungs, the chest tube site, or the chest tube collection system. Providing oxygen improves the oxygen saturation until the leak has been resolved. Auscultating the lung fields helps to identify absent or decrease lung sound due to collapsing lung.

A low-risk primigravida at 28-weeks gestation arrives for her regular antepartal clinic visit. Which assessment finding should the nurse consider within normal limits for this client? A. Pulse increase of 10 beats/minute B. Proteinuria C. Glucosuria D. Fundal height 0f 22 centimeters

A. Pulse increase of 10 beats/minute

The nurse is assessing a client with a small bowel obstruction who was hospitalized 24 hours ago. Which assessment finding should the nurse report immediately to the healthcare provider? A. Rebound tenderness in the upper quadrants B. Hypoactive bowel sounds in the lower quadrants C. Tympany with percussion of the abdomen D. Light colored gastric aspirate via the nasogastric tube

A. Rebound tenderness in the upper quadrants

A client who is newly diagnosed with type 2 diabetes mellitus (DM) receives a prescription for metformin (Glucophage) 500 mg PO twice daily. What information should the nurse include in this client's teaching plan? (Select all that apply.)

A. Recognize signs and symptoms of hypoglycemia. B. Report persist polyuria to the healthcare provider. C. Take Glucophage with the morning and evening meal.

A client with diabetic peripheral neuropathy has been taking pregabalin (Lyrica) for 4 days. Which finding indicates to the nurse that the medication is effective? A. Reduced level of pain B. Full volume of pedal pulses C. Granulating tissue in foot ulcer D. Improved visual acuity

A. Reduced level of pain

A gravida 2 para 1, at 38-weeks gestation, scheduled for a repeat cesarean section in one week, is brought to the labor and delivery unit complaining of contractions every 10 minutes. While assessing the client, the client's mothers enter the labor suite and says in a loud voice, "I've had 8 children and I know she's in labor. I want her to have her cesarean section right now!" what action should the nurse take? A. Request the mother to leave the room B. Tell the mother to stop speaking for the client C. Request security to remove her from the room D. Notify the charge nurse of the situation

A. Request the mother to leave the room

***A client who had a below the knee amputation is experiencing severe phantom limb pain (PLP) and ask the nurse if mirror therapy will make the pain stop. Which response by the nurse is likely to be most helpful? A. Research indicates that mirror therapy is effective in reducing phantom limb pain B. You can try mirror therapy, but do not expect to complete elimination of the pain C. Transcutaneous electrical nerve stimulators (TENS) have been found to be more effective D. Where did you learn about the use of mirror therapy in treating in treating phantom limb pain?

A. Research indicates that mirror therapy is effective in reducing phantom limb pain Rationale: pain relief associated with mirror therapy may be due to the activation of neurons in the hemisphere of the brain that is contralateral to the amputated limb when visual input reduces the activity of systems that perceive protopathic pain.

The nurse is assessing a 3-month-old infant who had a pylorotomy yesterday. This child should be medicated for pain based on which findings? Select all that apply: A. Restlessness B. Clenched Fist C. Increased pulse rate D. Increased respiratory rate. E. Increased temperature F. Peripheral pallor of the skin

A. Restlessness B. Clenched Fist C. Increased pulse rate D. Increased respiratory rate.

The healthcare provider prescribes a low-fiber diet for a client with ulcerative colitis. Which food selection would indicate to the nurse the client understands they prescribed diet? A. Roasted turkey canned vegetables B. Baked potatoes with skin raw carrots C. Pancakes whole-grain cereal's D. Roast pork fresh strawberries

A. Roasted turkey canned vegetables Rationale: Foods allowed on a low-fiber diet includes roasted or baked turkey and canned vegetables the foods in the other options are not low in fiber

During the infusion of a second unit of packed red blood cells, the client's temperature increases from 99 to 101.6 f. which intervention should the nurse implement? A. Stop the transfusion start a saline B. Observe for a maculopapular rash C. Report the fever to the blood bank D. Give a PRN dose of acetaminophen

A. Stop the transfusion start a saline

A male client reports to the clinic nurse that he has been feeling well and is often "dizzy" his blood pressure is elevated. Based on this findings, this client is at a greatest risk for which pathophysiological condition? A. Stroke B. Renal failure C. Left ventricular hypertrophy D. Pulmonary hypertension

A. Stroke

A male client is discharged from the intensive care unit following a myocardial infarction, and the healthcare provider low-sodium diet. Which lunch selection indicates to the nurse that this client understands the dietary restrictions? A. Turkey salad sandwich. B. Clam chowder C. Macaroni and cheese D. Bacon, lettuce, and tomato sandwich

A. Turkey salad sandwich.

The nurse weighs a 6-month-old infant during a well-baby check-up and determines that the baby's weight has tripled compared to the birth weight of 7 pounds 8 ounces. The mother asks if the baby is gaining enough weight. What response should the nurse offer? A. What food does your baby usually eat in a normal day? B. What was the baby's weight at the last well-baby clinic visit? C. The baby is below the normal percentile for weight gain D. Your baby is gaining weight right on schedule

A. What food does your baby usually eat in a normal day?

A male client, who is 24 hours postoperative for an exploratory laparotomy, complains that he is "starving" because he has had no "real food" since before the surgery. Prior to advancing his diet, which intervention should the nurse implement?

Auscultate bowel sounds in all four quadrants

A client who had a small bowel resection acquired methicillin resistant staphylococcus aureus (MRSA) while hospitalized. He treated and released, but is readmitted today because of diarrhea and dehydration. It is most important for the nurse to implement which intervention.

Maintain contact transmission precaution

When assessing a multigravida the first postpartum day, the nurse finds a moderate amount of lochia rubra, with the uterus firm, and three fingerbreadths above the umbilicus. What action should the nurse implement first? A. Massage the uterus to decrease atony B. Review the hemoglobin to determine hemorrhage C. Increase intravenous infusion D. Check for a distended bladder

D. Check for a distended bladder

The nurse is conducting health assessments. Which assessment finding increases a 56-year-old woman's risk for developing osteoporosis? A. Body mass index of (BMI) of 31 B. 20 pack-year history of cigarette smoking C. Birth control pill usage until age 45 D. Diabetes mellitus in family history

B. 20 pack-year history of cigarette smoking Rationale: Cigarette smoking (2 packs/day x 310 years = 20 packs-year) increases the risk of osteoporosis. BMI of 30 or greater falls in the category of obesity which increase weight bearing that is protective against osteoporosis. C contain estrogens, and are also protective against development of osteoporosis. D is not related to the development of osteoporosis.

The nurse is caring for four clients who are on the rehabilitation unit, which client should the nurse assess first? A. A client with an above-the-knee amputation who is complaining of phantorn pain. B. A client who is receiving a continuous tube feeding and is now vomiting. C. A client with left hemiplegia who is scheduled for hemodialysis today. D. A client with pneumonia who is scheduled for pulmonary function studies.

B. A client who is receiving a continuous tube feeding and is now vomiting.

A client is scheduled to receive an IW dose of ondansetron (Zofran) eight hours after receiving chemotherapy. The client has saline lock and is sleeping quietly without any restlessness. The nurse caring for the client is not certified in chemotherapy administration. What action should the nurse take? A. Ask a chemotherapy-certified nurse to administer the Zofran B. Administer the Zofran after flushing the saline lock with saline C. Hold the scheduled dose of Zofran until the client awakens D. Awaken the client to assess the need for administration of the Zofran.

B. Administer the Zofran after flushing the saline lock with saline Rationale: Zofran is an antiemetic administered before and after chemotherapy to prevent vomiting. The nurse should administer the antiemetic using the accepter technique for IV administration via saline lock. Zofran is not a chemotherapy drug and does not need to be administered by a chemotherapy- certified nurse.

The nurse is triaging several children as they present to the emergency room after an accident. Which child requires the most immediate intervention by the nurse? A. A 12-year-old with complaints of neck and lower back discomfort B. An 11-year-old with a headache, nausea, and projectile vomiting C. A 6-year-old with multiple superficial lacerations of all ectremities D. An 8-year-old with a full leg air splint for a possible broken tibia

B. An 11-year-old with a headache, nausea, and projectile vomiting

A male client with ulcerative colitis received a prescription for a corticosteroid last month, but because of the side effect he stopped taking the medication 6 year ago. Which finding warrants immediate intervention by the nurse? A. Hypotension and fever B. Anxiety and restlessness. C. Fluid retention D. Increased blood glucose.

B. Anxiety and restlessness.

The nurse reviews the laboratory findings of a client with an open fracture of the tibia. The white blood cell (WBC) count and erythrocyte sedimentation rate (ESR) are elevated. Before reporting this information to the healthcare provider, what assessment should the nurse obtain? A. Degree of skin elasticity B. Appearance of wound C. Bilateral pedal pulse force D. Onset of any bleeding

B. Appearance of wound

A multigravida, full-term, laboring client complains of "back labor". Vaginal examination reveals that the client's 3 cm with 50% effacement and the fetal head is at -1 station. What should the nurse implement? A. Turn the client to a lateral position B. Apply counter-pressure to the sacral area C. Notify the scrub nurse to prepare the OR D. Ambulate the client between contractions

B. Apply counter-pressure to the sacral area

A new member joins the nursing team spreads books on the table, puts items on two chairs, and sits on a third chair. The members of the group are forced to move closer and remove their possessions from the table what action should the nurse leader take? A. Move to welcome and accommodate a new person B. Ask the new person to move belonging to accommodate others C. Tell the new person to move belongings because of limited space D. Bring in additional chairs so that all staff members can be seated

B. Ask the new person to move belonging to accommodate others

While receiving a male postoperative client's staples de nurse observe that the client's eyes are closed and his face and hands are clenched. The client states, "I just hate having staples removed". After acknowledgement the client's anxiety, what action should the nurse implement? A. Encourage the client to continue verbalize his anxiety B. Attempt to distract the client with general conversation C. Explain the procedure in detail while removing the staples D. Reassure the client that this is a simple nursing procedure.

B. Attempt to distract the client with general conversation Rational: Distract is an effective strategy when a client experience anxiety during an uncomfortable procedure. (A & D) increase the client's anxiety.

Oxygen at 5l/min per nasal cannula is being administered to a 10 year old child with pneumonia. When planning care for this child, what principle of oxygen administration should the nurse consider? A. Taking a sedative at bedtime slows respiratory rate, which decreases oxygen? B. Avoid administration of oxygen at high levels for extendedperiods. C. Increase oxygen rate during sleep to compensate for slower respiratory rate. D. Oxygen is less toxic when it is humidified with a hydration source.

B. Avoid administration of oxygen at high levels for extendedperiods.

The nurse is preparing an older client for discharge following cataract extraction. Which instruction should be include in the discharge teaching? A. Do not read without direct lighting for 6 weeks. B. Avoid straining at stool, bending, or lifting heavy objects. C. Irrigate conjunctiva with ophthalmic saline prior to installing antibiotic ointment. D. Limit exposure to sunlight during the first 2 weeks when the cornea is healing.

B. Avoid straining at stool, bending, or lifting heavy objects. Rationale: after cataract surgery, the client should avoid activities which increase pressure and place strain on the suture line.

When providing diet teaching for a client with cholecystitis, which types of food choices the nurse recommend to the client? A. High protein B. Low fat C. Low sodium D. High carbohydrate.

B. Low fat Rationale: A client with cholecystitis is at risk of gall stones that can be move into the biliary tract and cause pain or obstruction. Reducing dietary fat decrease stimulation of the gall bladder, so bile can be expelled, along with possible stones, into the biliary tract and small intestine.

The nurse assesses a female client with obstructive sleep apnea syndrome (OSAS) who is 5 feet tall (152 cm) and weighs 155 pounds (70 kg), the client's 24 hour diet history includes: no breakfast, cheeseburger and fries for lunch; lasagna, chocolate ice cream and a cola drink for dinner, and 2 glasses of wine in the evening before going to bed for a total caloric intake of 3500 calories. What instructions should the nurse provide? (Select all that apply) A. Maintain current caloric intake B. Avoid use of alcohol as a sleep aide at bedtime C. Reduce intake of dairy products D. Start a weight loss program E. Set a goal of increasing BMI (Body Mass Index)

B. Avoid use of alcohol as a sleep aide at bedtime D. Start a weight loss program

When assessing a multigravida the first postpartum day, the nurse finds a moderate amount of lochia rubra, with the uterus firm, and three fingerbreadths above the umbilicus. What action should the nurse implement first? A. Massage the uterus to decrease atony B. Check for a destined bladder C. Increase intravenous infusion D. Review the hemoglobin to determined hemorrhage

B. Check for a destined bladder Rationale: a fundus that is dextroverted (up to the right) and elevated above the umbilicus is indicative of bladder distension/urine retention.

A client who is at 10-weeks gestation calls the clinic because she has been vomiting for the past 24 hours. The nurse determines that the client has no fever. Which instructions should the nurse give to this client? A. Remain on clear liquids until the vomiting subsides B. Come to the clinic to be seen by a healthcare provider C. Make an appointment at the clinic if a fever occurs D. Take nothing by mouth until there is no more nausea

B. Come to the clinic to be seen by a healthcare provider

The nurse is changing a client's IV tubing and closes the roller clamp on the new tubing setup when the bag of solution is....which action should the nurse take to ensure adequate filling of the drip chamber? A. Lower the IV bag to a flat surface B. Compress the drip chamber C. Open the roller clamp D. Squeeze the bag of IV solution

B. Compress the drip chamber

A client with Alzheimer's disease (AD) is receiving trazodone (Desyrel), a recently prescribed atypical antidepressant. The caregiver tells the home health nurse that the client's mood and sleep patterns are improved, but there is no change in cognitive ability. How should the nurse respond to this information? A. Explain that it may take several weeks for the medication to be effective B. Confirm the desired effect of the medication has been achieved. C. Notify the health care provider than a change may be needed. D. Evaluate when and how the medication is being administered to the client.

B. Confirm the desired effect of the medication has been achieved. Rationale: Trazodone o Desyrel, an atypical antidepressant, is prescribed for client with AD to improve mood and sleep.

The nurse is developing an educational program for older clients who are being discharged with new antihypertensive medications. The nurse should ensure that the educational materials include which characteristics? Select all that apply A. Written at a twelfth grade reading level B. Contains a list with definitions of unfamiliar terms C. Uses common words with few Syllables D. Printed using a 12 point type font E. Uses pictures to help illustrate complex ideas

B. Contains a list with definitions of unfamiliar terms C. Uses common words with few Syllables E. Uses pictures to help illustrate complex ideas Rationale: During the aging process older clients often experience sensory or cognitive changes, such as decreased visual or hearing acuity, slower thought or reasoning processes, and shorter attention span. Materials for this age group should include a list of terms, such as a medical terminology that the client may not know and use common words that expresses information clearly and simply. Simple, attractive pictures help hold the learner's attention. The reading level of material should be at the 4th to 5th grade level. Materials should be printed using large font (18-point or higher), not the standard 12-point font.

A client with osteoporosis related to long-term corticosteroid therapy receives a prescription for calcium carbonate. Which client's serum laboratory values requires intervention by the nurse? A. Total calcium 9 mg/dl (2.25 mmol/L SI) B. Creatinine 4 mg/dl (354 micromol/L SI) C. Phosphate 4 mg/dl (1.293 mmol/L SI) D. Fasting glucose 95 mg/dl (5.3 mmol/L SI)

B. Creatinine 4 mg/dl (354 micromol/L SI)

A mother runs into the emergency department with s toddler in her arms and tells the nurse that her child got into some cleaning products. The child smells of chemicals on hands, face, and on the front of the child's clothes. After ensuring the airway is patent, what action should the nurse implement first? A. Call poison control emergency number. B. Determine type of chemical exposure. C. Obtain equipment for gastric lavage. D. Assess child for altered sensorium.

B. Determine type of chemical exposure. Rational: once the type of chemical is determined, poison control should be called even if the chemical is unknown. If lavage is recommended by poison control, intubation and nasogastric tube may be needed as directed by poison control. Altered sensorium, such as lethargy, may occur if hydrocarbons are ingested

An adult client is exhibit the maniac stage of bipolar disorder is admitted to the psychiatric unit. The client has lost 10 pounds in the last two weeks and has no bathed in a week "I'm trying to start a new business and "I'm too busy to eat". The client is oriented to time, place, person but not situation. Which nursing problem has the greatest priority? A. Hygiene-self-care deficit B. Imbalance nutrition C. Disturbed sleep pattern D. Self-neglect

B. Imbalance nutrition Rationale: The client's nutritional status has the highest priority at this time, and finger foods are often provided, so the client who is on the maniac phase of bipolar disease can receive adequate nutrition. Other options are nursing problems that should also be addresses with the client's plan of care, but at this stage in the client's treatment, adequate nutrition is a priority

Assessment by the home health nurse of an older client who lives alone indicates that client has chronic constipations. Daily medications include furosemide for hypertension and heart failure and laxatives. To manage the client's constipation, which suggestions should the nurse provide? (Select all that apply) A. Decrease laxative use to every other day, and use oil retention enemas as needed. B. Include oatmeal with stewed pruned for breakfast as often as possible. C. Increase fluid intake by keeping water glass next to recliner. D. Recommend seeking help with regular shopping and meal preparation. E. Report constipation to healthcare provider related to cardiac medication side effects.

B. Include oatmeal with stewed pruned for breakfast as often as possible. C. Increase fluid intake by keeping water glass next to recliner. D. Recommend seeking help with regular shopping and meal preparation. Rational: older adult are at higher risk for chronic constipation due to decreased gastrointestinal muscle tone leading to reduce motility. Oatmeal with prunes increases dietary fiber and bowel stimulation, thereby decreasing need for laxatives. Increased fluid intake also decreases constipations. Assistance with food preparation might help the client eat more fresh fruits and vegetables and result on less reliance on microwaved and fast foods, which are usually high in sodium and fat with little fiber. Laxatives can be reduced gradually by improving the diet, without resorting to using enemas.

A client experiencing withdrawal from the benzodiazepines alprazolam (Xanax) is demonstrating severe agitation and tremors. What is the best initial nursing action? A. Administer naloxone (Narcan) per PNR protocol B. Initiate seizure precautions C. Obtain a serum drug screen D. Instruct the family about withdrawal symptoms.

B. Initiate seizure precautions Rationale: Withdrawal of CNS depressants, such as Xanax, results in rebound over-excitation of the CNS. Since the client exhibiting tremors, the nurse should anticipate seizure activity and protect the client.

A young adult female presents at the emergency center with acute lower abdominal pain. Which assessment finding is most important for the nurse to report to the healthcare provider? A. Pain scale rating at 9 on a 0-10 scale B. Last menstrual period was 7 weeks ago C. Reports white curdy vaginal discharge D. History of irritable bowel syndrome IBS

B. Last menstrual period was 7 weeks ago Rationale: Acute lower abdominal pain in A young adult female can be indicative of an ectopic pregnancy, which can be life threatening. Since the clients last menstrual period was seven weeks ago a pregnancy test to be obtained to ruled out ectopic pregnancy, which can result in intra-abdominal hemorrhage caused by a ruptured Fallopian tube. Although the severity of pain requires treatment, the most significant finding is the clients last menstrual period. Other options are not the most important concerns.

A client with Alzheimer's disease falls in the bathroom. The nurse notifies the charge nurse and completes a fall follow-up assessment. What assessment finding warrants immediate intervention by the nurse? A. Urinary incontinence B. Left forearm hematoma C. Disorientation to surroundings D. Dislodge intravenous site

B. Left forearm hematoma Rationale: The left forearm hematoma may be indicative an injury, such as broken bone, that requires immediate intervention. A may be likely be due to the inability to use the toilet due to the fall. Disorientation is a common symptom of Alzheimer's disease. IV Dislodged is not an urgent concern.

The daughter of an older female client tells the clinic nurse that she is no longer able to care for her mother since her mother has lost the ability to perform activities of daily living (ADLs) due to aging. Which options should the nurse discuss with the daughter? A. Home hospice agency B. Long-term care facility C. Rehabilitation facility D. Independent senior apartment E. Home health agency

B. Long-term care facility E. Home health agency

A nurse is planning discharge care for a male client with metastatic cancer. The client tells the nurse that he plans to return to work despite pain, fatigue, and impending death. Which goals is most important to include in this client's plan of care? A. Implements decisions about future hospices services within the next 3 months. B. Marinating pain level below 4 when implementing outpatient pain clinic strategies. C. Request home health care if independence become compromised for 5 days. D. Arranges for short term counseling stressors impact work schedule for 2 weeks.

B. Marinating pain level below 4 when implementing outpatient pain clinic strategies. Rationale: An outpatient pain clinic provides the interdisciplinary services needed to manage chronic pain. Also the client has a terminal disease and is being discharge home, hospice and health care are not indicating at this time. Short term counseling is not an option.

A client with a history of cirrhosis and alcoholism is admitted with severe dyspnea and ascites. Which assessment finding warrants immediate intervention by the nurse? A. Jaundice skin tone B. Muffled heart sounds C. Pitting peripheral edema D. Bilateral scleral edema

B. Muffled heart sounds Rationale: Muffled heart sounds may indicative fluid build-up in the pericardium and is life- threatening. The other one are signs of end stage liver disease related to alcoholism but are not immediately life- threatening.

The nurse is preparing a teaching plan for an older female client diagnosed with osteoporosis. What expected outcome has the highest priority for this client? A. Identifies 2 treatments for constipation due to immobility. B. Names 3 home safety hazards to be resolve immediately. C. State 4 risk factors for the development of osteoporosis. D. Lists 5 calcium-rich foods to be added to her daily diet.

B. Names 3 home safety hazards to be resolve immediately. Rational: a major teaching goal for an elderly client with osteoporosis is maintenance of safety to prevent falls. Injury due to a fall, usually resulting in a hip fracture, can result in reduced mobility and associated complications. Other goals are also important when teaching clients who have osteoporosis, but they do not have the priority of preventing falls, which relates to safety.

An adult male is brought to the emergency department by ambulance following a motorcycle accident. He was not wearing a helmet and presents with periorbital bruising and bloody drainage from both ears. Which assessment finding warrants immediate intervention by the nurse? A. Rebound abdominal tenderness B. Nausea and projectile vomit C. Rib pain with deep inspiration D. Diminished bilateral breath sounds

B. Nausea and projectile vomit Rationale: Projective vomiting is indicative of increasing intracranial pressure, which can lead to ischemic brain damage or death, so this finding warrants immediate intervention. Rebound abdominal tenderness may indicate internal bleeding. Diminished breath sound may be related to pain. Rib pain with inspiration may indicate rib fracture.

An older adult client with heart failure (HF) develops cardiac tamponade. The client has muffled, distant, heart sounds, and is anxious and restless. After initiating oxygen therapy and IV hydration, which intervention is most important for the nurse to implement? A. Observe neck for jugular vein distention B. Notify healthcare provider to prepare for pericardiocentesis C. Asses for paradoxical blood pressure D. Monitor oxygen saturation (Sp02) via continuous pulse oximetry

B. Notify healthcare provider to prepare for pericardiocentesis Rationale: Cardiac tamponade is pressure on the heart that occurs when blood or fluid builds up in the space between the heart muscle (myocardium) and the outer covering sac of the heart (pericardium). In this condition, blood or fluid collects in the pericardium, the sac surrounding the heart. This prevents the heart ventricles from expanding fully. The excess pressure from the fluid prevents the heart from working properly. As a result, the body does not get enough blood.

After an elderly female client receives treatment for drug toxicity, the HCP prescribes a 24- hour creatinine clearance test. Prior to starting the urine collection, the nurse notes that the client's serum creatinine is 0.3mg/dl. What action should the nurse implement? A. Initiate the urine collection as prescribed. B. Notify the HCP of the results. C. Evaluate the client's serum BUN level. D. Assess the client for signs of hypokalemia.

B. Notify the HCP of the results.

After the risk and benefits of having a cardiac catheterization are reviewed by the healthcare provider, an older adult with unstable angina is scheduled for the procedure. When the nurse presents the consent form for signature, the client asks how the wires will keep a heart heating during the procedure. What action should the nurse take? A. Explain the procedure again in detail and clarify any misconceptions. B. Notify the healthcare provider of the client's lack of understanding. C. Call the client's next of kin and have them provide verbal consent. D. Postpone the procedure until the client understands the risk and benefits.

B. Notify the healthcare provider of the client's lack of understanding. Rational: the nurse is only witnessing the signature, and is not responsible for the client's understanding of the procedure. The healthcare provider needs to clarify any questions and misconceptions. Explaining the procedure again is the healthcare provider's legal responsibility. The other options are not indicated.

The nurse is teaching a male adolescent recently diagnosed with type 1diabetes mellitus (DM) about self-injecting insulin. Which approach is best for the nurse to use to evaluate do you effectiveness of the teaching? A. Ask the adolescent to describe his level of comfort with injecting himself with insulin. B. Observe him as he demonstrates self-injection technique in another diabetic adolescent C. Have the adolescent list the procedural steps for safe insulin administration. D. Review his glycosylated hemoglobin level 3 months after the teaching session.

B. Observe him as he demonstrates self-injection technique in another diabetic adolescent Rational: watching the adolescent perform the procedure with another adolescent provides peer support the most information regarding his skill with self-injection. Other options do not provide information about the effectiveness of nurse's teaching.

The nurse is assessing a postpartum client who is 36 hours post-delivery. Which finding should the nurse report to the healthcare provider? A. White blood count of 19,000 mm3 B. Oral temperature of 100.6 F C. Fundus deviated to the right side D. Breasts are firm when palpated

B. Oral temperature of 100.6 F Rationale: A temperature greater than 100.4 F (38 C) (B), which is indicative of endometriosis (infection of the lining of the uterus), should be reported to the health care provider. (A and D) are findings that are within normal limits in the postpartum period. Fundal deviation to one side (C) is an expected finding related to a full bladder, so the nurse should encourage the client to void.

An older male client arrives at the clinic complaining that his bladder always feels full. He complains of weak urine flow, frequent dribbling after voiding, and increasing nocturia with difficulty initiating his urine stream. Which action should the nurse implement? A. Obtain a urine specimen for culture and sensitivity B. Palpate the client's suprapubic area for distention C. Advise the client to maintain a voiding diary for one week D. Instruct in effective technique to cleanse the glans penis

B. Palpate the client's suprapubic area for distention Rationale: the client is exhibiting classic signs of an enlarge prostate gland, which restricts urine flow and cause bothersome lower urinary tract symptoms (LUTS) and urinary retention, which is characterized by the client's voiding patterns and perception of incomplete bladder emptying.

The nurse has received funding to design a health promotion project for African-American women who are at risk for developing breast cancer. Which resource is most important in designing this program? A. A listing of African-American women so live in the community B. Participation of community leaders in planning the program C. Morbidity data for breast cancer in women of all races D. Technical assistance to produce a video on breast self-examination.

B. Participation of community leaders in planning the program Rationale: When developing a culturally-competent health promotion project, the participation of stakeholders and community leaders is most important. A and B might be useful background information, but t=first the program should be developed. D may be useful fulfilling the plan developed by the health care team and the community leaders if funding for this assistance is included in the budget.

In assessing a pressure ulcer on a client's hip, which action should the nurse include? A. Determine the degree of elasticity surrounding the lesion B. Photograph the lesion with a ruler placed next to the lesion C. Stage the depth of the ulcer using the Braden numeric scale D. Use a gloved finger to palpate for tunneling around the lesion

B. Photograph the lesion with a ruler placed next to the lesion Rationale: An ulcer extends into the dermis or subcutaneous tissue and is likely to increase in size and depth, so assessment should include photograph with measuring device to document the size of the lesion.

The charge nurse observes the practical nurse (PN) apply sterile gloves in preparation for performing a sterile dressing change. Which action by the PN requires correction by the charge nurse? A. Opening the package B. Picking up the second glove C. Picking up the first glove D. Positioning of the table

B. Picking up the second glove

A 17-year -old male is brought to the emergency department by his parents because he has been coughing and running a fever with flu-like symptoms for the past 24 hours. Which intervention should the nurse implement first? A. Obtain a chest X-ray per protocol. B. Place a mask on the client's face. C. Assess the client's temperature. D. Determine the client's blood pressure

B. Place a mask on the client's face.

During discharge teaching, an overweight client heart failure (HF) is asked to make a grocery list for the nurse to review. Which food choices included on the client's list should the nurse encourage? (Select all that apply) A. Canned fruit in heavy syrup. B. Plain, air-popped popcorn. C. Cheddar cheese cubes. D. Natural whole almonds. E. Lightly salted potato chips

B. Plain, air-popped popcorn. D. Natural whole almonds.

A client who underwent an uncomplicated gastric bypass surgery is having difficult with diet management. What dietary instruction is most important for the nurse to explain to the client? A. Chew food slowly and thoroughly before attempting to swallow B. Plan volume-controlled evenly-space meal thorough the day C. Sip fluid slowly with each meal and between meals D. Eliminate or reduce intake fatty and gas forming food

B. Plan volume-controlled evenly-space meal thorough the day Rationale: It is most important for the client to learn how to eat without damaging the surgical site and to keep the digestive system from dumping the food instead of digesting it. Eating volume-control and evenly-space meals thorough the day allows the client to fill full, avoid binging, and eliminate the possibility of eating too much one time. Chewing slowly and thoroughly helps prevent over eating by allowing a filling of fullness to occur. Taking sips, rather than large amounts of fluids keeps the stomach from overfilling and allow for adequate calories to be consumed. Gas forming foods and fatty foods should be avoiding to decrease risk of dumping syndrome and flatulence.

An older adult male who had an abdominal cholecystectomy has become increasingly confused and disoriented over the past 24 hours. He is found wandering into another client's room and is return to his room by the unlicensed assistive personnel (UAP). What actions should the nurse take? (Select all that apply). A. Apply soft upper limb restrains and raise all four bed rails B. Report mental status change to the healthcare provider C. Assess the client's breath sounds and oxygen saturation D. Assign the UAP to re-assess the client's risk for falls E. Review the client's most recent serum electrolyte values

B. Report mental status change to the healthcare provider C. Assess the client's breath sounds and oxygen saturation E. Review the client's most recent serum electrolyte values Rationale: The healthcare provider should be informed of changes in the client's condition (B) because this behavior may indicate a postoperative complication. Diminished oxygenation (C) and electrolyte imbalance (E) may cause increased confusion in the older adult. Raising all four bed rails (A) may lead to further injury if the client climbs over the rails and falls and restrains should not be applied until other measures such as re-orientation are implemented. The nurse should assess the client's increased risk for falls, rather than assigning this to the UAP (D).

An adult client present to the clinic with large draining ulcers on both lower legs that are characteristics of Kaposi's sarcoma lesions. The client is accompanied by two family member. Which action should the nurse take? A. Ask family member to wear gloves when touching the patient B. Send family to the waiting area while the client's history is taking C. Obtain a blood sample to determine is the client is HIV positive D. Complete the head to toes assessment to identify other sign of HIV

B. Send family to the waiting area while the client's history is taking Rationale: To protect the client privacy, the family member should be asked to wait outside while the client's history is take. Gloves should be worn when touching the client's body fluids if the client is HIV positive and these lesion are actually Kaposi sarcoma lesion. HIV testing cannot legally be done without the client explicit permission. A further assessment can be implemented after the family left the room.

While visiting a female client who has heart failure (HF) and osteoarthritis, the home health nurse determines that the client is having more difficulty getting in and out of the bed than she did previously. Which action should the nurse implement first? A. Inquire about an electric bed for the client's home use B. Submit a referral for an evaluation by a physical therapist. C. Explain the usual progression of osteoarthritis and HF D. Request social services to review the client's resources.

B. Submit a referral for an evaluation by a physical therapist.

The nurse is caring for a 17-year-old male who fell 20 feet 5 months ago while climbing the side of a cliff and has been in a sustained vegetative state since the accident. Which intervention should the nurse implement? A. Inquire about food allergies and food likes and dislikes B. Talk directly to the adolescent while providing care C. Initiate open communication with the teen's parents D. Monitor vital signs and neuro status every 2 hours

B. Talk directly to the adolescent while providing care

A client with polycystic kidney disease (PKD) receiving antibiotics for an infected cyst is experiencing severe pain. What action should the nurse implement? A. Hold the next dose of antibiotic until contacting the healthcare provider B. Teach the client how to use a dry heating pad over the painful area C. Encourage the client to practice pelvic floor exercises every hour D. Assist the client to splint the site by applying an abdominal binder

B. Teach the client how to use a dry heating pad over the painful area

What explanation is best for the nurse to provide a client who asks the purpose of using the log-rolling technique for turning? A. Working together can decrease the risk for back injury B. The technique is intended to maintain straight spinal alignment. C. Using two or three people increases client safety. D. Turning instead of pulling reduces the likelihood of skin damage

B. The technique is intended to maintain straight spinal alignment.

A female client with rheumatoid arthritis (RA) comes to the clinic complaining of joint pain and swelling. The client has been taking prednisone (Deltasone) and ibuprofen (Motrin Extra Strength) every day. To assist the client with self-management of her pain, which information should the nurse obtain? A. Presence of bruising, weakness, or fatigue B. Therapeutic exercise included in daily routine. C. Average amount of protein eaten daily D. Existence of gastrointestinal discomfort

B. Therapeutic exercise included in daily routine.

The nurse plans to collect a 24- hour urine specimen for a creatinine clearance test. Which instruction should the nurse provide to the adult male client? A. Clearance around the meatus, discard first portion of voiding, and collect the rest in a sterile bottle B. Urinate at specific time, discard the urine, and collect all subsequent urine during the next 24 hours. C. For the next 24 hours, notify the nurse when the bladder is full, and the nurse will collect catheterized specimens. D. Urinate immediately into a urinal, and the lab will collect specimen every 6 hours, for the next 24 hours.

B. Urinate at specific time, discard the urine, and collect all subsequent urine during the next 24 hours. Rationale: Urinate at specific time, discard the urine, and collect all subsequent urine during the next 24 hours is the correct procedure for collecting 24-hour urine specimen. Discarding even one voided specimen invalidate the test.

A male client is admitted with a severe asthma attack. For the last 3 hours he has experienced increased shortness of breath. His arterial blood gas results are: pH 7.22 PaCO2 55 mmHg; HCO3 25 mEq/L or mmol/L (SI). Which intervention should the nurse implement? A. Space care to provide periods of rest B. Instruct client to purse lip breathe C. Administer PRN dose of albuterol D. Position client for maximum comfort

C. Administer PRN dose of albuterol

***A female nurse who took drugs from the unit for personal use was temporarily released from duty. After completion of mandatory counseling, the nurse has asked administration to allow her to return to work. When the nurse administrator approaches the charge nurse with the impaired nurse request, which action is best for the charge nurse to take? A. Since treatment is completed, assign the nurse to the route RN responsibilities B. Ask to meet with impaired nurse's therapist before allowing her back on the unit. C. Allow the impaired nurse to return to work and monitor medication administration D. Meet with staff to assess their feelings about the impaired nurse's return to the unit.

C. Allow the impaired nurse to return to work and monitor medication administration Rationale: provides essential monitoring and helps ensure nurse compliance and promote client safety.

Based on principles of asepsis, the nurse should consider which circumstance to be sterile? A. One inch- border around the edge of the sterile field set up in the operating room B. A wrapped unopened, sterile 4x4 gauze placed on a damp table top. C. An open sterile Foley catheter kit set up on a table at the nurse waist level D. Sterile syringe is placed on sterile area as the nurse riches over the sterile field.

C. An open sterile Foley catheter kit set up on a table at the nurse waist level Rationale: A sterile package at or above the waist level is considered sterile. The edge of sterile field is contaminated which include a 1-inch border (A). A sterile objects become contaminated by capillary action when sterile objects become in contact with a wet contaminated surface.

When should intimate partner violence (IPV) screening occur? A. As soon as the clinician suspects a problem B. Only when a client presents with an unexplained injury C. As a routine part of each healthcare encounter D. Once the clinician confirms a history of abuse

C. As a routine part of each healthcare encounter Rationale: Universal screening for IPV is a vital means to identify victims of abuse in relationship. The suspicious of different clinicians vary greatly, so screening would not be implemented consistently. The client should be screened regardless of the presence of injury. Although history of abuse is difficult to confirm, screening should occur regardless, and this incident may know may be initial case of abuse.

When five family members arrive at the hospital, they all begin asking the nurse questions regarding the prognosis of their critically ill mother. What intervention should the nurse implement first? A. Include the family in client's care B. Request the chaplain's presence C. Ask the family to identify a specific spokesperson D. Page the healthcare provider to speak with family.

C. Ask the family to identify a specific spokesperson

A preoperative client states he is not allergic to any medications. What is the most important nursing action for the nurse to implement next? A. Record "no known drug allergies" on preoperative checklist B. Assess client's allergies to non-drug substances C. Assess client's knowledge of an allergy response D. Flag "no known drug allergies" on the front of the chart

C. Assess client's knowledge of an allergy response

A client who had a percutaneous transluminal coronary angioplasty (PTCA) two weeks ago returns to the clinic for a follow up visit. The client has a postoperative ejection fraction ejection fraction of 30%. Today the client has lungs which are clear, +1 pedal edema, and a 5pound weight gain. Which intervention the nurse implement? A. Arrange transport for admission to the hospital. B. Insert saline lock for IV diuretic therapy. C. Assess compliance with routine prescriptions. D. Instruct the client to monitor daily caloric intake.

C. Assess compliance with routine prescriptions. Rationale: Fluid retention may be a sign that the client is not taking the medication as prescribed or that the prescriptions may need adjustment to manage cardiac function post-PTCA (normal ejection fraction range is 50 to 75%)

A client with gestational diabetes, at 39 weeks of gestation, is in the second stage of labor. After delivering of the fetal head, the nurse recognizes that shoulder dystocia is occurring. What intervention should the nurse implement first? A. Prepare the client for an emergency cesarean birth B. Encourage the client to move to a hands-and-knees position. C. Assist the client to sharply flex her thighs up again the abdomen. D. Lower the head of the bed an apply suprapubic pressure.

C. Assist the client to sharply flex her thighs up again the abdomen. Rationale: Flexing the client's thighs against the abdomen (Mc Robert's maneuver) changes the angle o the pelvis and increase the pelvic diameter, making more room for the shoulders to emerge. ABD are implemented after C

A client in the intensive care unit is being mechanically ventilated, has an indwelling urinary catheter in place, and is exhibiting signs of restlessness. Which action should the nurse take first? A. Review the heart rhythm on cardiac monitors B. Check urinary catheter for obstruction C. Auscultated bilateral breath sounds D. Give PRN dose of lorazepam (Ativan)

C. Auscultated bilateral breath sounds Rationale: Restlessness often results from decreased oxygenation so breath sounds should be assessed first. Giving an anxiolytic such as lorazepam, might be indicated but first the client should be assessed for the cause of the restlessness. An obstruction in the urinary drainage system can cause a distended bladder that may result in restlessness, but patent airway is the priority intervention. The client should be assessed before evaluating the cardiac rhythm on the monitor.

A client with a recent colostomy expresses concern about the ability to control flatus. Which intervention is most important for the nurse to include in the client's plan of care? A. Adhere to a bland diet whenever planning to eat out B. Decrease fluid intake at meal times C. Avoid foods that caused gas before the colostomy D. Eliminate foods high in cellulose

C. Avoid foods that caused gas before the colostomy

What action should the school nurse implement to provide secondary prevention to a school-age children? A. Collaborate with a science teacher to prepare a health lesson B. Prepare a presentation on how to prevent the spread of lice C. Initiate a hearing and vision screening program for first-graders D. Observe a person with type 1 diabetes self-administer a dose of insulin

C. Initiate a hearing and vision screening program for first-graders Rationale: Community care occurs at primary, secondary, and tertiary levels of prevention. Primary prevention involves interventions to reduce the incidence of disease. Secondary prevention includes screening programs to detect disease. Tertiary prevention provides treatment directed toward clinically apparent disease. Secondary prevention focuses on screaming children for a specific disease processes such as hearing and vision screening. The other options are not examples of secondary prevention.

A nurse who is working in the emergency department triage area is presented with four clients at the same time. The client presented with which symptoms requires the most immediate intervention by the nurse? A. Low-grade fever, headache, and malaise for the past 72 hours B. Unable to bear weight on the left foot, with the swelling and bruising C. Chest discomfort one hour after consuming a large, spicy meal D. One-inch bleeding laceration on the chain of the crying five-year-old

C. Chest discomfort one hour after consuming a large, spicy meal Rationale: Emergency triage involves quick assessment to prioritize the need for further evaluation and care. Those with trauma, chest pain, respiratory distress, or acute neurological changes are priority. In this example, while clients with other conditions require attention, the client with chest discomfort is at greatest risk and is a priority.

A client who sustained a head injury following an automobile collision is admitted to the hospital. The nurse include the client's risk for developing increased intracranial pressure (ICP) in the plan of care. Which signs indicate to the nurse that ICP has increased? A. Increased Glasgow coma scale score. B. Nuchal rigidity and papilledema. C. Confusion and papilledema D. Periorbital ecchymosis.

C. Confusion and papilledema Rationale: papilledema is always an indicator of increased ICP, and confusion is usually the first sign of increased ICP. Other options do not necessarily reflect increased ICP.

A preeclamptic client who delivered 24h ago remains in the labor and delivery recovery room. She continues to receive magnesium sulfate at 2 grams per hour. Her total input is limited to 125 ml per hour, and her urinary output for the last hour was 850 ml. What intervention should the nurse implement? A. Discontinue the magnesium sulfate immediately B. Decrease the client's iv rate to 50 ml per hour C. Continue with the plan of care for this client D. Change the client's to NPO status

C. Continue with the plan of care for this client Rationale: continue with the plan. Diuresis in 24 to 48h after birth is a sign of improvement in the preeclamptic client. As relaxation of arteriolar spasms occurs, kidney perfusion increases. With improvement perfusion, fluid is drawn into the intravascular bed from the interstitial tissue and then cleared by the kidneys

While caring for a client's postoperative dressing, the nurse observes purulent drainage at the wound. Before reporting this finding to the healthcare provider, the nurse should review which of the client's laboratory values? A. Serum albumin B. Creatinine level C. Culture for sensitive organisms. D. Serum blood glucose (BG) level

C. Culture for sensitive organisms RATIONALE: A client who has a postoperative dressing with purulent drainage from the wound is experiencing an infection. The nurse should review the client's laboratory culture for sensitive organisms (C) before reporting to the healthcare provider. (A, B and D) are not indicated at this time.

The nurse is triaging victims of a tornado at an emergency shelter. An adult woman who has been wandering and crying comes to the nurse. What action should the nurse take? A. Check the client's temperature, blood sugar, and urine output. B. Transport the client for laboratory client for laboratory test and electrocardiogram (EKG) C. Delegate care of the crying client to an unlicensed assistant D. Send the client to the shelter's nutrient center to obtain water and food.

C. Delegate care of the crying client to an unlicensed assistant Rationale: According to the simple triage and Rapid Treatment (START) protocol of triage, the nurse should determine which client fit the objective of providing the greatest good for the greatest number of people who are most likely to survive. Delegating the care of the crying person to an unlicensed assistant allow the nurse to care for the injured who require intervention based on their ability to breath, maintain circulation and follow simple commands. A and B are not indicated at this time. Although food and water may be indicative, the woman's distress should not be dismissed by sending her to the shelter alone.

A young boy who is in a chronic vegetative state and living at home is readmitted to the hospital with pneumonia and pressure ulcers. The mother insists that she is capable of caring for her son and which action should the nurse implement next? A. Report the incident to the local child protective services. B. Find a home health agency that specializes in brain injuries. C. Determine the mother's basic skill level in providing care. D. Consult the ethics committee to determine how to proceed.

C. Determine the mother's basic skill level in providing care. Rational: Although the mother states she is a capable caregiver, the client is manifesting disuse syndrome complications, and the mother's skill in providing basic care should be determined. Further assessment is needed before implementing other nursing actions.

The nurse is presenting information about fetal development to a group of parents with...when discussing cephalocaudal fetal development, which information should the nurse gives the parents? A. A set order in fetal development is expected B. Growth normally occurs within one organ at a time C. Development progress from head to rump D. Organ formation is directed by brain development

C. Development progress from head to rump

The nurse reviews the signs of hypoglycemia with the parents of a child with Type I diabetes mellitus. The parents correctly understand signs of hypoglycemia if they include which symptoms? A. Fruity breath odor B. Polyphagia C. Diaphoresis D. Polydipsia

C. Diaphoresis

A client diagnosed with bipolar disorder is going home on a week-end pass. Which suggestions should give the client's family to help them prepare for the visit? A. Encourage the family to plan daily activities to keep the client busy B. Have friends and family visit the client at a welcome home party C. Discuss the importance of continuing the usual at-home activities D. Instruct family to monitor the client's choice of television programs

C. Discuss the importance of continuing the usual at-home activities

In preparing a diabetes education program, which goal should the nurse identify as the primary emphasis for a class on diabetes self-management? A. Prepare the client to independently treat their disease process B. Reduce healthcare costs related to diabetic complications C. Enable clients to become active participating in controlling the disease process D. Increase client's knowledge of the diabetic disease process and treatment options.

C. Enable clients to become active participating in controlling the disease process Rationale: The primary goal of diabetic self- management education is to enable the client to become an active participant in the care and control of disease process, matching levels of self- management to the abilities of the individual client. The goal is to place the client in a cooperative or collaborative role with healthcare professional rather than (A)

A 12 year old client who had an appendectomy two days ago is receiving 0.9% normal saline at 50 ml/hour. The client's urine specific gravity is 1.035. What action should the nurse implement? A. Evaluate postural blood pressure measurements B. Obtain specimen for uranalysis C. Encourage popsicles and fluids of choice D. Assess bowel sounds in all quadrants

C. Encourage popsicles and fluids of choice Rationale: specific gravity of urine is a measurement of hydration status (normal range of 1.010 to 1.025) which is indicative of fluid volume deficit when Sp Gr increases as urine becomes more concentrated.

A female client is extremely anxious after being informed that her mammogram was abnormal and needs to be repeated. Client is tearful and tells the nurse her mother died of breast cancer. What action should the nurse take? A. Provide the client with information about treatment options for breast cancer. B. Reassure the client that the final diagnosis has not been made. C. Encourage the client to continue expressing her fears and concerns. D. Suggest to the client that she seek a second opinion.

C. Encourage the client to continue expressing her fears and concerns. Rational: the nurse should show support for the client by encouraging her to continue expressing her concerns. A diagnosis has not yet been made, so it is too early to discuss treatment options. Other options dismiss the client's feelings or are premature given that the diagnosis is not yet made.

Following a motor vehicle collision (MCV), a male adult in severe pain is brought to the emergency department via ambulance. His injured left leg is edematous, ecchymotic around the impact of injury on the thigh, and shorter than his right leg. Based on these findings, the client is at greatest risk for which complication? A. Arterial ischemia B. Tissue necrosis C. Fat embolism D. Nerve damage

C. Fat embolism

***An older male client with a history of diabetes mellitus, chronic gout, and osteoarthritis comes to the clinic with a bag of medication bottles. Which intervention should the nurse implement first? A. Record pain evaluation B. Assess blood glucose C. Identify pills in the bag D. Obtain a medical history

C. Identify pills in the bag

A mother brings her 3-week-old son to the clinic because he is vomiting "all the time." In performing a physical assessment, the nurse notes that the infant has poor skin turgor, has lost 20% of his birth weight, and has a small palpable oval-shaped mass in his abdomen. What intervention should the nurse implement first? A. Give the infant 5% dextrose in water orally B. Insert a nasogastric tube for feeding C. Initiate a prescribed IV for parental fluid D. Feed the infant 3 ounces of Isomil

C. Initiate a prescribed IV for parental fluid

If the nurse is initiating IV fluid replacement for a child who has dry, sticky mucous membranes, flushed skin, and fever of 103.6 F. Laboratory finding indicate that the child has a sodium concentration of 156 mEq/L. What physiologic mechanism contributes to this finding? A. The intravenous fluid replacement contains a hypertonic solution of sodium chloride B. Urinary and Gastrointestinal fluid loss reduce blood viscosity and stimulate thirst C. Insensible loss of body fluids contributes to the hemoconcentration of serum solutes D. Hypothalamic resetting of core body temperature causes vasodilation to reduce body heat

C. Insensible loss of body fluids contributes to the hemoconcentration of serum solutes Rationale: Fever causes insensible fluid loss, which contribute to fluid volume and results in hemoconcentration of sodium (serum sodium greater than 150 mEq/L). Dehydration, which is manifested by dry, sticky mucous membranes, and flushed skin, is often managed by replacing lost fluids and electrolytes with IV fluids that contain varying concentration of sodium chloride. Although other options are consistent with fluid volume deficit, the physiologic response of hypernatremia is explained by hem concentration.

An adult client comes to the clinic and reports his concern over a lump that "just popped up on my neck about a week ago." In performing an examination of the lump, the nurse palpates a large, non-tender, hardened left subclavian lymph node. There is no overlying tissue inflammation. What do these finding suggest? A. Bacterial infection B. Lymphangitis C. Malignancy D. Viral infection

C. Malignancy

A male client with an antisocial personality disorder is admitted to an in-patient mental health unit for multiple substance dependency. When providing a history, the client justifies to the nurse his use of illicit drugs. Based on this pattern of behavior this client's history is most likely to include which finding? A. Phobias and panic attacks when confronted by authority figures. B. Suicidal ideations and multiple attempts/ C. Multiple convictions for misdemeanors and class B felonies. D. Delusions of grandiosity and persecution

C. Multiple convictions for misdemeanors and class B felonies.

After receiving report, the nurse can most safely plan to assess which client last? The client with... A. A rectal tube draining clear, pale red liquid drainage B. A distended abdomen and no drainage from the nasogastric tube C. No postoperative drainage in the Jackson-Pratt drain with the bulb compressed D. Dark red drainage on a postoperative dressing, but no drainage in the Hemovac®.

C. No postoperative drainage in the Jackson-Pratt drain with the bulb compressed Rationale: The most stable client is the one with a functioning drainage device and no drainage. This client can most safely be assesses last. Other clients are either actively bleeding, have an obstruction in the nasogastric tube which may result in vomiting, or may be bleeding and / or may have a malfunction in the Hemovac® drain.

Four hours after surgery, a client reports nausea and begins to vomit. The nurse notes that the client has a scopolamine transdermal patch applied behind the ear. What action should the nurse take? A. Reposition the transdermal patch to the client's trunk. B. Remove the transdermal patch until the vomiting subsides. C. Notify the healthcare provider of the vomiting. D. Explain that this is a side effect of the medication in the patch.

C. Notify the healthcare provider of the vomiting. Rational: transdermal scopolamine is used to prevent nausea and vomiting from anesthesia and surgery. The nurse should notify the healthcare provider if the medication is ineffective. The patch should be applied behind the ear and should remain in place to reduce the nausea and vomiting. Nausea and vomiting are no side effects of the medication.

***To reduce the risk of symptoms exacerbation for a client with multiple sclerosis (MS), which instructions should the nurse include in the client's discharge plan? (Select all that apply). A. Practice relaxation exercises B. Limit fluids to avoid bladder distention C. Space activities to allow for rest periods D. Avoid persons with infections E. Take warm baths before starting exercise

C. Space activities to allow for rest periods E. Take warm baths before starting exercise

***A native-American male client diagnosed with pneumonia, states that in addition to his prescribed medical treatment of IV antibiotics he wishes to have a spiritual cleaning performed. Which outcome statement indicates that the best plan of care was followed? A. Identifies his ethnocentric values and behaviors B. States an understanding of the medical treatment C. Participated actively in all treatments regimens D. Expresses a desire for cultural assimilation

C. Participated actively in all treatments regimens Rationale: indicates active participation by the client, which is required for treatment to be successful. The best plan of care should incorporate the valued and treatments of both cultures and in this case there is no apparent cultural clash between the two forms of treatment. The client has already identify he's cultural values (A). (B) Only considers one of the two treatment modalities desired by the client the client has already chosen how he wishes to assimilate his cultural values with the prescribed medical treatment (D).

A client who has been in active labor for 12 hours suddenly tells the nurse that she has a strong urge to have a bowel movement. What action should the nurse take? A. Allow the client to use a bedpan. B. Assist the client to the bathroom C. Perform a sterile vaginal exam D. Explain the fetal head is descending.

C. Perform a sterile vaginal exam Rationale: When a client in active labor suddenly expresses the urge to have a bowel movement, a sterile vaginal exam should be performed to determine if the fetus is descending.

A client with rheumatoid arthritis (RA) starts a new prescription of etanercept (Enbrel) subcutaneously once weekly. The nurse should emphasize the importance of reporting problem to the healthcare provider? A. Headache B. Joint stiffness C. Persistent fever D. Increase hunger and thirst

C. Persistent fever Rationale: Enbrel decrease immune and inflammatory responses, increasing the client's risk of serious infection, so the client should be instructed to report a persistent fever, or other signs of infection to the healthcare provider.

The nurse is demonstrating correct transfer procedures to the unlicensed assisted personnel (UAP) working on a rehabilitation unit. The UAPs ask the nurse how to safely move a physically disabled client from the wheelchair to a bed. What action should the nurse recommended? A. Hold the client at arm's length while transferring to better distribute the body weight. B. Apply the gait belt around the client's waits once standing position has been assumed. C. Place a client's locked wheelchair on the client's strong side next to the bed. D. Pull the client into position by reaching from the opposite side of the bed.

C. Place a client's locked wheelchair on the client's strong side next to the bed. RATIONALE: Placing the wheelchair on the client's strong side offers the greatest stability for the transfer. Holding the client arm's length or pulling from the opposite site of the bed reflect poor body mechanism. Using a gait belt offers additional safety for the client, but should be done after the wheelchair has be put into the proper place and the wheels have been locked and before the client has assumed a standing position.

During a Woman's Health fair, which assignment is the best for the Practical Nurse (PN) who is working with a register nurse (RN) A. Encourage the woman at risk for cancer to obtain colonoscopy. B. Present a class of breast-self examination C. Prepare a woman for a bone density screening D. Explain the follow-up need it for a client with prehypertension.

C. Prepare a woman for a bone density screening Rationale: A bone density screening is a fast, noninvasive screening test for osteoporosis that can be explained by the PN. There is no additional preparation needed (A) required a high level of communication skill to provide teaching and address the client's fear. (B) Requires a higher level of client teaching skill than responding to one client. (D) Requires higher level of knowledge and expertise to provide needed teaching regarding this complex topic.

A female client who was mechanically ventilated for 7 days is extubated. Two hours later...productive cough, and her respirations are rapids and shallow. Which intervention is most important? A. Review record of recent analgesia B. Provide frequent pulmonary toilet C. Prepare the client for intubation D. Obtain STAT arterial blood gases

C. Prepare the client for intubation

The nurse is planning preoperative teaching plan of a 12-years old child who is scheduled for surgery. To help reduce the child anxiety, which action is the best for the nurse to implement? A. Give the child syringes or hospital mask to play it at home prior to hospitalization. B. Include the child in pay therapy with children who are hospitalized for similar surgery. C. Provide a family tour of the preoperative unit one week before the surgery is scheduled. D. Provide doll an equipment to re-enact feeling associated with painful procedures.

C. Provide a family tour of the preoperative unit one week before the surgery is scheduled. Rationale: School age children gain satisfaction from exploring and manipulating their environment, thinking about objectives, situations and events, and making judgments based on what they reason. A tour of the unit allows the child to see the hospital environment and reinforce explanation and conceptual thinking.

A client's telemetry monitor indicates ventricular fibrillation (VF). What should the nurse do first? A. Administer epinephrine IV B. Give an IV bolus of amiodarone C. Provide immediate defibrillation D. Prepare for synchronized cardioversion

C. Provide immediate defibrillation

A client with emphysema is being discharged from the hospital. The nurse enters the client's room to complete discharge teaching. The client reports feeling a little short of breath and is anxious about going home. What is the best course of action? A. Postpone discharge instructions at this time and offer to contact the client by phone in a few days B. Invite the client to return to the unit for discharge teaching in a few days, when there is less anxiety C. Provide only necessary information in short, simple explanations with written instructions to take home D. Give detailed instructions speaking slowly and clearly while looking directly at the client when speaking

C. Provide only necessary information in short, simple explanations with written instructions to take home Rationale: Simple, short explanations should be provided. Information is not retained when the recipient is anxious, and too much information can increase worry. Ethically, discharge instructions may not be postponed.

An unlicensed assistive personnel (UAP) reports that a client's right hand and fingers spasms when taking the blood pressure using the same arm. After confirming the presence of spams what action should the nurse take? A. Ask the UAP to take the blood pressure in the other arm B. Tell the UAP to use a different sphygmomanometer. C. Review the client's serum calcium level D. Administer PRN antianxiety medication.

C. Review the client's serum calcium level Rationale: Trousseau's sign is indicated by spasms in the distal portion of an extremity that is being used to measure blood pressure and is caused by hypocalcemia (normal level 9.0-10.5 mg/dl, so C should be implemented.

When assessing and adult male who presents as the community health clinic with a history of hypertension, the nurse note that he has 2+ pitting edema in both ankles. He also has a history of gastroesophageal reflex disease (GERD) and depression. Which intervention is the most important for the nurse to implement? A. Arrange to transport the client to the hospital B. Instruct the client to keep a food journal, including portions size. C. Review the client's use of over the counter (OTC) medications. D. Reinforce the importance of keeping the feet elevated.

C. Review the client's use of over the counter (OTC) medications. Rationale: Sodium is used in several types of OTC medications. Including antacids, which the client may be using to treat his GERD. Further evaluation is need it to determine the need for hospitalization (A) A food journal (B) may help over, but dietary modifications are needed now since edema is present. (C) May relieve dependent edema, but not treat the underlying etiology.

A male client reports the onset of numbness and tingling in his fingers and around his mouth. Which lab is important for the nurse to review before contacting the health care provider? A. Capillary glucose B. Urine specific gravity C. Serum calcium D. White blood cell count

C. Serum calcium A calcium deficiency can cause numbness and tingling in the hands, arms, feet, legs, and around the mouth

The nurse is preparing to administer an IV dose of ciprofloxacin to a client with urinary tract infection. Which client data requires the most immediate intervention by the nurse? A. Urine culture positive for MRSA B. Serum sodium of 145 mEq/L (145 mmol/L SI) C. Serum creatinine of 4.5 mg/dl (398 mcmol/L SI) D. White blood cell count of of 12,000 mm3 (12 x 109/L SI)

C. Serum creatinine of 4.5 mg/dl (398 mcmol/L SI)

A young adult client is admitted to the emergency room following a motor vehicle collision. The client's head hit the dashboard. Admission assessment include: Blood pressure 85/45 mm Hg, temperature 98.6 F, pulse 124 beat/minute and respirations 22 breath/minute. Based on these data, the nurse formulates the first portion of nursing diagnosis as " Risk of injury" What term best expresses the "related to" portion of nursing diagnosis? A. Infection B. Increase intracranial pressure C. Shock D. Head Injury.

C. Shock

A 35 years old female client has just been admitted to the post anesthesia recovery unit following a partial thyroidectomy. Which statement reflects the nurse's accurate understanding of the expected outcome for the client following this surgery? A. Supplemental hormonal therapy will probably be unnecessary B. The thyroid will regenerate to a normal size within a few years. C. The client will be restricted from eating seafood D. The remainder of the thyroid will be removed at a later date.

C. The client will be restricted from eating seafood

The nurse is assessing an older adult with type 2 diabetes mellitus. Which assessment finding indicates that the client understands long- term control of diabetes? A. The fating blood sugar was 120 mg/dl this morning. B. Urine ketones have been negative for the past 6 months C. The hemoglobin A1C was 6.5g/100 ml last week D. No diabetic ketoacidosis has occurred in 6 months.

C. The hemoglobin A1C was 6.5g/100 ml last week Rationale: A hemoglobin A1C level reflects he average blood sugar the client had over the previous 2 to 3 month, and level of 6.5 g/100 ml suggest that the client understand long-term diabetes control. Normal value in a diabetic patient is up to 6.5 g/100 ml.

A client is admitted to a medical unit with the diagnosis of gastritis and chronic heavy alcohol abuse. What should the nurse administered to prevent the development of Wernicke's syndrome? A. Lorazepam (Ativan) B. Famotidine (Pepcid) C. Thiamine (Vitamin B1) D. Atenolol (Tenormin)

C. Thiamine (Vitamin B1) Rationale: Thiamine replacement is critical in preventing the onset of Wernickes encephalopathy, an acute triad of confusion, ataxia, and abnormal extraocular movements, such as nystagmus related to excessive alcohol abuse. Other medications are not indicated.

The public nurse health received funding to initiate primary prevention program in the community. Which program the best fits the nurse's proposal? A. Case management and screening for clients with HIV. B. Regional relocation center for earthquake victims C. Vitamin supplements for high-risk pregnant women. D. Lead screening for children in low-income housing.

C. Vitamin supplements for high-risk pregnant women. Rational: Primary prevention activities focus on health promotions and disease preventions, so vitamin for high-risk pregnant women provide adequate vitamin and mineral for fetal developmental.

A client who is experiencing musculoskeletal pain receives a prescription for ketorolac 15mg IM q6 hours. The medication is depended in a 39mg/ml pre-filled syringe. Which action should the nurse implement when giving the medication? A. Administer the entire pre-filled syringe deep in the dorsogluteal site. B. Use a separate syringe to remove 15mg from the pre-filled syringe and give in the back of the arm. C. Waste 0.5 ml from the pre-filled syringe and inject the medication in the ventrogluteal site. D. Call the healthcare provider to request a prescription change to match the dispensed 30mg dose.

C. Waste 0.5 ml from the pre-filled syringe and inject the medication in the ventrogluteal site. RATIONALE: The pre-filled contain 30mg /1ml, so 0.5ml should be wasted to obtain the correct dosage of 15mg for administration in the preferred IM ventrogluteal site. The nurse is responsible for calculating and preparing the prescribed dose using the available concentration, so other options are not indicated.

A nurse working on an endocrine unit should see which client first? A. An adolescent male with diabetes who is arguing about his insulin dose. B. An older client with Addison's disease whose current blood sugar level is 62mg/dl (3.44 mmol/l). C. An adult with a blood sugar of 384mg/dl (21.31mmol/l) and urine output of 350 ml in the last hour. D. A client taking corticosteroids who has become disoriented in the last two hours.

D. A client taking corticosteroids who has become disoriented in the last two hours. Rational: meeting the client's need for safety is a priority intervention. Mania and psychosis can occur during corticosteroids therapy, places the client at risk for injury, so the patient taking corticosteroids should be seen first.

The nurse who is working on a surgical unit receives change of shift report on a group of clients for the upcoming shift. A client with which condition requires the most immediate attention by the nurse? A. Gunshot wound three hours ago with dark drainage of 2 cm noted on the dressing. B. Mastectomy 2 days ago with 50 ml bloody drainage noted in the Jackson-pratt drain. C. Collapsed lung after a fall 8h ago with 100 ml blood in the chest tube collection container D. Abdominal-perineal resection 2 days ago with no drainage on dressing who has fever and chills.

D. Abdominal-perineal resection 2 days ago with no drainage on dressing who has fever and chills. Rationale: the client with an abdominal- perineal resection is at risk for peritonitis and needs to be immediately assessed for other signs and symptoms for sepsis.

While making rounds, the charge nurse notices that a young adult client with asthma who was admitted yesterday is sitting on the side of the bed and leaning over the bed-side-table. The client is currently receiving at 2 litters/minute via nasal cannula. The client is wheezing and is using pursed-lip breathing. Which intervention should the nurse implement? A. Assist the client to lie back in bed B. Call for an Ambu resuscitating bag C. Increase oxygen to 6 litters/minute D. Administer a nebulizer Treatment

D. Administer a nebulizer Treatment Rationale: The client needs an immediate medicated nebulizer treatment. Sitting in an upright position with head and arms resting on the over-bed table is an ideal position to promote breathing because it promotes lung expansion. Other actions me be accurate but not yet indicated.

A client arrives in the emergency center with a blood alcohol level of 500 mg/dl. When transferred to the observation unit, the client becomes demanding, aggressive, and shouts at the staff. Which assessments finding is most important for the nurse to identify in the first 24 hours? A. Decreased appetite B. Nausea and elevated blood pressure C. Difficulty walking D. Agitation and threats to harms staff

D. Agitation and threats to harms staff

The nurse is caring for a client with acute kidney injury (AKI) secondary to gentamicin therapy the client's serum blood potassium is elevated, which finding requires immediate action by the nurse? A. Tall peak T waves on the cardiac monitor B. Peripheral pitting edema at 2 + indentation C. Serum creatinine above 0.5 mg/dl or 44.2 micro-mmol/dl D. Anuria for the last 12 hours.

D. Anuria for the last 12 hours.

The nurse is teaching a male client with multiple sclerosis how to empty his bladder using the Crede Method. When performing a return demonstration, the client applies pressure to the umbilical areas of his abdomen. What instruction should the nurse provide? A. Stroke the inner thigh below the perineum to initiate urinary flow B. Contract, hold, and then relax the pubococcygeal muscle C. Pour warm water over the external sphincter at the distal glans D. Apply downward manual pressure at the suprapubic regions.

D. Apply downward manual pressure at the suprapubic regions. Rationale: The Crede Method is used for those clients with atonic bladders, which is a concomitant of demyelinating disorders like multiple sclerosis. The client is applying pressure in the wrong region (umbilical Are) and should be instructed to apply pressure at the suprapubic are.

The nurse who works in labor and delivery is reassigned to the cardiac care unit for the day because of a low census in labor and delivery. Which assignments is best for the nurse to give this nurse? A. Transfer a client to another unit B. Monitor the central telemetry C. Perform the admission D. Assist cardiac nurses with their assignments

D. Assist cardiac nurses with their assignments Rationale: When receiving staff from another specialty unit, the charge nurse should allow the nurse to assist where possible (D) without taking a client assignment so that the nurse is not asked to perform unfamiliar skills (A, B, C) are likely to involve skills the nurse is not accustomed to performing.

A confused, older client with Alzheimer's disease becomes incontinent of urine when attempting to find the bathroom. Which action should the nurse implement? A. Instruct the client to use the call button when a bedpan is needed B. Apply adult diapers after each attempt to void C. Check residual urine volume using an indwelling urinary catheter D. Assist the client's to a bedside commode every two hours

D. Assist the client's to a bedside commode every two hours

The nurse identifies an electrolyte imbalance, an elevated pulse rate, and elevated BP for a client with chronic kidney disease. Which is the most important action for the nurse to take? A. Monitor daily sodium intake. B. Record usual eating patterns. C. Measure ankle circumference. D. Auscultate for irregular heart rate.

D. Auscultate for irregular heart rate. Rational: Chronic kidney failure (CKF) is a progressive, irreversible loss of kidney functions, decreasing glomerular filtration rate (GFR), and the kidney's inability to excrete metabolic waste products and water, resulting in fluid overload, elevated pulse, elevated BP and electrolytes imbalances. The most important action for the nurse to implement is to auscultate for irregular heart rate (D) due to the decreased excretion of potassium by the kidneys. (A, B, and C) are not as important as monitoring for fatal cardiac dysrhythmias related to hyperkalemia.

A client with angina pectoris is being discharge from the hospital. What instruction should the nurse plan to include in this discharge teaching? A. Engage in physical exercise immediately after eating to help decrease cholesterol levels. B. Walk briskly in cold weather to increase cardiac output C. Keep nitroglycerin in a light-colored plastic bottle and readily available. D. Avoid all isometric exercises, but walk regularly.

D. Avoid all isometric exercises, but walk regularly. Rationale: Isometric exercise can raise blood pressure for the duration of the exercise, which may be dangerous for a client with cardiovascular disease, while walking provides aerobic conditioning that improves ling, blood vessel, and muscle function. Client with angina should refrain from physical exercise for 2 hours after meals, but exercising does not decrease cholesterol levels. Cold water cause vasoconstriction that may cause chest pain. Nitroglycerin should be readily available and stored in a dark-colored glass bottle not C, to ensure freshness of the medication.

The nurse is preparing a discharge teaching plan for a client who had a liver transplant. Which instruction is most important to include in this plan? A. Limit intake fatty foods for one month after surgery. B. Notify the healthcare provider if edema occurs. C. Increase activity and exercise gradually, as tolerated. D. Avoid crowds for first two months after surgery.

D. Avoid crowds for first two months after surgery. Rationale: Cyclosporine immunosuppression therapy is vital in the success of liver transplantation and can increase the risk for infection, which is critical in the first two months after surgery. Fever is often.

A 13 years-old client with non-union of a comminuted fracture of the tibia is admitted with osteomyelitis. The healthcare provider collects home aspirate specimens for culture and sensitivity and applies a cast to the adolescent's lower leg. What action should the nurse implement next? A. Administer antiemetic agents B. Bivalve the cast for distal compromise C. Provide high- calorie, high-protein diet D. Begin parenteral antibiotic therapy

D. Begin parenteral antibiotic therapy Rationale: The standard of treatment for osteomyelitis is antibiotic therapy and immobilization. After bond and blood aspirate specimens are obtained for culture and sensitivity, the nurse should initiate parenteral antibiotics as prescribed.

The nurse is planning care for a client who admits having suicidal thoughts. Which client behavior indicates the highest risk for the client acting on these suicidal thoughts? A. Express feelings of sadness and loneliness B. Neglects personal hygiene and has no appetite C. Lacks interest in the activity of the family and friends D. Begin to show signs of improvement in affect

D. Begin to show signs of improvement in affect Rationale: when a depressed client begins to show signs of improvement, it can be because the client has "figured out" how to be successful in committing suicide. Depressed clients, particularly those who have shown signs of potentially becoming suicidal, should be watched with care for an impending suicide attempt might be greater when the client appear suddenly happy, begin to give away possessions, or becomes more relaxed and talkative.

A client with a lower respiratory tract infection receives a prescription for ciprofloxacin 500mg PO q 12hours. When the client request an afternoon snack, which dietary choice should the nurse provide? A. Vanilla-flavored yogurt B. Low fat chocolate milk. C. Calcium fortified juice D. Cinnamon applesauce

D. Cinnamon applesauce RATIONALE: Dairy products and calcium fortified dairy products decrease the absorption of ciprofloxacin. Cinnamon applesauce contains no calcium, so this is the best snack selection. Since other options contains calcium, these snack should be avoided by a client who is taking ciprofloxacin.

The home health nurse is assessing a male client who has started peritoneal dialysis (PD) 5 days ago. Which assessment finding warrants immediate intervention by the nurse? A. Finger stick blood glucose 120 mg/dL post exchange B. Arteriovenous (AV) graft surgical site pulsations. C. Anorexia and poor intake of adequate dietary protein D. Cloudy dialysate output and rebound abdominal pain

D. Cloudy dialysate output and rebound abdominal pain

When administering an immunization in an adult client, the nurse palpates and administer the injection one inch below the acromion process into the center of the muscle mass. The nurse should document that the vaccine was administered at what site? A. Rectus femenis B. Ventrogluteous C. Vastus lateralis D. Deltoid

D. Deltoid Rationale: The acromion process is a parameter identified for the deltoid site.

The nurse is preparing an intravenous (IV) fluid infusion using an IV pump. Within 30 seconds of turning on the machine, the pump's alarm beeps "occlusion". What action should the nurse implement first? A. Flush the vein with 3 ml of sterile normal saline. B. Assess the IV catheter insertion site for infiltration. C. Verify the threading of the tubing through the IV pump. D. Determine if the clamp on the IV tubing is released

D. Determine if the clamp on the IV tubing is released Rational: When the pump immediately beeps, it is often because the IV tubing clamp is occluding the flow, so the clamp should be checked first to ensure that it is open. If the alarm is not eliminated after the tubing clamp is released, flushing the IV site with saline is a common practice to clean the needle or to identify resistance due to another source. Local signs of infiltration may indicate the need to select another vein, but the pump's beeping-this early in the procedure is likely due to a mechanical problem. If beeping continues after verifying that the clamp is released the placement or threading of the tubing through the pump should be verified.

The nurse assigned unlicensed assistive personnel (UAP) to apply antiembolism stockings to a client. The nurse and UAP enters the room, the nurse observes the stockings that were applying by the UAP. The UAP states that the client requested application of the stockings as seen on the picture, for increased comfort. What action should the nurse take? A. Ask the client if the stocking feel comfortable. B. Supervise the UAP in the removal of the stockings. C. Place a cover over the client's toes to keep them warm. D. Discussed effective use of the stockings with the client on UAP

D. Discussed effective use of the stockings with the client on UAP Rational: antiembolism stockings are designed to fit securely and should be applied so that there are no bands of the fabric constricting venous return. The nurse should discuss the need for correct and effective use of the stockings with both the client and UAP to improve compliance. Other options do not correct the incorrect application of the stockings.

The nurse note a depressed female client has been more withdrawn and non-communicative during the past two weeks. Which intervention is most important to include in the updated plan of care for this client? A. Encourage the client's family to visit more often B. Schedule a daily conference with the social worker C. Encourage the client to participate in group activities D. Engage the client in a non-threatening conversation.

D. Engage the client in a non-threatening conversation. Rationale: Consistent attempts to draw the client into conversations which focus on non-threatening subjects can be an effective means of eliciting a response, thereby decreasing isolation behaviors. There is not sufficient data to support the effectiveness of A as an intervention for this client. Although B may be indicated, nursing interventions can also be used to treat this client. C is too threatening to this client.

A client who is admitted to the intensive care unit with syndrome of inappropriate antidiuretic hormone (SIADH) has developed osmotic demyelination. Which intervention should the nurse implement first? A. Patch one eye. B. Reorient often. C. Range of motion. D. Evaluate swallow

D. Evaluate swallow Rational: Osmotic demyelination, also known as central pontine myelinolysis, is nerve damage caused by the destruction of the myelin sheath covering nerve cells in the brainstem. The most common cause is a rapid, drastic change in sodium levels when a client is being treated for hyponatremia, a common occurrence in SIADH. Difficulty swallowing due to brainstem nerve damage should be care, but determining the client's risk for aspiration is most important.

A clinical trial is recommended for a client with metastatic breast cancer, but she refuses to participate and tells her family that she does not wish to have further treatments. The client's son and daughter ask the nurse to try and convince their mother to reconsider this decision. How should the nurse respond? A. Ask the client with her children present if she fully understands the decision she has made. B. Discuss success of clinical trials and ask the client to consider participating for one month. C. Explain to the family that they must accept their mother's decision. D. Explore the client's decision to refuse treatment and offer support

D. Explore the client's decision to refuse treatment and offer support Rationale: as long as the client is alert, oriented and aware of the disease prognosis, the healthcare team must abide by her decisions. Exploring the decision with the client and offering support provides a therapeutic interaction and allows the client to express her fears and concerns about her quality of life. Other options are essentially arguing with the client's decisions regarding her end of life treatment or diminish the opportunity for the client to discuss her feelings

When conducting diet teaching for a client who was diagnosed with nutritional anemia in pregnancy, which foods should the nurse encourage the client to eat? (Select all that apply) A. Seeds, spices, lettuce B. Consomme, celery, carrot C. Oranges, orange juice, bananas D. Fortified whole wheat cereals, whole-grain pasta, brown rice E. Spinach, kale, dried raisins and apricots

D. Fortified whole wheat cereals, whole-grain pasta, brown rice E. Spinach, kale, dried raisins and apricots Rationale: Nutritional anemia in pregnancy should be supplemented with additional iron in the diet. Foods that are high in iron content are often protein based, whole grains (D), green leafy vegetables and dried fruits (E). (A, B, and C) are not iron rich sources

Oral antibiotics are prescribed for an 18-month-old toddler with severe otitis media. An antipyrine and benzocaine-otic also prescribed for pain and inflammation. What instruction should the nurse emphasize concerning the installation of the antipyrine/benzocaine otic solution? A. Place the dropper on the upper outer ear canal and instill the medication slowly. B. Warm the medication in the microwave for 10 seconds before instilling. C. Keep the medication refrigerated between administrations. D. Have the child lie with the ear up for one to two minute after installation.

D. Have the child lie with the ear up for one to two minute after installation.

The nurse is teaching a mother of a newborn with a cleft lip how to bottle feed her baby using medela haberman feeder, which has a valve to control the release of milk and a slit nipple opening. The nurse discusses placing the nipple's elongated tip in the back of the oral cavity. What instructions should the nurse provide the mother about feedings? A. Squeeze the nipple base to introduce milk into the mouth B. Position the baby in the left lateral position after feeding C. Alternate milk with water during feeding D. Hold the newborn in an upright position

D. Hold the newborn in an upright position Rationale: the mother should be instructed to hold the infant during feedings in a sitting or upright position to prevent aspiration. Impaired sucking is compensated by the use of special feeding appliances and nipples such as the haberman feeder that prevents aspiration by adjusting the flow of mild according to the effort of the neonate. Squeezing the nipple base may introduce a volume that is greater than the neonate can coordinate swallowing. The preferred positon of an infant after feeding is on the right side to facilitate stomach emptying. Sucking difficulty impedes the neonate's intake of adequate nutrient needed for weight gain and water should be provided after the feeding to cleanse the oral cavity and not fill up the neonate's stomach.

The nurse plans to administer a schedule dose of metoprolol (Toprol SR) at 0900 to a client with hypertension. At 0800, the nurse notes that client's telemetry pattern shows a second degree heart block with a ventricular rate of 50. What action should the nurse take? A. Administer the Tropol immediately and monitor the client until the heart rate increases. B. Provide the dose of Tropol as scheduled and assign a UAP to monitor the client's BP q30 minutes. C. Give the Tropol as scheduled if the client's systolic blood pressure reading is greater than 180. D. Hold the scheduled dose of Tropol and notify the healthcare provider of the telemetry pattern.

D. Hold the scheduled dose of Tropol and notify the healthcare provider of the telemetry pattern. Rationale: Beta blockers such as metoprolol (Tropol SR) are contraindicated in clients with second or third degree heart block because they decrease the heart rate. Therefore, the nurse should hold the medication.

The nurse walks into a client's room and notices bright red blood on the sheets and on the floor by the IV pole. Which action should the nurse take first? A. Clean up the spilled blood to reduce infection transmission. B. Notify the healthcare provider that the client appears to be bleeding. C. Apply direct pressure to the client's IV site. D. Identify the source and amount of bleeding.

D. Identify the source and amount of bleeding. Rationale: the nursed should first assess the client to determine the action that should be taken. Patient safety is the priority; other options are not priority.

An adult male who was admitted two days ago following a cerebrovascular accident (CVA) is confused and experiencing left-side weakness. He has tried to get out of bed several times, but is unable to ambulate without assistance. Which intervention is most important for the nurse to implement? A. Ask a family member to sit with the client B. Apply bilateral soft wrist restraints C. Assign staff to check client q15 minutes D. Install a bed exit safety monitoring device

D. Install a bed exit safety monitoring device

The charge nurse observes a new nurse preparing to insert an intravenous (IV) catheter. The new nurse has gathered supplies, including intravenous catheters, an intravenous insertion kit, and a 4x4 sterile gauze dressing to cover and secure the insertion site. What action should the charge nurse take? A. Plan to observe the secured IV site after the insertion procedure B. Confirm that the nurse has gathered the necessary supplies C. Remind the nurse to tape the gauze dressing securely in place D. Instruct the nurse to use a transparent dressing over the site

D. Instruct the nurse to use a transparent dressing over the site

The nurse inserts an indwelling urinary catheter as seen in the video what action should the nurse take next? A. Remove the catheter and insert into urethral opening B. Observe for urine flow and then inflate the balloon. C. Insert the catheter further and observe for discomfort. D. Leave the catheter in place and obtain a sterile catheter.

D. Leave the catheter in place and obtain a sterile catheter. Rationale: the catheter is in the vaginal opening.

While assisting a client who recently had a hip replacement into a bed pan, the nurse notices that there is a small amount of bloody drainage on the surgical dressing, the client's skin is warm to the touch, and there is a strong odor from the urine. Which action should the nurse take? A. Obtain a urine sample from the bed pan B. Remove dressing and assess surgical site C. Insert an indwelling urinary catheter D. Measure the client's oral temperature

D. Measure the client's oral temperature Rationale: The strong odor from the urine and skin that is warm to the touch may indicate that the client has a urinary tract infection. Assessing the client's temperature provides objective information regarding infection that can be reported to the healthcare provider. Urine should be obtained via a clean catch, not the bed pan where it has been contaminated. The drainage on the dressing is normal and does not require direct conservation at this time. An indwelling catheter should be avoided if possible because it increases the risk of infection.

A client with a history of using illicit drugs intravenously is admitted with Kaposi's sarcoma. Which intervention should the nurse include in this client's admission plan of care? A. Identify local support HIV support groups. B. Assess for symptoms of AIDS dementia. C. Observe for adverse drug reaction. D. Monitor for secondary infections.

D. Monitor for secondary infections.

The nurse is interviewing a client with schizophrenia. Which client behavior requires immediate intervention? A. Lip smacking and frequent eye blinking B. Shuffling gait and stooped posture C. Rocks back and forth in the chair D. Muscle spasms of the back and neck

D. Muscle spasms of the back and neck Rationale: An extra pyramidal symptom (EPS) characterized by abnormal muscle spasms of the neck (A) requires immediate intervention because it can cause difficulty swallowing and jeopardize the airway. Though (A, B and C) are also EPS caused by antipsychotic medication medications used to manage schizophrenia (D) has the highest priority to insure client safety is (A)

The nurse should observe most closely for drug toxicity when a client receives a medication that has which characteristic? A. Low bioavailability B. Rapid onset of action C. Short half life D. Narrow therapeutic index.

D. Narrow therapeutic index. Rationale: A drug with a narrow therapeutic index has a high risk for toxicity because there is a narrow range between the therapeutic dose and the toxic dose.

A client with myasthenia Gravis (MG) is receiving immunosuppressive therapy. Review recent laboratory test results show that the client's serum magnesium level has decreased below the normal range. In addition to contacting the healthcare provider, what nursing action is most important? A. Check the visual difficulties B. Note most recent hemoglobin level C. Assessed for he and Hand joint pain D. Observe rhythm on telemetry monitor

D. Observe rhythm on telemetry monitor Rationale: If not treated a low little Serum magnesium level can affect myocardial depolarization leading to a lethal arrhythmia, and the nurse should assess for dysrhythmias before contacting the healthcare provider. Other choices are common in MG but do not contribute the Safety risk of low magnesium levels.

In caring for a client with a PCA infusion of morphine sulfate through the right cephalic vein, The nurse assesses that the client in lethargic with a blood pressure of 90/60, pulse rate of 118 beats per minute, and respiratory rate of 8 breaths per minutes. What assessment should the nurse perform next? A. Note the appearance and patency of the client's peripheral IV site. B. Palpate the volume of the client's right radial pulse C. Auscultate the client's breath sounds bilaterally. D. Observe the amount and dose of morphine in the PCA pump syringe.

D. Observe the amount and dose of morphine in the PCA pump syringe.

Following an open reduction of the tibia, the nurse notes bleeding on the client's cast. Which action should the nurse implement? A. No action is required since postoperative bleeding can be expected B. Lower the client's head while assessing for symptoms of shock C. Call the health care provider and prepare to take the client back to the operating room D. Outline the area with ink and check it every 15 minutes to see if the area has increased

D. Outline the area with ink and check it every 15 minutes to see if the area has increased

The nurse is planning to assess a client's oxygen saturation to determine if additional oxygen is needed via nasal cannula. The client has a bilateral below-the-knee amputation and pedal pulses that are weak and threaty. What action should the nurse take? A. Document that an accurate oxygen saturation reading cannot be obtained B. Elevate to client's hands for five minutes prior to obtaining a reading from the finger C. Increase the oxygen based on the clients breathing patterns and lung sounds D. Place the oximeter clip on the ear lobe to obtain the oxygen saturation reading

D. Place the oximeter clip on the ear lobe to obtain the oxygen saturation reading Rationale: Pulse oximeter clips can be attached to the earlobe to obtain an accurate measurement of oxygen saturation. Other options will not provide the needed assessment.

The nurse is preparing to gavage feed a premature infant through an orogastric tube. During insertion of the tube, the infant's heart rate drops to 60 beats / minute. Which action should the nurse take? A. Continue the insertion since this is a typical response B. pause and monitor for a continues drop of the heart rate C. Insert the feeding tube into the infant's nasal passage D. Postpone the feeding until the infant's vital signs and stable

D. Postpone the feeding until the infant's vital signs and stable

When assessing the surgical dressing of a client who had abdominal surgery the previous day, the nurse observes that a small amount of drainage is present on the dressing and the wound's Hemovac suction device is empty with the plug open. How should the nurse respond? A. Replace the dressing and remove the drainage device B. Reposition the drainage device and keep the plug open C. Notify the healthcare provider that the drain is not working D. Recompress the wound suction device and secure to plug

D. Recompress the wound suction device and secure to plug Rationale: The plug of a wound suction device, such as a Hemovac, should be closed after compressing the device to apply gentle suction in a closed surgical wound to facilitate the evacuation of subcutaneous fluids into the device. Compressing the device and securing the plug should restore function of the closed wound device. A small amount of drainage should be marked on the dressing, but replacing the dressing is not necessary and the nurse should not remove the device. Other options are not indicated.

Following routine diagnostic test, a client who is symptom-free is diagnosed with Paget's disease. Client teaching should be directed toward what important goal for this client? A. Maintain adequate cardiac output B. Promote adequate tissue perfusion C. Promote rest and sleep D. Reduce the risk for injury

D. Reduce the risk for injury

When gathering for a group therapy session at 1400 hours, a female client complains to the nurse that a smoking break has not been allowed all day. The nurse responds that 15 minute breaks were called over the unit intercom after breakfast and after lunch. The nurse is using what communication technique in responding to the client? A. Doubt B. Observation C. Confrontation D. Reflection

D. Reflection

An older adult male is admitted with complications related to chronic obstructive pulmonary disease (COPD). He reports progressive dyspnea that worsens on exertion and his weakness has increased over the past month. The nurse notes that he has dependent edema in both lower legs. Based on these assessment findings, which dietary instruction should the nurse provide? A. Limit the intake of high calorie foods. B. Eat meals at the same time daily. C. Maintain a low protein diet. D. Restrict daily fluid intake.

D. Restrict daily fluid intake. Rationale: the client is exhibiting signs of cor pulmonale, a complication of COPD that causes the right side of the heart to fail. Restricting fluid intake to 1000 to 2000 ml/day, eating a high-calorie diet at small frequent meals with foods that are high in protein and low in sodium can help relive the edema and decrease workload on the right-side of the heart.

Following an outbreak of measles involving 5 students in an elementary school, which action is most important for the school nurse to take? A. Review the immunization records of all children in the elementary school B. Report the measles outbreak to all community health organizations C. Schedule a mobile public health vehicle to offer measles inoculations to unvaccinated children. D. Restrict unvaccinated children from attending school until measles outbreak is resolved.

D. Restrict unvaccinated children from attending school until measles outbreak is resolved.

When assessing a 6-month old infant, the nurse determines that the anterior fontanel is bulging. In which situation would this finding be most significant? A. Crying B. Straining on stool C. Vomiting D. Sitting upright.

D. Sitting upright. Rationale: The anterior fontanel closes at 9 months of age and may bulge when venous return is reduced from the head, but a bulging anterior fontanel is most significant if the infant is sitting up and may indicated an increase in cerebrospinal fluid. Activities that reduce venous return from the head, such as crying, a Valsalva maneuver, vomiting or a dependent position of the head, cause a normal transient increase in intracranial pressure.

A client with a history of diabetes and coronary artery disease is admitted with shortness of breath, anxiety, and confusion. The client's blood pressure is 80/60 mmHg, heart rate 120 beats/minute with audible third and fourth heart sounds, and bibasilar crackles. The client's average urinary output is 5 ml/hour. Normal saline is infusing at 124 ml/hour with a secondary infusion of dopamine at mcg/kg/minute per infusion pump. With intervention should the nurse implement? A. Irrigate the indwelling urinary catheter. B. Prepare the client for external pacing. C. Obtain capillary blood glucose measurement. D. Titrate the dopamine infusion to raise the BP.

D. Titrate the dopamine infusion to raise the BP. Rationale: the client is experiencing cardiogenic shock and requires titration per protocol of the vasoactive secondary infusion, dopamine, to increase the blood pressure. Low hourly urine output is due to shock and does not indicate a need for catheter irrigation. Pacing is not indicated based on the client's capillary blood glucose should be monitored, but is not directly indicated at this time.

While caring for a toddler receiving oxygen (02) via face mask, the nurse observes that the child's lips and nares are dry and cracked. Which intervention should the nurse implement? A. Ask the mother what she usually uses on the child's lips and nose B. Apply a petroleum jelly (Vaseline) to the child's nose and lips C. Use a topical lidocaine (Zylocaine viscous) analgesic for cracked lips D. Use a water soluble lubricant on affected oral and nasal mucosa

D. Use a water soluble lubricant on affected oral and nasal mucosa

An adult male who fell from a roof and fractures his left femur is admitted for surgical stabilization after having a soft cast applied in the emergency department. Which assessment finding warrants immediate intervention by the nurse? A. Onset of mild confusion B. Pain score 8 out of 10 C. Pale, diaphoretic skin D. Weak palpable distal pulses

D. Weak palpable distal pulses

An infant who is admitted for surgical repair of a ventricular septal defect (VSD) is irritable and diaphoretic with jugular vein distention. Which prescription should the nurse administer first?

Digoxin

***An Unna boot is applied to a client with a venous stasis ulcer. One week later, when the Unna boot is removed during a follow-up appointment, the nurse observes that the ulcer site contains bright red tissue. What action should the nurse take in response to this finding?

Document the ongoing wound healing.

***While completing an admission assessment for a client with unstable angina, which closed questions should the nurse ask about the client's pain?

Does your pain occur when walking short distances?

***An adult man reports that he recently experienced an episode of chest pressure and breathlessness when he was jogging in the neighborhood. He expresses concern because both of his deceased parents had heart disease and his father was a diabetic. He lives with his male partner, is a vegetarian, and takes atenolol which maintain his blood pressure at 138/74. Which risk factors should the nurse explore further with the client? Select all that apply

History of hypertension. Family heath history.

***During a left femoral artery arteriogram, the healthcare provider inserts an arterial sheath and initiate. Through the sheath to dissolve an occluded artery. Which interventions should the nurse implement? SATA

Instruct the client to keep the left leg straight Observe the insertion site for a hematoma Circle first noted drainage on the dressing

The nurse is teaching a client how to perform colostomy irrigations. When observing the client's return demonstration, which action indicated that the client understood the teaching?

Keeps the irrigating container less than 18 inches above the stoma


संबंधित स्टडी सेट्स

Interpersonal Effectiveness Chapter 1 A Framework for Interpersonal Skill Development

View Set

Review Questions #6: Oral Contraceptives

View Set

Financial Statement Analysis Review (Problems)

View Set

Commercial Property Ch. 13: Surety Bonds

View Set

Anatomy and Physiology chapters 1,6,7,8,9,10,11,12,13,14,15,16

View Set

Upper EXT Anatomy Chapter Assessment

View Set

Solving Systems of Equations by Elimination with Multiplication PART 1

View Set

Anterior compartment (leg): common A=dorsiflexion (DF)/ N=deep peroneal and Dorsal Muscles of Foot

View Set